Anda di halaman 1dari 108

Latihan Soal Bahasa Indonesia Kelas 9

Soal 1.1
Banyak orang menyebut bahwa Rudy Hartono merupakan maestro badminton dunia."
1. Makna kata 'maestro' dalam kalimat tersebut adalah...
A. orang yang hebat dalam bidang yang digeluti B. orang yang ahli dan berprestasi di bidang tertentu
C. orang yang mempunyai nama besar karena kesuksesannya D. orang yang menjuarai suatu
pertandingan dan tidak terkalahkan
Cermati kalimat dalam teks berikut ini:
Salah satu manfaat jeruk nipis adalah sebagai obat batuk. Kandungan buah ini bisa mengendalikan
otot-otot pernapasan sehingga dapat meredakan batuk. Adapun cara penggunaannya yaitu dengan cara
meminum air perasan dari jeruk nipis yang dapat dicampur dengan madu, kecap atau gula sehingga
rasa asamnya berkurang.
2. Ide pokok teks di atas adalah ....

A. manfaat jeruk nipis B. kandungan jeruk nipis C. campuran jeruk nipis D. cara penggunaan jeruk
nipis
3. Isi tersirat yang terkandung dalam teks di pertangaan nomor 2 di atas adalah .... A. Jeruk nipis dapat
dicampur dengan zat lainnya B. Jeruk nipis dapat ditanam di pekarangan rumah C. Jeruk nipis
memiliki khasiat untuk menjaga kesehatan D. Jeruk nipis lebih berkhasiat dibandingkan jeruk lainnya
Cermati kalimat dalam teks di bawah ini:
Lukisan karya Basuki Abdullah yang berjudul "Kakak dan Adik" menyajikan gambaran kesunyian
dan kesedihan. Gambaran kesunyian di lukisan tersebut diperkuat dengan penampakan ekspresi wajah
yang jernih, tetapi disertai tatapan mata kosong. Pakaian bersahaja dan berwarna gelap membuat
sosok kakak beradik dalam lukisan Basuki Abdullah itu tampak diliputi kesedihan."
4.Simpulan isi teks tanggapan tersebut adalah...
A. Kesunyian dan kesedihan digambarkan Basuki Abdullah dalam lukisan berjudul Kakak dan Adik.
B. Lukisan berjudul Kakak dan Adik karya Basuki Abdullah merupakan karya bertema persaudaraan.
C. Pakaian bersahaja yang tergambar dalam lukisan berjudul Kakak dan Adik mampu memunculkan
suasana sepi. D. Basuki Abdullah mengekspresikan perasaan sedih melalui tatapan kosong sosok
kakak beradik. 5. Cermati penggalan teks fabel berikut: .... Kura-kura lalu meminta bantuan kepada
monyet. "Maukah kau membantuku memetik buah pisang ini?" tanya kura-kura. "Aku bersedia, tetapi
buah pisang itu nanti dibagi dua," jawab monyet. "Baik!" jawab kura-kura. Monyet lalu memanjat
pohon pisang kura-kura. Bau harum buah pisang menggoda selera monyet. la lupa akan janjinya.
Kura-kura menunggu di bawah pohon pisang. "Nyet, Nyet, mana pisang bagianku?" teriak kura-kura.

"Sebiji pun tidak ada," jawab monyet rakus. "Nyet, ini pohon pisangku!" rengek kura-kura hampir
menangis. "Salah sendiri mengapa tidak bisa memanjat pohon!" ejek monyet. Raut wajah Kura-kura
memerah, ia mulai menangis. Hatinya sedih bercampur marah. la lalu menggoyang-goyang pohon
pisang itu. Tiba-tiba .... bruk! Pohon pisang itu tumbang. Monyet itu jatuh. Dia mengerang kesakitan.
Tubuhnya tertimpa batang pohon pisang. "Ampun kura-kura, tolong aku! Aku menyesal ...” kata
monyet. Tetapi, kura-kura sudah berlalu. la mencari sahabat baru. Pernyataan yang sesuai dengan
kutipan teks fabel di atas adalah ... A. Kura-kura bersedia membantu Monyet asalkan buah pisang
hasil petiknya dibagi dua. B. Kura-kura yang merasa sedih itu meninggalkan Monyet yang masih
berada di atas pohon. C. Monyet tidak memenuhi janji membagi pisang sehingga Kura-kura sedih dan
marah D. Monyet meminta Kura-kura menolong dirinya dan berjanji akan mengembalikan pisangnya.

6. Makna simbol "wajahnya merah" dalam penggalan teks fabel di pertanyaan nomor 5 adalah .... A.
malu B. marah C. terharu D. senang
7. Penyebab konflik yang diceritakan dalam penggalan teks fabel di pertanyaan nomor 5 adalah .... A.
Monyet mengingkari janji. B. Kura-kura tidak menepati janji. C. Kura-kura tidak mau berbagi hasil.
D. Monyet membagi hasil panen pisang.
8. Baca dan cermati kalimat yang menerangkan sejumlah peristiwa berikut:
(1) Pemilik kedai menawari Bagus semangkuk mi. (2) Bagus turun dari kereta api dengan berdesak-
desakkan. (3) Pemilik kedai merasa iba melihat keadaan Bagus. (4) Bagus merasa panik karena tidak
dapat membayar air mineral yang telah diminumnya. (5) Bagus tidak menyadari dompet yang ia
simpan di saku celananya dicuri.
Urutan peristiwa yang benar dari perincian kalimat di atas adalah.... A. (2), (3), (5), (1), (4) B. (2), (5),
(4), (3), (1) C. (3), (1), (2), (5), (2) D. (3), (2), (1), (5), (4)
9. Baca dengan cermat penggalan teks puisi di bawah ini:
dalam pengembaraanku yang ragu kulipat sisa-sisa harapan ketika pintu dan jendela menuju-Mu tak
lagi terbuka untukku
Pilihan kata yang tepat agar bait tersebut memiliki rima akhir yang sama adalah .... A. kata ragu
diganti dengan kata gamang B. kata harapan diganti dengan kata pilu C. kata kulipat diganti dengan
kata kusimpan D. kata terbuka diganti dengan kata tersisa
10. Baca dan cermati beberapa kalimat dalam teks iklan berikut:
(1) Nuvo ciptakan hari-hari Anda tanpa kuman. (2) Ikuti lomba dan workshop menyambut Bulan K3
Pertamina RU IV Balongan! (3) Selalu sediakan Energienis, penambah energi dan sahabat sarapan
pagi Anda! (4) Begitu banyak masalah akibat narkoba. Katakan Tidak untuk Narkoba!

Kalimat iklan yang bertujuan menyampaikan informasi yang mendidik masyarakat ditandai dengan
nomor .... A. (1) B. (2) C. (3) D. (4).
11. Berikut ciri-ciri Teks Prosedur, kecuali.... A. Menggunakan pola kalimat perintah (imperatif) B.
Menggunakan kata kerja aktif C. Menggunakan kata penghubung (konjungsi) untuk mengurutkan
kegiatan D. Menggunakan kalimat yang pendek
12. Judul yang paling tepat untuk sebuah Teks Prosedur adalah... A.
Empat Tips Memutihkan Kulit dengan Bahan Alami B.
Fungsi Kulit untuk Tubuh Manusia C.
Mengenal Struktur Kulit di Tubuh Manusia D.
Anatomi Lapisan Kulit dan Fungsinya Bagi Tubuh Manusia
13. Baca dan cermati penggalan teks di bawah ini:
Sebagai akibat ketentuan-ketentuan hasil KMB (Konferensi Meja Bundar), Indonesia harus
menanggung beban utang luar negeri dan dalam negeri dalam jumlah besar. Padahal, kondisi ekonomi
Indonesia kala itu masih belum pulih dari efek perang dengan Belanda. Pemasukan kas negara juga
baru terbatas dari sektor pertanian dan perkebunan. Di sisi lain, kas negara juga sedang defisit.
Paragraf di atas termasuk jenis teks apa?
A. Teks Eksplanasi B. Teks Eskposisi C. Teks Prosedur D. Teks Eksposisi
14. Berikut kata yang termasuk jenis konjungsi kausalitas (penyebaban), kecuali....

A. Setelah Itu B. Karena C. Oleh sebab itu D. Sehingga.


15. Kalimat majemuk hubungan akibat, ditandai oleh kata penghubung....
A. Dengan B. Seolah-olah C. Sehingga D. Padahal.

SOAL 1.2
1. Salah satu proses sosial disosiatif yang terjadi dalam masyarakat ialah kontravensi Contoh
kontravensi dalam masyarakat adalah …. A. dua kelompok pelajar terlibat tawuran akibat saling
mengolok B. pelajaran A memfitnah B agar tidak menang dalam pemilihan ketua kelas C. tim
bulutangkis Indonesia berhasil meraih juara umum dalam Sea Games D. atlet panahan
Indonesia berhasil mengalahkan atlet Filipina Jawaban: B

2. Berikut yang merupakan contoh mobilitas sosial vertikal adalah …. A. kantor Kinan bulan ini
menjalankan sistem rotasi pekerjaan ke daerah lain B. Galuh mendapatkan promosi jabatan di
kantor C. Hesa pindah tempat tinggal ke daerah lain D. Cahaya pindah kewarganegaraan
mengikuti suaminya (B)

3. Perhatikan paragraf berikut! Kerusakan alam di muka bumi semakin sulit ditahan. Lapisan
ozon di Kutub Utara dan Selatan semakin menganga akibat produksi emisi dari kegiatan industri.
Disadari atau tidak, kita sudah merasakan akibat dari kerusakan alam tersebut. Kita mengalami
hujan hampir sepanjang tahun. Banjir, tanah longsor, dan gagal panen pun terjadi di seluruh
pelosok negeri secara silih berganti. Kerusakan lapisan ozon juga berpengaruh terhadap
perubahan iklim dan siklus cuaca yang sangat ekstrem. Gagasan utama paragraf tersebut
adalah …. A. masalah kerusakan lapisan ozon di Kutub Utara B. akibat yang ditimbulkan dari
kerusakan lapisan ozon C. bencana banjir merupakan akibat dari rusaknya lapisan ozon D.
gagal panen terjadi karena pengaruh cuaca ekstrem Jawaban: ?

4. Perhatikan paragraf berikut

(1) Semua orang tua tentu berharap agar kelak anaknya menjadi orang sukses. (2) Indikator dari
kesuksesan biasanya dilihat dari pekerjaan. (3) Oleh sebab itu, banyak orangtua memilih
pendidikan yang tepat agar anaknya dapat bersaing di dunia kerja. (4) Pendidikan yang tepat
memang akan membuat peluang kerja pada masa depan semakin terbuka luas.
Kalimat utama paragraf tersebut ditandai dengan nomor ... A. 1 B. 2 C. 3 D. 4
Jawaban: ?

5. Proses pembuatan batik dengan cara melukis pada kain putih disebut dengan batik ...
A. cap B. tulis C. hias D. lukis Jawaban: ?

6. Perhatikan kedua teks berikut!


Teks 1 Ada seekor harimau tinggal di hutan. Di hutan itu, tidak ada hewan lain selain harimau
dan seekor serigala. Di sebelah hutan ini terdapat satu perkampungan kecil. Suatu waktu, ia
pergi ke dekat kampung untuk mencari makan. Harimau itu terus menerus melakukan itu hari
demi hari. Tiap hari ia memangsa hewan-hewan ternak milik penduduk desa.Penduduk desa
berusaha memburu harimau ini, namun mereka selalu gagal.

Teks 2 Burung-burung hering mendongakkan kepala dan tepat di depan mata telah berdiri
seekor badak. Ia memasang wajah yang seram dan menakutkan. Cula tajam di atas kepalanya
sangat menyeramkan. Burung-burung itu pun ketakutan sehingga mereka menjauhi badak.
Burung-burung hering terkejut saat melihat beberapa burung kecil hinggap di punggung badak.
Salah satu burung hering memberanikan diri bertanya.
Perbedaan penggunaan bahasa kedua teks tersebut adalah …
A. Teks 1 : terdapat kata tidak baku Teks 2 : tidak terdapat kata tidak baku
B. Teks 1 : terdapat kalimat pertentangan Teks 2 : terdapat kalimat sebab-akibat
C. Teks 1 : menggunakan ungkapan klasik Teks 2 : menggunakan ungkapan sehari-hari
D. Teks 1 : menggunakan kata bermakna kias Teks 2 : menggunakan kata bermakna lugas
Halaman Selanjutnya Jawaban: ?

7. Batik merupakan warisan lisan dan non bendawi yang telah ditetapkan oleh UNESCO
tepatnya pada tanggal ...
A. 2 oktober 2009 B. 4 oktober 2009 C. 5 oktober 2009 D. 1 oktober 2009 Jawaban: A

8. Dalam kegiatan perdagangan internasional seringkali suatu negara mengalami hambatan.


Hambatan perdagangan internasional adalah regulasi atau peraturan pemerintah yang
membatasi perdagangan bebas. Berikut ini manakah contoh hambatan perdagangan
internasional yang disebut kuota?

A. Indonesia membatasi tekstil dari India karena Indonesia juga memproduksi tekstil B.
Pemerintah Jepang menjual mobil ke luar negeri lebih murah daripada di dalam negeri C.
Pemerintah Indonesia mengenakan pajak yang tinggi untuk impor mobil mewah D. Pemerintah
Indonesia membeli bawang putih dari negara lain karena sangat dibutuhkan Jawaban: A

9. Globalisasi selain sebagai peluang ternyata juga dapat menjadi ancaman. Selain berdampak
positif terhadap perubahan juga dapat berdampak negatif. Diantara dampak negatif globalisasi
adalah… A. kesenjangan ekonomi B. meningkatnya efektivitas dan efisiensi C. taraf hidup
meningkat D. keterbukaan informasi yang semakin berkembang Jawaban: A

10. Tujuan pembentukan organisasi perdagangan Masyarakat Ekonomi Asean (MEA)


diantaranya yaitu ... A. menarik lebih banyak foreign direct investment B. meningkatkan
perdagangan antar negara anggota Asean C. menjadikan Asean sebagai tempat produksi yang
kompetitif sehingga produk Asean memiliki daya saing D. sebagai kawasan yang memiliki
perkembangan ekonomi yang merata dengan memprioritaskan pada UKM Jawaban: D

11. Persiapan BPUPKI diawali dengan pembahasan mengenai dasar Negara Indonesia
Merdeka. BPUPKI dibentuk dengan tujuan…
A. Untuk mempersiapkan kemerdekaan Indonesia
B. Merumuskan dasar negara untuk Indonesia merdeka
C. Membahas hal-hal penting yang berkaitan dengan Indonesia merdeka
D. Mempelajari dan menyelidiki hal-hal penting yang berhubungan dengan pembentukan Negara
Indonesia merdeka Jawaban: D

12. Organisasi perdagangan yang dibentuk tanggal 1 Januari 1995 adalah ...
A. WTO B. MEA C. AFTA D. MEE
Jawaban: ?

13. Perpindahan penduduk dari satu tempat ketempat lainnya dalam satu negara disebut ...
A. transmigrasi B. urbanisasi C. reboisasi D. migrasi Jawaban: D

14. Keberagaman suku, agama, ras, dan antar golongan dalam masyarakat Indonesia tidak
boleh menjadi penghalang terwujudnya persatuan dan kesatuan bangsa.
Sebagai warga negara Indonesia kita wajib ….
A. Saling menghormati dan toleran terhadap keberagaman B. Menghormati dan menjaga
kerukunan umat seagama C. Menjalin persaudaraan dengan orang suku sama D.
Menghilangkan segala macam perbedaan yang ada
Jawaban: A
15. Perdagangan internasional akan memberikan dampak positif bagi suatu negara. Dampak
tersebut diantaranya yaitu ...
A. pelaksanaan politik dumping B. munculnya pergeseran budaya
C. munculnya ketergantungan suatu negara D. munculnya devisa negara
Jawaban: D

Soal Latihan 1.3


Cermati teks diskusi berikut untuk menjawab soal nomor 1&2 !

Dampak Bermain Game Online pada Anak


Istilah game online sudah tidak asing lagi bagi anak-anak dan remaja masa kini. Dengan daya
jelajah internet yang sangat luas dan tanpa batas, game online sudah bisa dinikmati kapan pun
dan di mana pun. Dampaknya pun kini mulai dirasakan banyak pihak. ( .... ) Akan tetapi, banyak
juga pihak yang berpendapat bahwa game online juga memiliki sisi positif. Dengan demikian,
game online kini menimbulkan pro-kontra di masyarakat.

1. Kalimat yang tepat untuk melengkapi bagian yang rumpang dalam paragraf tersebut adalah...
A. Terutama para orang tua yang memiliki anak remaja yang setiap hari gemar bermain game
online.
B. Ada pihak yang beranggapan bahwa game online banyak menimbulkan dampak negatif. v
C. Karena itu, persoalan ini tentunya memerlukan perhatian dan pengawasan dari para orang tua.
D. Anak-anak dan remaja pun kini bisa bermain game online melalui telepon genggam mereka.

2. Variasi judul yang sesuai dengan isi teks tersebut di no 2 adalah...


A. Hubungan Game Online dan Anak di Masyarakat
B. Dampak Anak yang Bermain Game Online di Masyarakat
C. Pengaruh Bermain Game Online Bersama Anak
D. Pro dan Kontra Pengaruh Game Online pada Anak ….v

Cermati kutipan teks berikut!


Buku kumpulan cerpen senyum karyamin karya Ahmad Tohari terbitan PT. Gramedia
berisi 13 cerpen. Kesemua cerpen tersebut diramu dengan bahasa yang lancar dengan
selingan sealak-alek jawa. Lah-lho gusti pangeran dan lain-lain. Meskipuan begitu, buku
ini juga cocok untuk konsumsi orang lain selain keluarga jawa terutama orang yang ingin
mengetahui kultur tempat cerpen itu disampulnya bergambar seorang anak muda yang
sedang memakul kampak sebagai tanda optimisme kerja.

3. Tanggapan tersebut berisi tentang …..


A. isi buku dan permasalahnya. B. kemenarikan buku dan perwajahannya
C. bahasa dan perwajahan/sampul buku.. v D. judul buku dan pengarangnya

4. Kalimat pro di bawah ini adalah…


A. Saya senang sekali dengan adanya kegiatan pentas seni ini. v
B. Apa yang disampaikannya sama sekali tidak sesuai dengan hatiku.
C. Lebih baik ditinjau ulang kembali saran yang sudah disampaikan itu.
D. Pendapatmu itu sangat menentang aturan yang ada.
Cermati kalimat berikut!
Para hadirin dipersilakan mencari tempat duduk sesuai dengan nomornya.
5. Perbaikan kalimat tersebut agar efektif adalah...
A. Hadirin silakan mencari tempat duduk sesuai nomor. v
B. Hadirin dipersilakan duduk sesuai dengan nomornya.
C. Para hadirin dipersilakan mencari kursi sesuai nomornya.
D. Hadirin dipersilakan mencari tempat duduk dengan nomornya

Cermati kutipan teks berikut!


Husin adalah seorang anak dari keluarga yang sederhana. Orang tuanya tidak mampu untuk
membiayai Husin untuk melanjutkan pendidikan ke perguruan tinggi. Walaupun orang tuanya tidak
mampu membiayai dirinya ke perguruan tinggi, Husin tetap semangat belajar. Karena Husin tetap
semangat belajar dan tidak putus asa, Husin berhasil meraih beasiswa penuh untuk melanjutkan
pendidikan ke perguruan tinggi.
6. Pesan yang ingin disampaikan penulis dalam teks cerita inspiratif di atas adalah...
A. cari peluang lain dan tidak perlu berusaha melanjutkan pendidikan jika orang tua tidak mampu
B. cari pekerjaan saja jika sudah tidak ada kemampuan ekonomi untuk melanjutkan pendidikan
C. cari beasiswa dan mintalah bantuan ke pihak yang mampu untuk membantu biaya pendidikan
D. tetap semangat dan selalu berusaha menghadapi tantangan/masalah untuk meraih cita-cita
v

Cermati kutipan teks berikut!


Berkali-kali setiap hari kita memutar gagang untuk mengalirkan air bersih. Dengan menekan
gagang yang lain kita menyiram kotoran badan kita. Akan tetapi, seberapa seringkah kita
memikirkan sistem yang kompleks dalam penyediaan air dan pembuangan kotoran kota?
Sumber air kota biasanya berasal dari tempat yang jauh, tetapi sistem pembuangan kotorannya
memasuki perairan-perairan yang dekat. Jika pengaruh zat dalam ekosistem air tidak mampu
menanggulangi banyak buangan kotoran, pantai dan sumber-sumber dalam wilayah yang jauh
searah dengan aliran kota akan menjadi tercemar.

Cermati kutipan teks berikut!


Buku kumpulan cerpen senyum karyamin karya Ahmad Tohari terbitan PT. Gramedia berisi 13
cerpen. Kesemua cerpen tersebut diramu dengan bahasa yang lancar dengan selingan sealak-
alek jawa. Lah-lho gusti pangeran dan lain-lain. Meskipuan begitu, buku ini juga cocok untuk
konsumsi orang lain selain keluarga jawa terutama orang yang ingin mengetahui kultur tempat
cerpen itu disampulnya bergambar seorang anak muda yang sedang memakul kampak sebagai
tanda optimisme kerja.
7. Kritikan logis terhadap isi bacaan di atas adalah….
A. Seharusnya kita tetap harus memperhatikan sistem pembuangan kotoran..
B. Kita tidak harus mandi menggunakan air bersih.
C. Seharusnya kita tidak menggunakan sumber air dari pegunungan.
D. Sistem pembuangan kotoran memasuki perairan yang dekat sangat merusak lingkungan v

Cermatilah teks fabel berikut untuk menjawab soal nomor 8&9!


...Kura-kura lalu meminta bantuan kepada monyet.
"Maukah kau membantuku memetik buah pisang ini?" tanya kura-kura.
"Aku bersedia, tetapi buah pisang itu nanti dibagi dua," jawab monyet.
"Baik!" jawab kura-kura. Monyet lalu memanjat pohon pisang kura-kura. Bau harum buah pisang
menggoda selera monyet. la lupa akan janjinya. Kura-kura menunggu di bawah pohon pisang.
"Nyet, Nyet, mana pisang bagianku?" teriak kura-kura.
"Sebiji pun tidak ada," jawab monyet rakus.
"Nyet, ini pohon pisangku!" rengek kura-kura hampir menangis.
"Salah sendiri mengapa tidak bisa memanjat pohon!" ejek monyet.
Raut wajah Kura-kura memerah, ia mulai menangis. Hatinya sedih bercampur marah. la lalu
menggoyang-goyang pohon pisang itu. Tiba-tiba .... bruk! Pohon pisang itu tumbang. Monyet itu
jatuh. Dia mengerang kesakitan. Tubuhnya tertimpa batang pohon pisang.
"Ampun kura-kura, tolong aku! Aku menyesal ...” kata monyet.
Tetapi, kura-kura sudah berlalu. la mencari sahabat baru.

8. Pernyataan yang sesuai dengan kutipan teks fabel tersebut adalah...


A. Monyet meminta Kura-kura menolong dirinya dan berjanji akan mengembalikan pisangnya.
v
B. Kura-kura yang merasa sedih itu meninggalkan Monyet yang masih berada di atas pohon.
C. Kura-kura bersedia membantu Monyet asalkan buah pisang hasil petiknya dibagi dua.
D. Monyet tidak memenuhi janji membagi pisang sehingga Kura-kura merasa sedih dan marah.

9. Makna simbol wajahnya merah pada teks fabel tersebut adalah...


A. marah v B. terharu C. senang D. malu

Perhatikan kutipan teks drama berikut!


Ara : To, kamu dengar tidak, kalau kampung sebelah ada yang terkena virus Corona?
Anto : Ya, aku sempat dengar dari beberapa warga. Pak RT juga sudah sering sosialisasi tentang
waspada virus Corona.
Ara : Oh, yang cuci tangan pakai sabun dan pakai masker itu ya?
Anto : [....]
Ara : Wah, begitu yah? Itu sesuai juga dengan anjuran pemerintah ya?
Anto : Iya, Ra. Karena itu, sebaiknya kamu jangan dekat-dekat aku sekarang.
Ara : Hahaha, kamu bisa saja Ra.

10. Dialog yang tepat untuk melengkapi bagian rumpang kutipan teks drama tersebut
adalah...
A. Ya, kamu harus pakai sabun yang bagus dan makser yang mahal agar aman.
B. Jangan lupa bawa disinfektan dan pakai setiap kamu pegang benda atau bersentuhan dengan
orang
lain.
C. Bukan cuma itu, kita juga harus mengkonsumsi makanan sehat dan bergizi. Jangan lupa juga
untuk
jaga jarak atau social distancing. v
D. Selain itu, kamu juga jangan banyak berpergian. Lebih baik diam di rumah dan isolasi mandiri.

Cermati kutipan teks berikut!


Irfan anak dari keluarga yang kaya. Ia pandai di sekolahnya. Namun, ia tidak sombong
dan mau berteman dengan siapa saja. Sikap Irfan tersebut membuat teman- temannya
suka kepadanya. Irfan memang anak yang [ ...].
11. Ungkapan yang sesuai untuk melengkapi teks tersebut adalah ....
A. besar hati B. rendah hati v C. besar kepala D. hati emas

12. Dalam kutipan teks cerita di no. 32 di atas lebih menonjolkan unsur …
A. tema B. alur C. tokoh v D. latar
Bacalah kalimat-kalimat berikut dengan cermat!
(1) Mentega dan minyak mengandung lemak karena saat dioleskan ke kertas buram, kertas
tersebut berubah menjadi transparan.(2) Kita dapat mempunyai slime yang aman digunakan
tanpa menggunakan boraks setelah melakukan percobaan ini.(3) Ragi dikenal sebagai bahan
yang umum digunakan dalam fermentasi untuk menghasilkan etanol.(4) Manfaat percobaan ini
adalah dapat mengetahui perubahan berat kentang yang dimasukan pada larutan gula sebanyak
15 persen dan 50 persen, dan pada air biasa.

13. Kalimat kompleks (majemuk) yang mengandung hubungan penyebaban ditandai nomor ....
A. (1) v B. (2) C. (3) D. (4)

Bacalah kutipan teks dibawah ini!


Ayahku adalah seorang laki-laki yang lumayan tampan. Saat ini, usianya 35 tahun. Dia memiliki
tubuh atletis karena selalu berolah raga setiap hari. Dia memiliki tinggi 170 cm. Dia memiliki
rambut pendek hitam, mata yang hitam, berwajah oval dan dagu yang lancip. Dia memiliki kumis
dan jenggot yang tipis. Warna kulitnya sedikit coklat.
14. Informasi yang terdapat dalam teks deskripsi tersebut adalah ….
A. hobi ayah B.sosok ayah v C. keburukan ayah D. istri
ayah

Bacalah paragraf-paragraf berikut dengan cermat!

1. Melalui media cetak dan elektronik kita dapat menyaksikan bagaimana para pengedar narkoba
membangun jaringan secara intensif. Akibatnya, peredaran narkoba begitu merebak hingga ke
daerah-daerah pelosok bahkan sampai ke lembaga pemasyarakatan.
2. Mungkin banyak di antara hadirin yang belum memahami isu globalisasi. Untuk itu, kali ini saya
ingin mengajak hadirin untuk mengingat kembali beberapa pola hidup yang dapat mengakibatkan
globalisasi, apa pengaruhnya bagi kita semua, dan bagaimana kita menghadapi pengaruhnya.
Idealnya, biaya pembangunan di suatu negara harusnya dibiayai oleh rakyat di negara tersebut
melalui setoran pajak.
3. Inilah mengapa dikatakan pajak sebagai harga diri bangsa. Karena pajak sebagai harga diri
bangsa, seharusnya setiap wajib pajak merasa bangga setelah melakukan kewajibannya kepada
negara.
4. Langkah pertama adalah dengan memasukkan air hangat dengan takaran sesuai selera.
Selanjutnya, mencelupkan teh gantung ke dalam gelas berisi air hangat. Jangan lupa
memasukkan gula secukupnya kemudian aduk hingga larut. Teh siap di sajikan.

15. Paragraf-paragraf tersebut termasuk kutipan teks pidato persuasif, kecuali yang ditandai
dengan nomor ....
A. (1) B. (2) C. (3) D. (4) v

16. unsur cerpen yang berkaitan dengan tempat cerita berlangsung, kapan terjadinya cerita, dan
dalam keadaan bagaimana cerita tersebut terjadi disebut dengan…
A. Alur B. Tema C. Latar v D. Tokoh

Cermati paragraf berikut!


(1) Letusan atau erupsi gunung Merapi sangat dasyat hingga berdampak pada masyarakat.
(2) Pada saat musim hujan, akan sering terlihat warna-warni bercahaya di ujung cakrawala.
(3) Banjir berpotensi mengakibatkan kerusakan pada berbagai aspek kehidupan dan properti.
(4) Gempa tektonik disebabkan oleh pergeseran lempeng-lempeng tektonik secara mendadak.
17. Kata bergaris bawah yang bersinonim terdapat pada kalimat bernomor...
A. (4) B. (3) C. (2) D. (1) v

Cermati kutipan teks berikut!


Kebiasaan menyontek justru akan memadamkan semangat belajar. Para penyontek biasa berpikir
untuk menyiapkan bahan-bahan untuk membuat sontekan, nilai yang diperoleh pun kadang lebih
bagus dari siswa yang jujur. Nilai boleh saja bagus, tetapi sebenarnya ia tidak tahu apa-apa dan
tidak ada apa-apanya. Ilmu pengetahuannya tidak pernah bertambah karena rajin menyontek.
Lalu apa bedanya dengan mereka yang tidak pernah sekolah?

18. Simpulan isi penggalan teks tanggapan tersebut adalah...


A. Hindari menyontek karena tidak akan menjadikan kita pandai. v
B. Menyontek justru akan memadamkan semangat belajar.
C. Ilmu pengetahuan tidak akan bertambah karena menyontek.
D. Kebiasaan menyontek membuat tidak tahu apa-apa.

19. Contoh kalimat yang menyatakan hasil kegiatan percobaan adalah ....
A. Banyak lubang di sepanjang jalan itu.
B. Tidak lama lagi, musim hujan akan turun di daerah kita
C. Sepertinya perlu sistem pengelolaan sampah yang lebih baik dengan melibatkan banyak pihak.
D. Karena terkena cahaya, pertumbuhan tanaman itu, menjadi lebih baik v

20. Kalimat di bawah ini yang menggunakan kata hubung yang tepat adalah …
A. Lebih baik Shalat daripada tidur v
B. Ia salah satu pengusaha daripada yang berhasil
C.Orang tua daripada Abdullah mendapat kecelakaan
D.Bahasa daripada orang itu tidak dapat dimengerti

Cermati teks berikut!


Amir, Ridho, Ali, dan Surya tergabung dalam regu piket hari Kamis. Surya selalu datang terlambat
ke sekolah sehingga jarang melaksanakan piket pagi. Saat teman-temannya sibuk membersihkan
kelas, Surya juga tidak mau membantu.
21. Berdasarkan ilustrasi di atas, ungkapan yang tepat untuk menunjukan sifat Surya adalah...
A. berat tangan v B. panjang tangan C. ringan tangan D. cuci
tangan

Bacalah paragraf berikut untuk menjawab soal nomor 22,23 dan 24!
Untuk menghindari peredaran dan penyalahgunaan narkoba di kalangan peserta didik, kesadaran
dan pengetahuan tentang bahaya narkoba perlu ditanamkan. Banyak cara yang dapat dilakukan,
baik melalui penyampaian materi, forum diskusi, maupun dalam bentuk pemberian tugas-tugas
sekolah.Pemberian tugas sekolah dapat dilakukan guru dalam bentuk tugas membuat poster,
cerpen, atau tulisan yang bertema anti narkoba.
Dengan cara-cara di atassecara tidak langsung para peserta didik akan mengenal dampak
penyalahgunaan narkoba. Melalui pengenalan itu, terbentuklah pola pikir untuk menjauhi
penggunaan narkoba pada diri peserta didik.
22. Kutipan teks pidato tersebut bertopik ....
A. peredaran dan penyalahgunaan narkoba di indonesia
B. pemberantasan mata rantai peredaran narkoba
C. antisipasi penyalahgunaan narkoba di kalangan peserta didik v
D. pemberlakuan hukuman bagi pengedar narkoba
23. Kutipan teks pidato tersebut di no 22 bertujuan mengajak pembaca/pendengar untuk ....
A. membantu polisi dalam pemberantasan peredaran narkoba
B. mengantisipasi penyalahgunaan narkoba di kalangan peserta didik
C. mengajukan usul pemberlakukan hukuman seberat-beratnya kepada pengedar narkoba
D. berupaya aktif dalam upaya penanganan penyalahgunaan narkoba v

24. Struktur dalam teks diskusi adalah ....


A. Pendahuluan, Isi & Nasihat B. Pendahuluan, Isi & Simpulan v
C. Pendahuluan, Simpulan & Isi D. Simpulan, Isi & Nasihat

Cermati kutipan teks berikut!


Kegiatan membaca merupakan upaya dalam …..informasi dari bacaan hal tersebut merupakan
langkah awal dalam upaya memperkaya diri dengan ilmu pengetahuan langkah selanjutnya
adalah…..upaya pemahaman yang tepat terhadap isi bacaan.
25. Isian yang tepat adalah….
A. menyerap dan melakukan.. v B. menyerap dan dilakukan
C. menterapkan dan melakukan. D. penyerapan dan dilakukan

Cermati kutipan teks cerita inspiratif berikut !


”Bu..., aku memberi sedekah kepadanya sebanyak itu. Saat menerimanya, ia berucap hamdalah
berkali-kali seraya bersyukur kepada Allah. Tidak itu saja, ia mendoakan aku, mendoakan dirimu,
anak-anak, dan keluarga kita. Panjang sekali ia berdoa! Dia hanya menerima karunia dari Allah
Swt. sebesar 10 ribu saja sudah sedemikian hebatnya bersyukur. Padahal, aku sebelumnya
melihat di ATM saat aku mengecek saldo dan ternyata di sana ada jumlah yang mungkin ratusan
bahkan ribuan kali lipat dari 10 ribu rupiah. Saat melihat saldo itu, aku hanya mengangguk-angguk
dan tersenyum. Aku terlupa bersyukur, dan aku lupa berucap hamdalah.
26. Struktur cerita di atas termasuk ….A. Orientasi B. Resolusi v C. Komplikasi D.
koda

27. Contoh judul yang tidak tepat dari cerita inspiratif adalah …
A. Aku dan Impianku B. Cara Membuat Pepes Udang v
C. Sedekah Uang Sepuluh Ribu Rupiah C. Perjuangan Untuk Diriku Sendiri

Cermati teks berikut!


Pada saat kita tidur, baik siang maupun malam hari, selalu saja ada makhluk kecil bersayap
bernama nyamuk. Hal ini mungkin hal yang sepele bagi sebagian orang dan dapat diatasi dengan
menyemprotkan cairan pembasmi nyamuk yang banyak beredar di pasar. Namun, tahukah Anda
jika cairan pembasmi tersebut mengandung berbagai bahan kimia yang dapat membahayakan
kesehatan kita? Untuk mengatasi hal tersebut, kini terdapat cara alternatif yang lebih aman untuk
menangkap dan memerangkap nyamuk. Cara membuatnya cukup sederhana dan bahan untuk
membuatnya pun dapat kita temukan dalam kehidupan sehari-hari.

28. Judul yang tepat untuk melengkapi kutipan teks laporan hasil percobaan di atas adalah ....
A. Cara Membuat Perangkap Alternatif dan Aman
B. Cara Aman Membuat Cairan Pembasmi Nyamuk v
C. Cara Alternatif Membuat Nyamuk Terperangkap
D. Cara Membuat Perangkap Nyamuk Sederhana
Cermati kutipan teks berikut!
OSIS SMP PGRI Kalimulya mengadakan kegiatan gotong royong membersihkan parit, selokan,
dan sampah di sekitar lingkungan. Kegiatan ini bertujuan agar lingkungan sekitar tetap sehat dan
bersih.

29. Slogan yang sesuai dengan ilustrasi tersebut adalah …


A. Gotong royong menjalin kerja sama sesama warga.
B. Lingkungan bersih dan sehat tanggung jawab kita bersama. v
C. Mari bersama-sama bergotong royong di lingkungan sekitar.
D. Hidup sehat dan bersih merupakan cita-cita warga bersama.

Bacalah teks berikut dengan cermat!


(1) Apa yang telah dilakukan oleh para pendahulu kita sangatlah penting untuk diteladani.
(2) Mereka sadar bahwa kaum mudalah yang akan menentukan nasib bangsa ini. (3)
Untuk itulah, kita sebaga pemuda harus meneruskan semangat persatuan mereka. (4)
Ayo, wahai pemuda harapan bangsa, kita songsong masa depan yang lebih cerah untuk
nusa dan bangsa kita!

30. Kalimat emotif pada teks terdapat pada no .... A. (1) B. (2) v C. (3) D.
(4)

Cermati kutipan teks cerita inspiratif berikut !


Deggg...!!! Hati Budiman tergedor dengan begitu kencang. Rupanya wanita tadi sungguh berharap
tambahan sedekah agar ia dan putrinya bisa makan. Sejurus kemudian mata Budiman
membuntuti kepergian mereka berdua yang berlari menyeberang jalan, lalu masuk ke sebuah
warung tegal untuk makan di sana.
31. Struktur cerita di atas termasuk ….A. Orientasi B. Resolusi C. Komplikasi v D.
koda

Cermati kutipan teks dialog berikut!


Ulangan sedang berlangsung. Suasana benar-benar hening. Para siswa tampak fokus pada soal
yang dihadapinya. Pak Rudin duduk sambil sesekali melempar pandangan ke luar Ruang 09.
Bandi : Din, aku minta jawaban soal nomor 8 dan 9!
Andin : A dan D!
Ita : […. ]
Banu : 10 A, 11 D, nomor 15 aku belum!
Adi : Huss, jangan kencang-kencang nanti pengawasya dengar!
Ita : Soalnya sulit sekali, masih banyak yang belum aku kerjakan.

32. Kalimat yang tepat untuk melengkapi kutipan teks drama tersebut adalah...
A. Ulangan sedang berlangsung. Jangan rebut-ribut dong!
B. Kalau soal nomor 10, 11, dan 15 jawabannya apa, Ban? v
C. Soalnya sulit sekali, masih banyak yang sudah aku kerjakan
D. Memang betul jawabannya tidak sulit ?

Cermati kalimat berikut!


Para hadirin dipersilakan mencari tempat duduk sesuai dengan nomornya.
33. Perbaikan kalimat tersebut agar efektif adalah...
A. Hadirin silakan mencari tempat duduk sesuai nomor. v
B. Hadirin dipersilakan duduk sesuai dengan nomornya.
C. Para hadirin dipersilakan mencari kursi sesuai nomornya.
D. Hadirin dipersilakan mencari tempat duduk dengan nomornya
Cermati kutipan teks pantun yang rumpang berikut!
(1) Melihat perahu memasang layar
(2) ...
(3) Siswa baru sedang rajin belajar
(4) ...

34. Isian Larik/baris (no 2 dan 4) yang tepat untuk melengkapi bagian teks pantun tersebut
adalah...
A. (2) Pergi ke laut memancing ikan (4) Tidak lupa membaca buku
B. (2) Cuaca terang tiada terkira (4) Tetap semangat dan gembira
C. (2) Siap-siap juga membawa jaring (4) Tetap di rumah secara daring v
D. (2) Matahari terik memancar (4) Baca cerita diam di kamar

Cermati teks laporan hasil percobaan berikut!


Daun salam memiliki berbagai khasiat. Hal ini merupakan berita menggembirakan bagi orang-
orang yang kurang suka dengan obat-obatan kimiawi. Ada berbagai manfaat daun salam, antara
lain meningkatkan kesehatan jantung, mencegah stroke, menurunkan tekanan darah, dan
menurunkan kolesterol. [ ... ] Ternyata daun salam juga dapat mencegah berbagai macam kanker.

35. Kalimat yang tepat untuk melengkapi paragraf tersebut adalah ...
A. Ada beberapa langkah mengolah daun salam untuk mengobatan berbagai penyakit.
B. Selain manfaat tersebut, masih ada satu manfaat yang belum diketahui banyak orang. v
C. Penyakit yang bisa disembuhkan dengan daun salam adalah penyakit-penyakit ringan.
D. Para penderita cukup minum rebusan daun salam dalam takaran yang telah ditentukan

36. Majas personifikasi terdapat pada kalimat di bawah ini ....


A. Bulan menyapaku yang sedang termenung sendiri. v
B. Aku tak sudi berteman dengan orang yang sombong itu.
C. Dia memang anak yang tak bernyali besar.
D. Dia bagaikan tong kosong nyaring bunyinya.

37. Yang tidak termasuk Majas Hiperbola di bawah ini adalah…


A. Nina tukang bohong v
B. Adikku paling pintar sejagat alam raya ini.
C. Jantungku hampir copot saat melihat Agus lewat di depanku.
D. Langit seakan runtuh saat kudengar dia telah tiada.

Cermati kutipan teks berikut!

Teks 1 : Karimunjawa di Jepara berada di sebelah utara Pulau Jawa. Lokasi wisata bahari yang
satu ini sejak dulu sudah menjadi salah satu alternatif liburan yang sangat diinginkan oleh banyak
orang. Fasilitas di sana sangat lengkap. Bahkan, banyak juga backpacker yang berani untuk
berwisata ke sana dengan budget rendah.

Teks 2 : Kota Batu identik sebagai Kota Apel. Karena besarnya jumlah pengunjung untuk
berwisata di Kota Batu, potensi perkebunan apel juga dikembangkan sebagai objek wisata oleh
masyarakat sekitar. Saat ini, perkebunan apel yang tersebar di Kota Batu telah menjadi salah satu
objek wisata favorit, terutama bagi wisatawan lokal.

38. Persamaan isi kedua kutipan teks tersebut adalah ....


A. menjelaskan ketertarikan wisatawan luar negeri
B. membahas penelitian pengembang wisata
C. menggambarkan objek wisata domestic v
D. menguraikan jenis wisata bahari di Jawa
39. Kata yang bukan terdapat dalam kalimat aktif di bawah ini yaitu …
A. ditandai v B. memakan C. tertawa D. belajar

Cermati kutipan teks cerita inspiratif berikut!

Ada seorang sahabat menuturkan kisahnya. Dia bernama Budiman. Sore itu ia menemani istri dan
seorang putrinya berbelanja kebutuhan rumah tangga bulanan di sebuah toko swalayan. Usai
membayar, tangan-tangan mereka sarat dengan tas plastik belanjaan.

40. Struktur cerita di atas termasuk ….


A. koda v B. Resolusi C. Komplikasi D. Orientasi

Coba perhatikan paragraf di bawah ini dengan saksama!


Kerusakan alam di muka bumi semakin sulit dikendalikan saat ini. Lapisan ozon di Kutub Utara
dan Selatan kini semakin menganga akibat produksi emisi dari kegiatan industri. Disadari atau
tidak, kita sudah merasakan akibat nyata dari kerusakan alam tersebut. Contohnya, kita
mengalami hujan hampir sepanjang tahun.Banjir, tanah longsor, dan gagal panen mulai terjadi di
seluruh pelosok negeri secara silih berganti. Kerusakan lapisan ozon ini juga berpengaruh
terhadap perubahan iklim dan siklus cuaca yang sangat ekstrem di beberapa kawasan.

41. Apakah gagasan utama paragraf di atas….


A. Masalah kerusakan lapisan ozon di Kutub Utara
B. Akibat/efek yang ditimbulkan dari kerusakan lapisan ozon
C. Bencana banjir jadi akibat dari rusaknya lapisan zon
D. Gagal panen terjadi karena pengaruh cuaca yang ekstrem
Jawaban : B.

42. Coba perhatikan paragraf di bawah ini!


(1) Semua orang tua tentu memiliki harapan agar kelak anaknya jadi orang sukses. (2) Indikator
dari kesuksesan biasanya dapat dilihat dari pekerjaan.(3) Oleh karenanya, banyak orang tua
memilih pendidikan yang tepat agar anaknya dapat bersaing secara nyata di dunia kerja. (4)
Pendidikan yang tepat memang akan membuat peluang kerja di masa depan semakin terbuka.

Kalimat utama paragraf di atas ditandai dengan nomor berapa? A. 1 v B. 3 C. 2 D. 4

43. Perhatikanlah kedua teks berikut ini!

Teks 1 : Ada seekor harimau yang tinggal di hutan. Di hutan tersebut, tidak ada hewan lain selain
harimau dan seekor serigala. Di sebelah hutan terdapat sebuah perkampungan kecil. Suatu
ketika, harimau pergi ke dekat kampung untuk mencari makan. Harimau terus menerus
melakukan hal itu hari ke hari. Tiap hari ia memangsa hewan-hewan ternak milik penduduk di
desa tersebut. Penduduk desa mulai berusaha memburu harimau, namun mereka selalu gagal.

Teks 2 : Burung-burung hering mendongakkan kepala, namun tepat di depan mata telah berdiri
seekor badak. Badak selalu memasang wajah yang seram dan menakutkan. Cula tajam di atas
kepala badak terlihat sangatlah menyeramkan. Burung-burung itu merasa ketakutan sehingga
mereka menjauhi badak. Burung-burung hering terkejut saat melihat beberapa burung kecil yang
berani hinggap di punggung badak. Salah satu burung hering lalu memberanikan diri bertanya.
Letak perbedaan penggunaan bahasa kedua teks tersebut adalah …

A. Teks 1: Terdapat kata tidak baku Teks 2: Tidak terdapat kata tidak baku
B. Teks 1: Adanya kalimat pertentangan Teks 2: Adanya kalimat sebab-akibat
C. Teks 1: Menggunakan ungkapan klasik Teks 2: Menggunakan ungkapan sahari-hari
D. Teks 1: Menggunakan kata bermakna kias Teks 2: Menggunakan kata bermakna lugas
Jawaban : B.

44. Bacalah kalimat berikut dengan saksama!


Seorang pakar transportasi (…..) bahwa pemerintah lebih baik membangun infrastruktur
transportasi laut dan udara sebab Indonesia adalah negara kepulauan.
Apakah kata bentukan yang tepat untuk melengkapi kalimat di atas?
A. Menunjukkan B. Menyarankan C. Menanyakan D. Meningkatkan

45. Bacalah teks berikut ini dengan saksama!


(……) digital di depan ruang dingin vaksin menunjuk angka 3 derajat celcius. Angka tersebut
terbilang normal bagi ratusan ribu vaksin yang ada di dalamnya.Vaksin-vaksin tersebut tergolong
sensitif beku sehingga suhu di dalam cold room (ruang pendingi) harus stabil antara 3—8 derajat
celcius saja.
Apakah istilah yang tepat untuk melengkapi rumpang dalam kalimat di atas?
A. Inkubator B. Stetoskop C. Proyektor D. Termometer

46. Perhatikan kutipan teks berikut!


Demam berdarah dengue banyak menjangkit pasien di daerah tropis dan subtropis di
seluruh belahan dunia. Asia berada di urutan pertama dalam jumlah korban DBD setiap
tahunnya. Hal tersebut mungkin dipengaruhi juga oleh curah hujan di Asia yang
sangat tinggi, terutama di kawasan Asia Timur dan Selatan.
Jumlah penderita DBD setiap tahun meningkat, hampir 95% (sembilan puluh lima persen)
DBD menyerang anak di bawah usia 15 tahun.

Apakah simpulan yang tepat pada bacaan di atas?


A. Demam berdarah dengue banyak menyerang anak-anak di bawah usia 15.
B. Demam berdarah dengue banyak menyerang anak-anak di bawah usia 15 tahun di kawasan
Asia Timur dan Selatan saja.
C. Demam berdarah dengue banyak menyerang anak-anak rata-rata berusia 15 tahun khususnya
di daerah tropis dan subtropics. v
D. Demam berdarah dengue banyak menyerang anak-anak di bawah usia 15 tahun di daerah
tropis dan subtropis.

47. Baca teks berikut dengan saksama lalu kerjakan soal nomor 48 dan 49!
(1) Buku bertajuk “Tidur Berbantal Koran” adalah sebuah memoar atau autobiografi dari
pengarangnya, yakni N. Mursidi. Semua pengalaman N. Mursidi, mulai menjadi penjaja koran
hingga menjadi penulis serta wartawan ibu kota disajikan dengan jelas dalam autobiografi
tersebut.
(2) Cover penyajian buku tersebut pun cukup menarik. Penyajian tulisan mengalir dan diselingi
cerita-cerita pengalaman menarik dan lucu yang menggugah. Bahasa yang digunakan pun sangat
mudah dicerna. Melalui buku tersebut, pembaca diajak untuk menapaki jejak perjuangan penulis
yang awalnya begitu sulit menembus koran, namun berkat ketekunan dan kerja kerasnya akhirnya
berhasil menjadi penulis produktif yang dikenal.
(3) Satu-satunya kritik terhadap buku tersebut tertuju pada label yang diberikan oleh penerbit.
(4) Di sampul buku tepatnya di atas nama penulisnya tertera tulisan “Sebuah Novel”. Padahal
pada bagian pendahuluan di dalamnya disebutkan memoar.
(5) Singkatnya, memoar yang tersaji menarik dan ditulis dengan jujur dan apa adanya akan
mendorong pembaca untuk tidak menyerah dalam menggapai mimpi, walau seribu kesulitan dan
tantangan menghalangi.

Ditandai dengan nomor berapakah pernyataan yang berupa keunggulan pada teks di atas?
A. (1) B. (3) C. (2) D. (4)

48. Rumusan simpulan pada teks tersebut di no 47 yakni…


A. Satu-satunya kritik terhadap buku adalah pada judul.
B. Buku tersebut cocok untuk memotivasi orang yang ingin jadi penulis. v
C. Berkat ketekunannya, jejak perjuangan penulis ini dituangkan.
D. Semua pengalaman N. Mursidi ditulis dalam buku dengan judul Tidur Berbantal Koran.

49. Perhatikan urutan laporan percobaan berikut!


(1) Alat dan bahan (2) Tujuan (3) Hasil Pengamatan
(4) Prosedur / Cara Kerja (5) Simpulan

Urutan yang benar adalah…..a. 1,3,5,2,4 b. 2,5,4,1,3 c. 5,3,4,2,1 d. 2,1,4,3,5

50. Hikmah yang dapat diambil pelajarannya dalam cerita inspiratif disebut…

a. koda b. komplikasi c. orientasi d. resolusi

Soal Latihan 1.4


Bacalah kutipan teks dibawah ini!

Ayahku adalah seorang laki-laki yang lumayan tampan. Saat ini, usianya 35 tahun. Dia memiliki
tubuh atletis karena selalu berolah raga setiap hari. Dia memiliki tinggi 170 cm. Dia memiliki
rambut pendek hitam, mata yang hitam, berwajah oval dan dagu yang lancip. Dia memiliki kumis
dan jenggot yang tipis. Warna kulitnya sedikit coklat.

1. Informasi yang terdapat dalam teks deskripsi tersebut adalah ….


a. Hobi Ayah
b. Postur tubuh Ayah
c. Sifat Ayah
d. Warna kesukaan Ayah
Bacalah kutipan teks dibawah ini!

Ayahku adalah seorang laki-laki yang lumayan tampan. Saat ini, usianya 35 tahun. Dia memiliki
tubuh atletis karena selalu berolah raga setiap hari. Dia memiliki tinggi 170 cm. Dia memiliki
rambut pendek hitam, mata yang hitam, berwajah oval dan dagu yang lancip. Dia memiliki kumis
dan jenggot yang tipis. Warna kulitnya sedikit coklat.

2. Makna istilah dari kata atletis dan oval pada kutipan teks di atas adalah ….
a. Berotot dan lonjong
b. Sehat dan bulat
c. Berotot dan bulat
d. Sehat dan lonjong

(1) Setiap memandang wajahnya, hatiku terasa teduh. (2) Ya, wajah ibuku memang meneduhkan.
(3) Matanya yang bulat menyiratkan kasih sayang kepada orang lain. (4) Setiap ada masalah,
ibuku selalu menghadapinya dengan kepala dingin. (5) Tangannya yang mungil, menyuapi kami
untuk pertama kalinya dan menghapus air mata kami saat kami menangis. (6) Sifat rendah hatinya
selalu menginspirasi kami setiap hari.

3. Kalimat yang menggunakan makna istilah terperangah dengan tepat adalah ….


a. Nadin terperangah mengetahui kucingnya hilang.
b. Adik terperangah karena cicak yang tiba-tiba jatuh.
c. Intan terperangah ketika Adi menepuknya.
d. Ibu terperangah mengetahui aku menjadi penulis novel terkenal.

(1) Setiap memandang wajahnya, hatiku terasa teduh. (2) Ya, wajah ibuku memang
meneduhkan. (3) Matanya yang bulat menyiratkan kasih sayang kepada orang lain. (4) Setiap
ada masalah, ibuku selalu menghadapinya dengan kepala dingin. (5) Tangannya yang mungil,
menyuapi kami untuk pertama kalinya dan menghapus air mata kami saat kami menangis. (6)
Sifat rendah hatinya selalu menginspirasi kami setiap hari.

4. Kalimat yang bermajas metafora pada kutipan teks di atas terdapat pada nomor ….
a. (1) dan (3)
b. (4) dan (5)
c. (5) dan (6)
d. (6) dan (2)

5. Kalimat berikut yang menggambarkan ciri-ciri objek berdasarkan warna adalah ….


a. Sapi adalah hewan berkaki empat yang memiliki kelenjar susu.
b. Tubuh mereka berkulit putih dan berambut pirang.
c. Cicak memiliki kemampuan untuk memutuskan ekornya ketika dirinya merasa
terancam.
d. Rumah Dina berada di Gang Sahabat yang terletak sebelum pangkalan ojek.

6. Kalimat deskripsi yang menjelaskan ciri-ciri objek berdasarkan warna adalah ….


A. Bunga melati meski mungil namun terkenal sebagai simbol kesucian karena berwarna
putih bersih.
B. Si merah stroberi, meski rasanya asam manis tetapi anak-anak sangat menyukai buah ini.
C. Raja buah adalah durian, apalagi durian dari Pulau Sumatra yang sangat terkenal rasanya.
D. Buah paling Ajaib adalah kelapa karena tidak ada yang tau bagaimana air ada di
dalamnya.
7. Bacalah kalimat yang diambil dari penggalan teks deskripsi berikut!
Saat perjalanan menuruni tangga, kita disuguhkan pemandangan yang cukup asri. Banyak
monyet yang bergelantungan di dahan pohon.
Penggambaran objek di atas melibatkan citraan ….
A. perasaan
B. penciuman
C. penglihatan
D. pendengaran
8.Perhatikan kalimat deskripsi di bawah ini!

Rambutmu selembut sutera.” Kalimat tersebut merupakan kalimat dengan serapan


pancaindra….
A. Penglihatan
B. Pendengaran
C. Perabaan
D. Penciuman
9. Bacalah kutipan paragraf deskripsi di bawah ini!

Dari jauh sudah tercium wangi nasi goreng. Disajikan lengkap dengan ayam suwir, telur
dadar, dan acar makin menggunggah selera makan. Begitu dicicip, wow, perpaduan bawang
putih, kemiri, cabai, dan terasi sangat menggoyang lidah. Hati menjadi berbunga-bunga
menikmati seporsi nasi goreng kesukaanku.

Kalimat deskripsi penggambaran pengecapan terdapat pada kalimat ke ….


A. 1
B. 2
C. 3
D. 4
10.Bacalah teks di bawah ini!

Ketika berjalan di sepanjang pantai saat sore hari, Anda akan digoda oleh aroma ikan bakar
dari warung-warung makan di sana.

Pancaindra yang digunakan pada kalimat di atas adalah ....

A. penglihatan
B. pendengaran
C. penciuman
D. perabaan

11. Perhatikan ciri-ciri gurindam di bawah ini!

1) Satu nait memiliki dua baris.

2) Mengandung nasihat atau moral.

3) Padat dan singkat.

4) Menggunakan bahasa luhur.

5) Tema kehidupan sehari-hari.


6) Mengandung persoalan hidup.

8) Rima dan irama yang seimbang.

7) Bersajak a-a.

Di bawah ini yang termasuk gurindam adalah ….

A. Hidup manusia tanpa ilmu

Bagai taman tanpa bunga

B. Bersikapla jujur dan adil

Seperti matahari yang bersinar

C. Ciptakan perdamaian di hati

Seiring bunga indah mekar di taman

D. Pandai-pandai memilih kawan

Salah-salah menjadi lawan

12. Bagaimana irama dalam pantun biasanya di susun ….

A. A-A-B-B

B. A-B-C-D

C. A-B-A-B

D. B-B-A-A

13. Perhatikan pantun di bawah ini!

Ramai orang membeli jamu


Di bawah pokok cuaca redup
Bersungguh-sungguh mencari ilmu
Ilmu dicari penyuluh hidup
Apa tujuan utama dari pantun di atas ….
A. Menyampaikan cerita panjang.
B. Menghibur dengan lelucon.
C. Menyampaikan nasihat.
D. Menunjukkan kecerdasan penulis.

14. Cermatilah gurindam berikut!


Jika hendak mengenal orang mulia
Lihatlah kepada kelakuan dia
Maksud isi gurindam tersebut adalah ....
A. Kemuliaan seseorang bergantung pada derajat sosialnya.
B. Kemuliaan dan derajat seseorang terlihat dari kelakuannya sehari-hari.

C. Lihatlah kelakuan sehari-hari orang mulia yang kita kenal.

D. Kelakuan seseorang dilihat dari kehidupanya setiap hari.

15. Bacalah kutipan teks fantasi berikut.

Ivan menendang kerikil di jalan dengan kasar hingga terpelanting berhamburan. Debu
mengepul dari kerikil-kerikil itu. Lagi-lagi ia dijadikan bahan tertawaan! Ini semua gara-
gara kue basah Ibu! Setiap hari Ivan harus bangun pukul setengah empat pagi dan
membantu Ibu membuat aneka kue basah. Ivan juga harus pergi lebih pagi untuk
mengantarkan kue-kue itu ke beberapa warung menuju sekolah. Hal yang paling
memalukan, Ivan menitipkan kue itu juga di kantin sekolah! Ketika Fiam, anak paling
usil di kelasnya tahu, ia segera mengejek Ivan. Dan begitu Fiam memulai, julukan
“tukang kue” untuknya pun langsung diikuti teman-teman sekelas.
Apa yang menjadi fokus cerita pada kutipan tersebut ....
A. Ivan yang suka menendang kerikil.
B. Pengalaman Ivan di sekolah.
C. Pembuatan kue basah oleh ibu Ivan.
D. Kegiatan pagi Ivan membantu ibu.

16. Bacalah teks berikut dengan cermat!

Laila adalah seorang gadis miskin yang pandai. Sebagian waktunya ia gunakan untuk belajar
dan membantu orang tuanya. Selain itu, Laila juga suka menghabiskan waktunya untuk
menggambar.

Sayangnya, kini ia tidak dapat menggambar lagi karena pensil yang dimilikinya sudah hampir
habis dan sangat pendek sehingga tidak dapat digunakan lagi. Laila juga tidak bisa membeli
pensil baru karena tidak memiliki cukup uang.

Dalam kesehariannya, Laila membantu orang tuanya memunguti plastic yang ada di jalan.
Saat mengambil plastik, Laila menemukan ada sebuah pensil yang tergeletak di jalan. Laila
mengambilnya dengan senang karena akan dapat menggambar lagi sepulang mencari plastik.

Saat di rumah, Laila mulai mengeluarkan pensil yang ia temukan tadi di jalan. Laila mencoba
menggambar bunga di kertasnya. Alangkah kagetnya ketika selesai menggambar, tiba-tiba
menjadi bunga sesungguhnya dan tergeletak di atas kertas tempat ia menggambar.
Laila merasa kaget dan tidak percaya. Ia mulai menggambar ayam untuk memastikan apakah
yang ia lihat memang nyata. Sesaat setelah ia menggambar ayam, alangkah kagetnya tiba-tiba
di hadapannya ada seekor ayam hidup yang berkokok sangat kencang.
Kini ia menyadari bahwa ia memiliki sebuah pensil ajaib. Dengan sigap ia segera
menggambar berbagai keperluan yang dibutuhkan oleh keluarganya. Ia menggambar beras,
makanan, lauk-pauk, uang, dan berbagai kebutuhan lainnya.
Saat orang tua Laila datang, alangkah kagetnya mereka melihat rumah dipenuhi banyak
benda yang mereka butuhkan. Ibunya hampir menangis karena merasa sangat bahagia
kebutuhan mereka dapat tercukupi.
Meski begitu, Laila menggunakan pensil ajaibnya dengan bijak. Ia tidak sembarangan
menciptakan benda dengan pensil ajaibnya. Ia tahu bahwa bersikap berlebihan nantinya akan
menimbulkan petaka baik untuk dirinya maupun dengan keluarganya.

Tema yang sesuai dengan cerita fantasi di atas adalah…

A. Pensil Ajaib

B. Gambar Ajaib

C. Gadis miskin yang pandai

D. Tangan Ajaib Laila

17. Bacalah kutipan cerita fantasi berikut!

Gadis kecil bernama Marsha yang tinggal dengan ibunya ingin pergi ke hutan untuk mencari
jamur. Marsha mulai masuk ke dalam hutan. Tak lama kemudian Marsha tersesat dan
menemukan sebuah rumah kecil, yakni rumah seekor beruang. Marsha tak diperbolehkan
oleh si beruang untuk pulang ke rumah dan harus terus melayaninya. Marsha berusaha
memikirkan cara pulang. Dia membakar kacang polong dan berpura – pura meminta bantuan
beruang untuk mengantar kacang polong ke ibunya. Dia memindah kacang polong dan masuk
ke keranjang, dan si beruang membawanya ke Desa. Akhirnya Marsha dapat bertemu ibunya
kembali.

Alur yang digunakan pada kutipan cerita tersebut adalah…

A. Alur maju dan mundur

B. Alur mundur

C. Alur maju

D. Alur campuran

18. Bacalah cuplikan cerita fantasi berikut!

Minggu pagi yang cerah, Ardi, Handi, dan Dani berada di Candi Borobudur. Mereka
merupakan siswa pilihan dari sebuah SMP yang sedang melakukan tugas pengamatan
untuk karya ilmiah remaja. Di Tengah keramaian orang yang sedang berwisata, mereka
sibuk menyelesaikan laporannya.

Dalam cuplikan tersebut termasuk bagian dalam struktur teks cerita fantasi…

A. Abstrak

B. Orientasi

C. Komplikasi

D. Resolusi
19.Perhatikan kutipan teks cerita fantasi berikut ini!

“Dasar ibu tidak berguna, kau ibu yang sangat bodoh!” ucap Laila. (sambil melemparkan
pakaian ke lantai)
“Apa salah ibu?” ucap sang ibu. (sambil menangis)
“Salahmu adalah kau miskin, sehingga aku ikut miskin, dan menderita. Lihatlah teman-
temanku memiliki pakaian yang sangat bagus. Sedangkan aku hanya memiliki pakaian
jelek.” ucap Laila (sambil berlalu pergi)

Sang ibu ditinggalkan begitu saja dalam keadaan menangis. Tanpa terduga, tetes demi
tetes air mata yang keluar dari pipi sang ibu berjatuhan mutiara mengkilat.

Majas yang muncul pada kutipan teks cerita fantasi di atas adalah…
A. majas personifikasi
B. majas hiperbola
C. majas sarkasme
D. majas litotes

20.Perhatikan kutipan teks cerita fantasi berikut ini!


Pada suatu hari hiduplah seorang raja yang sangat tampan, dan populer bernama Pangeran
Haidar. Pangeran tersebut memiliki dua orang saudara laki-laki, bernama Pangeran Joko,
dan Pangeran Kendil. Mereka berdua sangat iri dengan ketampanan dan kepopuleran
Pangeran Haidar. Sehingga suatu hari, mereka memfitnah Pangeran Haidar dengan
menyebarkan berita buruk tentang Pangeran Haidar.

“Kalian perlu tahu, Pangeran Haidar bukanlah anak raja sesungguhnya.” ucap Pangeran
Joko kepada para dayang.
“Benarkah itu?” tanya para dayang.
“iya benar, saat itu Ratu Purbasari, ibu Pangeran Haidar telah mengkhianati Raja.” ucap
Pangeran kendil. (sambil tersenyum dalam hati)

Berita itu dengan cepat tersebar kepada masyarakat lainnya. Hingga sampai terdengar di
telinga Pangeran Haidar. Mengetahui hal itu, Pangeran Haidar hanya berdoa selama tujuh
hari, tujuh malam tanpa henti. Tiba-tiba muncullah petir di siang hari. Tak lama terdengar
suara menggema ke seluruh negeri, memberitahukan kebenaran yang sesungguhnya.

Tokoh utama pada teks cerita fantasi di atas adalah….


A. Ratu Purbasari
B. Pangeran Joko
C. Pangeran Kendil
D. Pangeran Haidar

21.Perhatikan kedua kutipan teks cerita fantasi berikut ini!

Teks cerita fantasi 1


“Dasar ibu tidak berguna, kau ibu yang sangat bodoh!” ucap Laila. (sambil melemparkan
pakaian ke lantai)
“Apa salah ibu nak?” ucap sang ibu. (sambil menangis)
“Salahmu adalah kau miskin, sehingga aku ikut miskin, dan menderita. Lihatlah teman-
temanku memiliki pakaian yang sangat bagus. Sedangkan aku hanya memiliki pakaian
jelek.” ucap Laila (sambil berlalu pergi)

Teks cerita fantasi 2


“Ibu kau sedang apa?” ucap Bella. (berjalan menghampiri ibunya)
“Ibu sedang membuat kue untuk berjualan esok hari nak,” jawab ibu.
“Baiklah, biar aku bantu ibu.” ucap Bella.
“Kau baru saja pulang sekolah, istirahat saja dahulu, kau pasti lelah.” ucap ibu

Perbandingan karakter tokoh pada kedua teks cerita fantasi di atas adalah….
A. Tokoh Laila memiliki karakter protagonis, sedangkan Bella memiliki karakter
antagonis.
B. Tokoh Laila memiliki karakter antagonis, sedangkan Bella memiliki karakter
protagonis.
C. Tokoh Bella dan Laila memiliki karakter antagonis.
D. Tokoh Bella dan Laila memiliki karakter protagonis.

22. Rara menanyakan apakah saya dapat membantunya dalam mengerjakan PR.
Kalimat langsung yang tepat untuk kalimat tersebut adalah…
A. Rara bertanya, “Apakah saya dapat membantu dalam mengerjakan PR?”
B. Rara bertanya, “Apakah saya dapat membantunya dalam mengerjakan PR?”
C. Rara bertanya, “Apakah kamu dapat membantu saya dalam mengerjakan PR?”
D. Rara bertanya, “Apakah kamu dapat membantunya dalam mengerjakan PR?”

23. Penulisan kalimat langsung yang benar adalah…


A. “Apa kabar, Paman?” Sapa Diman sambil menjabat tangan pamannya.
B. Pak Lurah mengatakan, “bahwa terhadap narkoba” perlu digalakkan.
C. “Guru berkata, Lanjutkan pekerjaan kalian dan segera dikumpulkan.”
D. “Sudah larut malam mengapa belum tidur?” jawan ayah kepadaku.

24. “Selanjutnya, tuangkan minyak ke dalam wajan”.


Pada kutipan teks prosedur diatas penggunaan kata transisi yaitu….
A. Selanjutnya
B. Tuangkan
C. Ke dalam
D. Wajan

25. Simak teks Prosedur di bawah ini:



Saat memasukkan Kartu SIM ke ponsel, pastikan ponsel telah dimatikan dan ikuti petunjuk di
bawah ini:
1. Tekan pengunci ponsel bagian belakang dan geser penutup lalu lepaskan penutup dari
ponsel.
2. Setelah itu, dorong dua pengunci ke arah yang berlawanan dan lepaskan baterai.
3. Selanjutnya, geser kartu SIM dengan hati-hati ke dalam slot dan pastikan penyambungan
sinyal yaitu bagian emas telah menghadap ke konektor ponsel.
4. Kemudian, letakkan baterai dan luruskan sampai terkunci pada tempatnya.
5. Terakhir, masukkan kedua tangkai slot pengunci belakang sesuai slot di ponsel dan geser
penutup depan tombol telepon sampai terkunci pada tempatnya.
Yang bukan termasuk kata Transisi pada teks di atas adalah...
A. Masukkan
B. Setelah itu
C. Terakhir
D. Kemudian

26. Bacalah contoh kutipan teks Prosedur di bawah ini:



Pasangkan kedua tangan dengan posisi tangan kanan di atas hingga jari-jari tangan dalam
keadaan saling mengunci, .... gosoklah secara perlahan dan ulangi gerakan tersebut dengan
posisi tangan kiri di atas.
Kata Transisi yang tepat untuk mengisi titik-titik di atas adalah..
A. akhirnya
B. lalu
C. dan
D. sehingga

27. Perhatikan kutipan teks Prosedur di bawah ini:



Setelah dicuci bersih, olesi jagung dengan mentega dan kecap manis pedas. Setelah jagung
dibumbui, bakar jagung di atas api.
Kalimat yang tepat setelah mengalami perbaikan dengan pelesapan adalah...
A. Cuci jagung hingga bersih, olesi dengan mentega dan kecap manis pedas, lalu bakar
di atas api.
B. Cuci hingga bersih, olesi, lalu bakar.
C. Setelah dicuci, olesi jagung dengan mentega dan kecap. Jagung dibumbui, bakar
jagung di atas api.
D. Setelah dicuci bersih, olesi mentega dan kecap manis pedas. Setelah dibumbui, bakar
di atas api.

28. Perhatikan langkah-langkah menginstal aplikasi di gawai berikut ini!

(l) Buka aplikasi playstore dan cari aplikasi media sosial.

(2) Dilarang menginstal aplikasi selain dari aplikasi playstore!

(3) Pilih aplikasi yang ada di playstore dan instal.

(4) Tunggu sampai aplikasi selesai terpasang di gawai kamu.

(5) Jangan matikan gawai ketika aplikasi masih dalam proses instal!

(6) Setelah terpasang, sebaiknya buka aplikasi menggunakan akun sosial mediamu sendiri.
Dari langkah-langkah tersebut, manakah yang termasuk kalimat larangan?

A. (1) dan (3)

B. (2) dan (4)

C. (2) dan (5)

D. (5) dan (6)

29. Kalimat perintah di bawah ini yang paling tepat dalam membuat donat adalah .…

A. Gunakan tepung yang mengandung protein tinggi.

B. Jangan membuka adonan selama proofing!

C. Sebaiknya goreng dengan minyak banyak dan api kecil untuk mendapatkan warna donat
yang cantik.

D. Jangan membolak balik adonan selama menggoreng!

30. Perhatikan kutipan teks prosedur berikut!


Paspor adalah dokumen resmi yang dikeluarkan oleh pejabat yang berwenang dari
suatu negara yang memuat identitas pemegangnya dan berlaku untuk melakukan
perjalanan antar negara. Bagaimana cara mengurus paspor? Berikut ini cara mengurus
paspor dengan baik dan benar:
Kutipan teks prosedur di atas, termasuk struktur teks prosedur bagian?
A. Alat/bahan
B. Penegasan ulang
C. Langkah-langkah
D. Tujuan

B. ISIAN
31. Bacalah penggalan teks deskripsi berikut!

Grojogan sewu adalah salah satu objek wisata air terjun alami di Tawangmangu,
karanganyar, jawa tengah. Dari pintu masuk, kita harus menuruni seribuan anak tangga untuk
menuju air terjun.

Saat perjalanan menuruni tangga, kita disuguhkan pemandangan yang cukup asri. Banyak
monyet yang bergelantungan di dahan pohon. Ada juga yang duduk atau berlairian di sekitar
anak tangga.

Isi dalam penggalan teks deskripsi di atas adalah…

Jawaban : Perjalanan menuju air terjun Grojogan Sewu


32. Bisikan angin membelai daun-daun pohon.
Majas yang digunakan dalam kalimat di atas adalah…
Personifikasi
33. Perhatikan pantun berikut ini!
Tangan dicuci menjadi wangi,
………………………………
Apalah arti punya ilmu tinggi,
Kalau hati setinggi langit.
Lengkapilah sampiran dari pantun tersebut!
Jawaban: Kaki dicuci tangan berkelit
34. Perhatikan pantun berikut ini!
Jalan-jalan ke kampung jati
Jalan dikenang sepanjang masa
Orang tua jangan disakiti
……………………………….
Lengkapi larik keempat pada pantun tersebut!
Jawaban: Takut kita mendapat dosa
35. Perhatikan gurindam berikut ini!
Jaga hati jauhi dengki dendam
Hidup menjadi lebih tentram
Makna gurindam tersebut adalah….
Jawaban : kita harus menjauhi diri dari dendam, agar hidup kita tentram.
36. Perhatikan kutipan gurindam berikut ini!
Jauhi hati dari buruknya prasangka
………………………………….
Lengkapilah larik kedua pada gurindam tersebut!
Jawaban : Agar hati tidak tersiksa

37. …
Mendadak, ekor Nataga mengeluarkan api besar. Nataga mengibaskan api pada ekornya yang
keras, membentuk lingkaran sesuai tanda yang dibuat oleh semut, rayap, dan para tikus.
Kutipan Cerita di atas termasuk jenis Cerita fantasi… total

38. Udara pagi hari terasa sangat segar


Suasana pagi hari membuat rasa tenang dan nyaman
Suasana dan pemandangan yang sangat indah
Dada terasa sesak ketika membayangkan musibah yang dialami orang di luar sana.

Kutipan cerapan di atas menggunakan panca indra seolah-olah…


Jawaban: Merasakan

Mematikan Komputer dengan Benar


1) Tutuplah semua aplikasi yang Anda gunakan.
2) Klik menu Start (XP)/Logo Windows (7) di pojok kiri bawah.
3) Pilih Shut down dan tunggu beberapa saat hingga komputer Anda benar-benar mati.
4) Setelah komputer benar-benar mati, kemudian tekan tombol pada monitor dan
speaker, stabilizer dan perangkat komputer lainnya.
5) Setelah itu, sebaiknya cabut kabel dari stop kontak. Hal ini untuk menghemat daya
dan mengantisipasi terjadinya korsleting listrik.
39. Dari kutipan teks di atas, kalimat yang berisi saran adalah ….

Baca baik-baik !

Mematikan Komputer dengan Benar


1) Tutuplah semua aplikasi yang Anda gunakan.
2) Klik menu Start (XP)/Logo Windows (7) di pojok kiri bawah.
3) Pilih Shut down dan tunggu beberapa saat hingga komputer Anda benar-benar mati.
4) Setelah komputer benar-benar mati, kemudian tekan tombol pada monitor dan
speaker, stabilizer dan perangkat komputer lainnya.
5) Setelah itu, sebaiknya cabut kabel dari stop kontak. Hal ini untuk menghemat daya
dan mengantisipasi terjadinya korsleting listrik.

40. Dari kutipan teks prosedur di atas, kalimat yang berisi keterangan tujuan adalah ….

C. URAIAN
41. Bacalah teks deskripsi berikut !
Pantan Terong yang Instagramable
Rafa menghabiskan akhir pekan bersama keluarganya. Kali ini mereka berwisata ke
sebuah puncak yang sedang populer di Aceh, yaitu Pantan Terong. Rafa tak menyesal
mengusulkan destina destinasi wisata ini kepada keluarganya. Ia sering membacanya di
internet. Kali ini ia dapat menyaksikan keindahan tempat ini dengan matanya sendiri. Setiba
di rumah, ia pun segera menuliskan pengalamannya ini di blognya.
Kami berangkat dari Banda Aceh pukul 01.00 siang. Pukul 08.00 malam kami tiba di
rumah Paman di Kota Takengon. Setelah makan malam, Paman menyuruh kami bergegas
tidur. Kami akan pergi segera setelah salat subuh. Siapa tahu kami bisa menyaksikan
matahari terbit di Pantan Terong!

Bagaimana objek yang dideskripsikan dalam tulisan Rafa di atas?


Jawaban : Objek yang dideskripsikan dalam tulisan Rafa adalah tempat wisata Pantan Terong
yang instagramable.

42. Perhatikan pantun rumpang berikut!

Anak ayam turun sepuluh


Mati satu tinggal Sembilan
…………………………….
…………………………….
Sumber : https://www.dapurimajinasi.com/2021/11/20-soal-melengkapi-pantun-dan.html
Lengkapilah pantun rumpang di atas!

Jawaban : Tuntutlah ilmu dengan sungguh-sungguh.

Supaya kamu tidak ketinggalan.

43.Gurindam memiliki 2 bait yang terdiri dari 8 sampai 14 suku kata dengan larik pertama
merupakan syarat dan larik kedua merupakan jawaban.
Buatlah satu gurindam berdasarkan ciri-ciri tersebut…..

Sebelum bekerja pikir dahulu

Agar pekerjaan selamat selalu

44.Kata Abdi, “coba kamu membantu saya menyelesaikan tugas ini!”. Kalimat tersebut kurang
tepat, perbaiki kalimat langsung tersebut…….
“Coba kamu membantu saya menyelesaikan tugas ini!”
Es Kopi Susu
1) Masukkan gula dan kopi ke dalam gelas.
2) Lalu tuangkan air hangat kemudian aduk kopi hingga larut.
3) Tambahkan susu atau creamer, kemudian aduk hingga merata.
4) Diamkan beberapa saat hingga kopi tidak terlalu panas.
5) Terakhir, masukkan es batu secukupnya.
6) Es kopi susu siap untuk dinikmati.

45. Tulislah tujuan dari teks prosedur di atas!


Teks prosedur di atas bertujuan untuk memberitahukan langkah-langkah dalam membuat
es kopi susu

SOAL PENILAIAN AKHIR SEMESTER


MGMP BAHASA INDONESIA
KELAS 9
PAKET A

PILIHAN GANDA

1. Bacalah teks berikut dengan cermat!

(1) Fitri yakin, yang mengambil uangnya bukan Tineke walaupun uangnya ditemukan di tas
Tineke. Tineke berusaha tidak terpancing dengan ejekan teman-temannya dan menenangkan
hati dengan membaca buku. la ingin sekali masalah ini cepat selesai tetapi ia tidak ingin
menyusul Fajar yang sedang bersarapan di kantin, ia tetap menunggu di kelas.

(2) Buatlah sketsa gambar terlebih dahulu pada kertas dengan menggunakan jangka dengan
diameter 8 cm, kemudian gunting sesuai pola jangka tersebut. Tempelkan pada kain perca
lalu jiplak polanya dan gunting kain percanya, ulangi sampai 60 bulatan. Jahit pinggirnya lalu
tarik kemudian di tali, ulangi sampai 60 bulatan. Lalu, rangkaikan bulatan satu dengan
bulatan yang lainnya, bentuk pola seperti taplak meja. jahit bulatan satu dengan yang lainnya.
Rapihkan jahitannya agar terlihat lebih indah.

(3) Saat ini kenakalan remaja marak terjadi di masyarakat. Tawuran antar pelajar serta
pergaulan bebas menjadi salah satu contoh kenakalan remaja. Kenakalan tersebut tentunya
menimbulkan keresahan di masyarakat. Selain itu, kenakalan tersebut menimbulkan perilaku-
perilaku yang menyimpang sehingga menimbulkan tindakan-tindakan yang anarkis.

(4) Terlepas dari kekurangan tersebut, nilai plus yang ditawarkan oleh film ini jauh lebih
besar. Secara keseluruhan, film “Sabtu Bersama Bapak” telah berhasil dan layak ditonton
bersama keluarga. Lewat film ini, penonton akan memahami betapa kebersamaan keluarga
adalah sesuatu yang begitu penting.

1. Paragraf yang berisi kutipan laporan percobaan ditandai oleh nomor ....

A. (1)

B. (2)

C. (3)

D. (4)

Perhatikan Tabel di bawah ini !


Tabel Pengamatan

No Jumlah Baterai Beda Potensial (volt) Nyala Lampu


.

1. 1 baterai 5 redup

2. 2 baterai 10 terang

3. 3 baterai 15 sangat terang


2. Pernyataan yang sesuai dengan informasi tentang nyala lampu pada tabel tersebut
adalah . . .

A. Semakin sedikit jumlah baterai yang digunakan, maka nyala lampunya akan
semakin redup.

B. Semakin banyak jumlah baterai yang digunakan, maka nyala lampunya akan
semakin redup.

C. Semakin sedikit jumlah baterai yang digunakan, maka nyala lampunya akan
semakin terang.

D. Semakin banyak jumlah baterai yang digunakan, maka nyala lampunya akan
semakin terang.

Perhatikan langkah-langkah teks laporan percobaan di bawah ini!

1) Peras jus lemon ke dalam mangkuk lalu tambahkan beberapa tetes air ke dalam
mangkuk tersebut.

2) Kemudian aduklah air dan jus lemon tadi dengan memakai sendok. Celupkan
cotton bud ke dalam campuran lalu tulis pesan diatas kertas putih.

3) Tunggulah sampai jus tersebut kering sehingga tidak terlihat. Guna membaca
pesan itu dapat dilakukan dengan memanaskan kertas yang dipegang dengan
mendekatkannya pada bola lampu yang menyala.

4) Jus lemon ialah senyawa organik yang bisa teroksidasi dan bisa berubah warna
menjadi coklat saat dipanaskan. Bahan yang diencerkan ini membuat pesan sulit
untuk dibaca.
3. Kesimpulan tujuan dari laporan percobaan di atas adalah ….

A. membuat tinta pulpen supaya tidak terlihat

B. membuat pembuatan cotton bud

C. mengolah minuman yang terbuat dari buah lemon

D. membuat tinta pupen

Baca baik-baik kutipan laporan percobaan di bawah ini !

Air jeruk merupakan senyawa asam lemah sehingga menghasilkan reaksi


gelembung yang sedikit dan tidak adanya nyala lampu.

4. Frase benda pada penggalan laporan percobaan di atas yaitu …

A. merupakan senyawa

B. air jeruk

C. tidak ada

D. menghasilkan reaksi

5. .Perhatikan kutipan teks laporan berikut!

…. . Jeruk memiliki banyak jenis dan setiap jenisnya memiliki karakteristik


tersendiri. Ada buah jeruk yang memiliki segmen yang bisa dipisahkan, [....] ada
juga yang tidak bisa dipisahkan. Buah ini memiliki kandungan vitamin C yang
cukup tinggi. Vitamin ini sangat baik untuk mendukung kesehatan gusi serta
mempertahankan daya tahan tubuh yang lebih kuat. ….

https://apps.detik.com/detik/

Yang termasuk kata benda abstrak dalam kutipan teks laporan tersebut adalah ...

A. jeruk
B. vitamin

C. kesehatan

D. segmen

Soal nomor 6

6. Perhatikan kutipan teks laporan berikut!

[1] Komodo merupakan hewan sejenis reptil besar yang berkaki empat dan
wujudnya menyerupai kadal. [2] Bedanya, ukuran hewan ini sangatlah besar. [3]
Hewan langka ini sudah masuk kategori dilindungi. [4] Pemerintah, melalui Dinas
Pariwisata sudah mencanangkan bila hewan ini menjadi satwa nasional bagi
negeri ini. …. .

https://apps.detik.com/detik/

Bagian pernyataan umum dalam kutipan teks laporan tersebut ditunjukkan oleh nomor
... .

A. [1]

B. [2]

C. [3]

D. [4]

7. Perhatikan kutipan teks laporan berikut!

[1] Komodo merupakan hewan sejenis reptil besar yang berkaki empat dan
wujudnya menyerupai kadal.. [2] Bedanya, ukuran hewan ini sangatlah besar. [3]
Hewan langka ini sudah masuk kategori dilindungi. [4] Pemerintah, melalui Dinas
Pariwisata sudah mencanangkan bila hewan ini menjadi satwa nasional bagi
negeri ini. … .

https://apps.detik.com/detik/

Penggunaan konjungsi yang tepat pada kalimat {4] adalah ... .

A. karena

B. bahwa

C. jika

D. supaya
8. Perhatikan kutipan teks laporan berikut!

…. . [1] Jeruk memiliki banyak jenis dan setiap jenisnya memiliki karakteristik
tersendiri. [2] Ada buah jeruk yang memiliki segmen yang bisa dipisahkan, [ .... ]
ada juga yang tidak bisa dipisahkan. [3] Buah ini memiliki kandungan vitamin C
yang cukup tinggi. [4] Vitamin ini sangat baik untuk mendukung kesehatan gusi
serta mempertahankan daya tahan tubuh yang lebih kuat. …. .

https://apps.detik.com/detik/

Konjungsi yang tepat untuk melengkapi bagian yang rumpang pada kutipan teks
laporan tersebut adalah ...

A. sehingga

B. kemudian

C. oleh sebab itu

D. namun

9. (1) Sukses adalah impian semua orang. Kesuksesan bisa digapai dengan seribu cara. (2)Tapi
untuk bisa menjadi orang yang sukses, tentunya dibutuhkan kerja keras, doa, dan juga strategi
yang cerdas. (3)Salah satu pintu menuju kesuksesan adalah dengan menjadi wirausaha.
(4)Ubahlah hidup dengan langkah yang berani. (5)Keputusan yang besar akan membuat
perubahan besar dalam hidup Anda.

Kalimat yang merupakan kalimat aktif dalam paragraf tersebut ditandai dengan nomor…
A. (1)
B. (2)
C. (4)
D. (5)

10. Laporan ini disusun untuk memberikan laporan atas kegiatan percobaan yang telah kami
lakukan. Kegiatan yang kami lakukan adalah menguji kandungan gizi pada beberapa jenis
makanan tradisional.

Kutipan teks tersebut di atas terdapat pada bagian ....


A. Tujuan percobaan
B. Alat dan bahan
C. Objek percobaan
D. Simpulan percobaan

11. Daun pandan suji digunakan sebagai indikator alami karena dapat menunjukkan sifat asam dan
basa suatu larutan. Sebelum menjadi ekstrak larutan daun pandan suji yang siap digunakan,
daun pandan suji ditumbuk hingga halus. Setelah itu, ditambahkan larutan etanol pada
tumbukan daun pandan suji dan disaring dengan kertas saring. Larutan daun pandan suji
berwarna hijau. Tujuan penyaringan tumbukan daun pandan suji untuk melarutkan zat •zat
yang terkandung dalam daun pandan suji. Jika ditambahkan reagen akan menunjukkan hasil
berupa perubahan warna.

Kalimat di atas termasuk ke dalam….


A. Tujuan percobaan
B. Tempat percobaan
C. Bahan percobaan
D. Bagian uraian

12. Jamur merupakan salah satu tumbuhan pengurai yang mampu hidup dalam berbagai kondisi
alam. Salah satu uji coba yang kami lakukan pada fungsi amanita adalah memberikan tetesan
merkuri. Perlakuan tetesan merkuri ini dilakukan untuk mengetahui seberapa jauh
kemampuan hidup dari jamur amanita.

Teks di atas termasuk ke dalam….


A. Pernyataan umum
B. Tujuan
C. Deskripsi bagian
D. Pernyataan argumen

13. Perhatikan kutipan berikut!


Berdasarkan hasil uji hedonik pada jenis abon, abon yang dibuat dengan metode
tradisional lebih disukai daripada abon yang dibuat dengan metode modern.
Tekstur abon tradisional cenderung lebih kasar daripada abon modern. Perbedaan
tekstur abon dimungkinkan karena penggunaan mesin penggiling yang membuat
serat daging terpisah seluruhnya. Selain itu, warna abon dipengaruhi oleh
banyaknya penggunaan gula dan lama penggorengan abon. Umumnya abon yang
baik dicirikan dengan warna cokelat kekuningan. Abon berwarna selain cokelat
memang kurang disukai.

Pernyataaan Umum yang terdapat pada teks di atas adalah….


A. Berdasarkan uji hedonik pada jenis abon menyatakan bahwa abon yang dibuat
dengan metode tradisional lebih disukai dibandingkan metode modern. Tekstur
abon tradisional lebih kasar dan warna cokelat kekuningan sehingga lebih
disukai.
B. Hasil uji hedonik pada abon dibuat dengan cara tradisional dan modern
berbeda. Abon dibuat dengan metode tradisional lebih disukai daripada abon
dibuat dengan metode modern.
C. Tekstur abon tradisional dan modern berbeda karena penggunaan mesin
penggiling yang membuat daging terpisah seluruhnya. Warna abon juga
dipengaruhi penggunaan gula dan lama penggorengan.
D. Abon yang dibuat dengan metode tradisional lebih disukai karena teksturnya
lebih kasar. Perbedaan tekstur abon dimungkinkan karena penggunaan mesin
penggiling daging.

14. Perhatikan kutipan teks pidato persuasif berikut!


Seseorang yang kecanduan game online sangat mudah dikenali. Mata mereka akan
kelihatan sayu dan kemerahan karena sering terpapar layar gawai. Mereka sering
menyendiri karena terbiasa asyik dengan gawainya. Di sekolah pun mereka tidak
bisa konsentrasi dan merasa cepat bosan dengan aktivitas di kelas. Karena
dampaknya yang buruk, mari kita kurangi game online agar tidak kecanduan.

Isi teks pidato di atas adalah….


A. Ajakan untuk mengurangi kegiatan game online
B. Trik jitu agar seseorang tidak kecanduan game online
C. Ciri-ciri orang yang belum kecanduan game online
D. Dampak positif seseorang yang menyukai game online

15. Lingkungan sekolah seharusnya bisa mencerminkan budaya kita yang selalu
mengedepankan kenyamanan. Pencerminan budaya di lingkungan sekolah akan
berdampak positif bagi masyarakat sekolah. Alangkah tidak bijaknya jika lingkungan
sekolah kita terlihat kotor dan tidak terasa nyaman lagi.

Bagian Struktur pidato yang ditonjolkan pada kutipan di atas adalah….


A. Pendapat atau pendirian
B. Pernyataan Posisi
C. Tahap Argumen
D. Penguatan pernyataan posisi

16. Cermati kutipan teks pidato berikut!


Tawuran antarpelajar ini sangatlah menganggu ketertiban dan keamanan
lingkungan sekitarnya. Saat ini, tawuran tidak hanya terjadi di sekolah atau
lingkungan sekitarnya. Banyak remaja yang melakukan aksi tawuran di jalanan
dengan menggunakan alat-alat bantu, seperti senjata tajam. Hal tersebut tentu
menimbulkan kerugian karena dapat mengakibatkan jatuhnya korban yang tidak
bersalah.

Tema kutipan pidato tersebut adalah ....


A. gangguan lingkungan
B. sosial budaya
C. perilaku sosial
D. keagamaan

17. Cermati kutipan pidato persuasif berikut!

Kebersihan lingkungan merupakan hal yang sangat penting guna menjaga


kesehatan diri sendiri dan lingkungan sekitar. Lingkungan yang sehat akan
meminimalisir penyebaran penyakit dan akan memberikan kenyamanan saat
berada di lingkungan tersebut.

Oleh karena itu lingkungan juga merupakan faktor pendorong keberhasilan


proses belajar mengajar di kelas. Siswa dan guru akan melaksanakan kegiatan
pembelajaran secara efektif karena didukung oleh keadaan lingkungan yang
nyaman. Maka dari itu, marilah kita sebagai warga sekolah dapat menjaga
kebersihan lingkungan sekolah dimulai dari diri sendiri, seperti membuang
sampah pada tempatnya dan tidak meninggalkan barang-barang di laci karena
dapat menjadi sarang nyamuk. Dengan demikian, menjaga kebersihan lingkungan
menjadi sangat penting guna menciptakan lingkungan yang sehat dan nyaman.

Kutipan teks tersebut merupakan struktur isi teks pidato persuasif bagian ….
A. penguatan peryataan posisi
B. tahapan argumen
C. pernyataan posisi
D. argumen posisi

18. Bacalah kutipan pidato berikut!


Bapak dan Ibu yang saya hormati.
Apotek hidup adalah tumbuh-tumbuhan yang berkhasiat obat yang ditanam di
halaman rumah. Tumbuhan yang dipelihara ini dapat dijadikan obat penyakit
tertentu. Misalnya papaya, daunnya berguna untuk menyembuhkan malaria.

Isi kutipan pidato tersebut adalah ….


A. pengertian dan manfaat apotek hidup
B. malaria dapat disembuhkan oleh daun papaya
C. contoh tumbuhan yang ditanam di apotek hidup
D. ajakan membuat apotek hidup

19. Dengan mengucapkan hamdalah, saya menyatakan bahwa acara sambutan siswa baru
kelas x tahun pelajara 2022-2023 dimulai. Semoga Tuhan senantiasa memberkati.

Penggalan teks tersebut di atas merupakan struktur teks bagian…

A. Pendahuluan

B. Isi

C. Penutup

D. Penjelasan

20. 1) Para tamu undangan yang saya hormati, izinkan saya mewakili semua teman-teman
menyampaikan kesan selama menempuh pendidikan di sini. 2) Selama saya menjalani 3
tahun di sini. 3) Bapak/ Ibu selalu membimbing saya . 4) Kami tidak hanya dibiasakan ilmu
pengetahuan tetapi juga budi pekerti . 5) Saya berharap bahwa semua dapat menjadi bekal
kami dalam menjalani hidup yang bermanfaat .

Kalimat argumen pada teks pidato di atas terdapat pada kalimat….

A. 1 dan 2

B. 3 dan 4
C. 1 dan 4

D. 2 dan 5

21. Penggunaan kata sapaan yang tepat terdapat pada kalimat…

A. Hadiri yang berbahagia, perkembangan zaman saat ini menuntut kita untuk aktif dan terus
berjuang keras.

B. Para hadirin yang berbahagia, perkembangan zaman saat ini menuntut kita untuk aktif dan
terus berjuang keras.

C. Hadirin sekalian yang berbahagis, perkembangan zaman saat ini menuntut kita untuk aktif
dan terus berjuang keras

D. Para hadirin sekalian, perkembangan zaman saat ini menuntut kita untuk aktif dan terus
berjuang keras

22. Pertama-tama saya ucapkan terima kasih kepada semua teman yang telah percaya pada
saya untuk mengemban amanah ini. Selanjutnya pada kesempatan yang berbahagia ini,
sebagai ketua kelas terpilih saya ingin mengajak semua teman-teman untuk menjaga
kerukunan daripada kita. Dengan demikian, segala kegiatan belajar mengajar dapat berjalan
lancar. Dan saya yakin kita dapat bekerjasama.
Simpulan dalam kutipan pidato tersebut adalah…

A. Jika terpilih saya berharap kegiatan belajar mengajar akan berjalan lancar.

B. Sebagai ketua kelas terpilih akan mengajak semua teman agar rukun. Supaya KBM menjadi
lancar.

C. Sebagai ketua kelas terpilih saya akan berusaha meningkatkan kebersihan dan kerukunan.

D. Dengan kerukunan maka akan tercipta lingkungan belajar yang nyaman.

23. Selanjutnya pada kesempatan yang berbahagia ini, sebagai ketua kelas terpilih saya
ingin mengajak semua teman-teman untuk menjaga kerukunan daripada kita. Dengan
demikian, segala kegiatan belajar mengajar dapat berjalan lancar.
Dalam kutipan tersebut yang termasuk kalimat persuasif adalah….

A. Dengan demikian, segala kegiatan belajar mengajar akan berjalan lancar.

B. Pertama-tama saya ucapkan terima kasih kepada semua teman yang telah percaya pada saya
untuk mengemban amanah ini.

C. Selanjutnya pada kesempatan yang berbahagia ini, sebagai ketua kelas terpilih saya ingin
mengajak semua teman-teman untuk menjaga kerukunan daripada kita

D. Dan saya yakin kita dapat bekerjasama.


24. Demikianlah sambutan yang dapat kami sampaikan. Maafkan atas segala
kesalahan, baik yang disengaja maupun yang tidak kami sengaja. Kami sampaikan
selamat jalan kepada Bapak Hamdi. Saya harap kita bisa mengikhlaskannya.
Kata emotif yang terdapat dalam kutipan tersebut adalah….

A. Harap

B. Baik

C. Sengaja

D. Bisa

25. Di bawah ini kalimat yang memuat istilah khusus yang berkenaan dengan pidato persuasif
bertema kemacetan adalah ...

A. Kita hendaknya bersikap dengan bijaksana untuk mengatasi masalah ini. Kita harus
menemukan solusi yang tepat untuk mengatasi masalah ini.

B. Salah satu tindakan yang dapat kita lakukan adalah menggunakan alat transportasi
umum pada saat akan bepergian, seperti bus, KRL, dan MRT

C. Jam-jam tersebut terjadi lebih kurang pukul tujuh sampai sembilan pagi dan pukul
empat sore sampai tujuh malam.

D. Di saat itulah aktivitas penduduk sedang berlangsung di jalan raya, seperti pada jam-
jam berangkat dan pulang kerja.

26. Bi Enoi, seorang perempuan yang menjadi tulang punggung keluarga. Setiap pagi, ia akan
menggenjot sepedanya, menembus kabut dan udara pagi yang mengecup sumsum. Sudah
hampir lima tahun ia menyadap karet sendirian. Perkaranya sudah basi sekali, kelima
anaknya perempuan semua. Lalu, Kuwar, suaminya yang berbadan tegap dengan kulit kuning
langsat dan wajah rupawan itu, meninggalkannya dan kelima anaknya. Tak ada yang bisa
dilakukan Bi Enoi selain banting tulang untuk menghidupi kelima anaknya.

Tema dalam kutipan cerpen tersebut adalah . . . .

A. penderitaan hidup yang dialami anak-anak kecil

B. perjuangan seorang ibu yang sebagai orang tua tunggal

C. tanggung jawab seorang lelaki terhadap anak istrinya


D. keresahan seorang perempuan dalam mencari nafkah

27. Pada zaman dahulu, hidup seekor burung merak yang angkuh. Suatu hari, ia bertemu
dengan seekor burung bangau. Merak ingin memamerkan bulunya yang indah kepada
bangau. Ia merentangkan bulunya di bawah sinar matahari untuk membuat bangau kagum.
Watak burung merak pada cerita di atas adalah...

A. rendah hati

B. baik hati

C. angkuh

D. usil

28. Bacalah kutipan cerpen berikut !


Minggu pagi, Dimas mengajak Aldi menuju sungai di dekat sekolahnya.
Di sana, mereka bergabung dengan teman-temannya yang lain. Aldi tertegun
melihat tumpukan sampah. “Hai, jangan melamun! Ayo, segera kita bantu Rio
dan Beni membersihkan sampah!” ujar Dimas. “Baiklah,” jawab Aldi. “Ayo,
semangat!” teriak Dimas kepada teman-temannya.
Tokoh utama cerita tersebut adalah ….

A. Aldi
B. Dimas
C. Beni
D. Rio

29. Bacalah kutipan cerpen berikut!


(1) Betapa gembiranya Ardi. (2) Saat pulang sekolah ia mendapat tawaran dari
pamannya Abdulmanam, untuk bersekolah di Jakarta. (3) Bagaimanpun di
Jakarta harus berjuang . (4). Jakarta adalah kota besar, yang selama ini ia
impikan, yakni bisa melihat dan tinggal di Jakarta.
Bukti bahwa latar waktu kutipan tersebut pada siang hari ditunjukan pada
nomor ....

A. (4)
B. (3)
C. (2)
D. (1)

30. Bacalah kutipan cerita berikut!


Tanah di pekuburan umum itu masih basah ketika para pelayat sudah pulang.
Sementara aku masih duduk sambil sesekali menyeka air mata. Ibu yang selama ini
paling aku hormati dan sayangi tadi malam telah menghadap Sang Pencipta.
Latar suasana yang tergambar pada kutipan cerita tersebut adalah ….

A. Kecewa
B. Sedih
C. Marah
D. khawatir

31. Bacalah teks cerita pendek berikut!


Kukuruyu adalah raja bangsa ayam. la bersahabat dengan Halili, raja bangsa
ikan tongkol. Mereka saling kunjung, meskipun hidup di alam yang berbeda.
Suatu hari Kukuruyu mengajak Halili ikut pesta dansa di kampung nelayan.
Mereka sepakat datang, saat senja tiba. Halili berpesan kepada Kukuruyu agar
memberi tahu jika waktu fajar telah tiba. Jika tidak, bangsa manusia akan
menyantap mereka. Kukuruyu menyanggupinya.
Makanan enak tersaji di pesta. Kukuruyu dan Halili makan terlalu banyak.
Mereka pun tertidur lelap hingga melewati waktu fajar karena kekenyangan. Semua
ikan tongkol ditangkap oleh nelayan. Halili kesal karena Kukuruyu ingkar janji. Halili
bersumpah akan memakan bangsa ayam yang datang ke laut. Persahabatan
mereka pun berubah menjadi permusuhan.
Amanat cerita tersebut adalah….

A. Jalinlah persahabatan dengan saling menghormati!


B. Tepatilah janji yang pernah diucapkan!
C. Jangan menginginkan barang yang bukan haknya!
D. Kasihilah makhluk lain dengan sepenuh hati!

32. Bacalah kutipan cerpen berikut!


Baik Hati pun pergi ke sebidang tanah tandus itu sambal membawa bibit semangka
dan kuda pincang yang menjadi bagiannya. Meski mendapat warisan yang kurang
menguntungkan, Baik Hati tidak putus asa. Ia langsung menanami lahan tandus itu
dengan bibit semangka.
Kuda pincang yang ia tambatkan di pohon membuang kotoran sembarangan.
Namun, Baik Hati tidak marah. Ia malah mengumpulkan kotoran itu dan
menjadikannya pupuk.
Sudut pandang pada kutipan cerita pendek tersebut adalah ... .

A. Sudut pandang orang ketiga pengamat.


B. Sudut pandang orang pertama pelaku sampingan.
C. Sudut pandang orang ketiga serba tahu.
D. Sudut pandang orang pertama pelaku utama.

33. Bacalah teks cerita pendek berikut !


Seperti apakah rasanya hidup menjadi orang yang tak dimaui? Tanyakan
pertanyaan ini padanya. Jika dia bisa berkata-kata, maka yakinlah dia akan
melancarkan jawabnya. Konon dia lahir tanpa diminta. Korban gagal gugur
kandungan dari seorang perempuan. Hasil sebuah hubungan gelap yang dilaknat
warga dan Tuhan.
Perempuan yang saat ini disebut “ibunya” bukanlah ibu yang sebenarnya. Dia
hanya inang yang berkasihan lalu bergantian menyusui lapar mulut dua orang bayi;
bayi berwajah penyok yang dibuang orang di pinggir kampung.
Paragraf di atas termasuk ke dalam struktur cerpen ....
A. Orientasi
B. Komplikasi
C. Koda
D. resolusi

34. Bacalah kutipan cerita berikut!


“Hai Gemblong!” sapa teman-teman Daryati dengan nada penuh candaan.
Memang, ia sering mendapatkan sapaan seperti itu di kelasnya, bahkan juga dari
teman kelas lain. Tubuh Daryati tidak gemuk. Sapaan itu diberikan karena ibunya
memang seorang bakul gemblong, makanan dari ketela pohon yang dikukus lalu
ditumbuk halus. Namun untaian candaan itu tak pernah dipedulikannya meski
secara jujur Daryati merasa tak nyaman. Ia merasa mendapat tempat untuk berlatih
sabar.
Penyebab konflik yang dialami tokoh pada kutipan cerita tersebut adalah ….

A. sering dipanggil dengan nada ejekan


B. tubuh Daryati gemuk alias gendut
C. ibunya seorang bakul gemblong
D. ia harus sabar setiap hari

35. Bacalah kutipan cerpen berikut !

Setiba di pantai, aku segera melangkahkan kakiku di hamparan pasir putih.


Ombak menyapaku dengan riang gembira. Empasan angin di wajahku membuat
suasana hatiku menjadi tenang. Kicauan camar juga menambah suasana menjadi
syahdu.

Majas personifikasi dalam paragraf di atas terdapat di kalimat ….

A. Pertama

B. Kedua

C. Ketiga

D. keempat

36. Bacalah kutipan cerpen berikut !

Hubungan dua bersaudara itu sangat dekat sehingga banyak hal yang saling
diketahui. Namun, kini mereka berpisah karena urusan pekerjaan. Saudara tua tinggal
di Jakarta dan saudara muda tinggal di London. Semenjak kepergian saudara muda,
saudara tua merasa kesepian. Berbagai cara telah dicoba untuk mengusir rasa sepi,
tetapi selalu sia-sia.
Majas yang tepat untuk melengkapi paragraf tersebut adalah ...

A. Ia kini beratap langit beralas bumi.

B. Ia di Jakarta tetapi hatinya di London.

C. Ia menanti saudaranya selamanya.

D. Ia merasa sepi di kota yang ramai

37. (1) Jangan-jangan bentol merah itu karena Aira terkena bulu ulat atau gigitan
serangga.

(2) Zahra jadi merasa bersalah dan duduk merenung.

(3) Sekarang dia akan lebih berhati-hati lagi mengingat kondisi Aira yang rentan.

(4) Tadi siang sebelum demam, Aira ikut bermain di kebun bersamanya.

Urutan kalimat yang tepat agar menjadi alur cerpen yang benar adalah….

A. 2-4-1-3

B. 2-3-1-2

C. 2-1-3-4

D. 4-3-1-2

38. Bacalah kutipan cerpen berikut !

Di tengah kota yang ramai, hati Nurul seperti taman bunga yang sepi.
Kenangan indah menjadi bunga-bunga yang layu, dan harapan seperti matahari yang
terus bersinar di balik awan kelabu. Nurul merasa terasing di tengah kota yang ramai.

Kalimat yang mengandung majas metafora terdapat pada ….

A. Di tengah kota yang ramai, hati Nurul seperti taman bunga yang sepi.

B. Kenangan indah menjadi bunga-bunga yang layu.

C. Harapan seperti matahari yang terus bersinar di balik awan kelabu.

D. Terasing di tengah kota yang ramai.

39. "Penting bagi pemerintah untuk memprioritaskan pendidikan guna menciptakan


masyarakat yang terdidik dan berpengetahuan. Peningkatan kualitas pendidikan dapat
membuka pintu peluang bagi generasi masa depan. Namun, perlu diingat pula bahwa
pengembangan infrastruktur juga memegang peran penting dalam meningkatkan
kesejahteraan masyarakat."

Informasi penting berdasarkan teks tanggapan di atas adalah ...

A. Peningkatan kualitas pendidikan

B. Pengembangan infrastruktur

C. Kesejahteraan masyarakat

D. Semua jawaban benar

40. Bacalah kutipan teks tanggapan berikut untuk menjawab soal nomor 40-42!

Sebuah karya seni kelas tinggi dari sang pelukis maestro Affandi, melukiskan
sebuah pemandangan alam perkebunan cengkeh, area perkebunan berbukit yang
masih alami tampak terlukis apa adanya dari alam, untuk menghidupkan suasana pada
lukisan, dihadirkannya figur manusia sebagai objek pendukung, tetapi inti dari
lukisan, yang menunjukkan adanya aktivitas kehidupan yang menyatu dengan alam.
Ekspresi goresan khas Affandi terlihat unik, yang menjadikan lukisan ini istimewa.

Seperti pada kebanyakan lukisan Affandi yang selalu menempatkan matahari


sebagai bagian dari objek utama, tetapi dalam lukisan ini, penempatan matahari
tampak unik, seolah sang pelukis mengambil perspektif posisi di balik matahari.
Tampak dalam lukisan matahari tidak di balik bukit, tetapi di atas bukit dan menutupi
bukit.
Keunikan ini mungkin hanya dimiliki oleh Affandi, sebagai cara sudut pandang
dia dalam berekspresi, di mana kualitas imajinasinya sebagai seorang pelukis maestro
ternama.

Objek yang ditanggapi dalam kutipan teks tanggapan tersebut adalah ….

A. pemandangan alam perkebunan cengkeh


B. lukisan yang dibuat oleh pelukis Affandi
C. kehidupan dari pelukis ternama Affandi
D. keunikan dari pelukis maestro Affandi

41. Bagian yang merupakan struktur konteks pada kutipan teks tersebut adalah …

A. paragraf pertama
B. paragraf kedua
C. paragraf ketiga
D. paragraf 1 dan 2
42. Bagian yang merupakan struktur deskripsi pada kutipan teks tersebut adalah …
A. paragraf pertama
B. paragraf kedua
C. paragraf ketiga
D. paragraf 1 dan 2

43. Contoh Soal Teks Tanggapan

Teks 1
Buku pengkajian Puisi karya Rahmad Joko Pradopo ini pembahasannya sangat
bagus karena dilengkapi dengan contoh-contoh analisis semiotik maupun
hubungan intertekstualnya. Analisis yang cukup rinci akan menambah
pengetahuan pembaca untuk memahami puisi. Buku ini pantas dimiliki pembaca
yang ingin belajar memahami puisi.

Teks 2
Buku Kesenian Indonesia pada Era Global mengupas konsep budaya dan
bagian-bagiannya serta jenis dan bentuk budaya Indonesia. Buku Kesenian
Indonesia pada Era Global juga menjelaskan keunikan dan potensi budaya
Indonesia di dunia internasional hingga realisasi diplomasi budaya Indonesia di
dunia internasional. Namun, bentuk budaya Indonesia yang ditampilkan pada
buku Kesenian Indonesia pada Era Global kurang mewakili budaya daerah
setiap wilayah di Indonesia.

Pernyataan yang benar tentang bentuk teks tanggapan di atas adalah ...

a. Teks 1 berbentuk kritikan dan teks 2 berbentuk pujian

b. Teks 1 berbentuk pujian dan teks 2 berbentuk kritikan

c. Teks 1 dan teks 2 berbentuk pujian

d. Teks 1 dan teks 2 berbentuk kritikian

44. Kalimat pujian yang baik terhadap sebuah patung adalah ...

a. Patung tersebut memiliki detail yang menarik, sayang ukiran patung


tersebut tidak rata

b. Patung tersebut memiliki bentuk yang unik, tetapi bentuk tangan


patung tidak proporsional
c. Patung yang sangat indah akan menarik untuk dinikmati oleh
masyarakat.

d. Perpaduan unsur modern dan tradisional membuat patung ini begitu


istimewa

Sebuah karya seni kelas tinggi dari sang pelukis maestro Affandi, melukiskan
sebuah pemandangan alam perkebunan cengkeh, area perkebunan berbukit yang
masih alami tampak terlukis apa adanya dari alam, untuk menghidupkan suasana
pada lukisan, dihadirkannya figur manusia sebagai objek pendukung, tetapi inti
dari lukisan, yang menunjukkan adanya aktivitas kehidupan yang menyatu dengan
alam.

45. Kalimat di atas bertujuan untuk …


a. Menunjukkan kelebihan karya
b. Menunjukkan kekurangan karya
c. Memuji karya seni lukisan Affandi
d. Mendeskripsikan karya

46. Manakah dari kalimat-kalimat berikut yang merupakan kalimat kompleks


a. Ayah pergi ke kantor setiap pagi.
b. Setelah makan malam, kami duduk bersama di ruang keluarga sambil bercerita.
c. Bunga-bunga itu sangat indah.
d. Dia tertawa dengan gembira.

47. Di Indonesia masih banyak ditemukan layanan yang terkesan tidak menyeluruh,
peralatan kurang canggih, dan dokter-dokter yang terlihat tidak percaya diri. Padahal
kalau ditilik secara keseluruhan, RS di Indonesia banyak yang memiliki layanan yang
lebih bagus serta peralatan lebih lengkap dan tentu saja lebih murah karena lokasinya
dekat. Tenaga kesehatan yang profesional pada tiap-tiap bidang menimbulkan
ketidakpuasan pada pasien. Seharusnya peralatan canggih di rumah sakit diimbangi
juga dengan pelayanan yang memuaskan untuk pasien.
Kalimat yang salah, pada kutipan teks tanggapan di atas terdapat pada kalimat ….

A. pertama
B. kedua
C. ketiga
D. keempat

48. Setelah sukses membesut Rindu (terbitan Republik, 2014) yang mencetak
bestseller, Tere Liye hadir kembali dengan novel barunya, Pulang. …. yang telah
menghasilkan lebih dari 20 buku menghadirkan novel dengan tema dan genre yang
berbeda dibanding novel-novel sebelumnya.

Kata rujukan yang tepat untuk melengkapi kalimat kedua pada teks di atas adalah ….
A. beliau
B. dia
C. Orang itu
D. penulis

49. Kalimat yang menunjukkan kelebihan pada teks tanggapan kritis di bawah ini
adalah ….
A. Di Indonesia masih banyak ditemukan layanan yang terkesan tidak
menyeluruh, peralatan kurang canggih, dan dokter-dokter yang terlihat tidak percaya
diri.
B. Padahal kalau ditilik secara keseluruhan, RS di Indonesia banyak yang
memiliki layanan yang lebih bagus serta peralatan lebih lengkap dan tentu saja lebih
murah karena lokasinya dekat.
C. Tenaga kesehatan yang profesional pada tiap-tiap bidang menimbulkan
ketidakpuasan pada pasien.
D. Seharusnya peralatan canggih di rumah sakit diimbangi juga dengan pelayanan
yang memuaskan untuk pasien.

50. Enrico malas menulis, hingga kelas IX tulisannya masih jelek, tidak jauh berbeda
dengan saat di SD. Buku tulisnya banyak yang kosong dan bersih dari tulisan.
Tanggapan yang tepat untuk kutipan teks di atas adalah ….
A. Sebaiknya Enrico mengandalkan orang lain untuk menulis dibukunya.
B. Sebaiknya Enrico kadang-kadang menulis agar tulisannya rapi.
C. Sebaiknya Enrico itu tidak malas untuk menulis, agar tulisannya bisa lebih rapi.
D. Sebaiknya Enrico tetap malas menulis agar tulisannya rapi.
SOAL PENILAIAN AKHIR SEMESTER
MGMP BAHASA INDONESIA
KELAS 9
PAKET B

A. PILIHAN GANDA

1. Bacalah kutipan teks laporan percobaan berikut dengan cermat!


Komodo adalah hewan reptil besar berkaki empat dan wujudnya menyerupai kadal-kadalan.
Ukuran hewan ini sangatlah besar. Jika dilihat sekilas, komodo tampak seperti biawak.
Namun, ketika perhatikan dengan saksama, ukurannya jauh lebih besar daripada biawak.

Informasi yang sesuai dengan paragraf pertama teks laporan hasil observasi di atas adalah …

A. Kadal berukuran sedang dan tidak terlalu besar


B. B. Komodo dan biawak adalah hewan yang sama
C. C. Ukuran komodo lebih besar daripada biawak
D. D. komodo termasuk hewan mamalia berkaki dua.

2. Bacalah kutipan teks laporan berikut, dengan saksama!


Kecambah adalah sebuah hasil pertumbuhan awal dari tanaman famili
kacang-kacangan. Pertumbuhan lekuk pada kacang sendiri dipengaruhi oleh
beberapa aspek penting salah satunya sinar matahari.Dalam pengamatan kali ini
kami akan membuat sebuah laporan bagaimana pengaruh matahari dan sumber air
mengatur lekuk kecambah.Apakah sesuai dengan teori fundamental pertumbuhan
atau tidak. Durasi penelitian yang dilakukan selama dua minggu terhitung mulai
biji kacang hijau mengeluarkan tunas.

Kesimpulan pada teks laporan di atas adalah….

A. Kecambah adalah sebuah hasil pertumbuhan awal dari tanaman famili kacang-
kacangan yang dipengaruhi oleh sinar matahari dan air dibutuhkan waktu dua minggu.

B. Pertumbuhan lekuk pada kacang sendiri dipengaruhi oleh beberapa aspek penting salah
satunya sinar matahari.

C. Dalam pengamatan kali ini kami akan membuat sebuah laporan bagaimana pengaruh
matahari dan sumber air mengatur lekuk kecambah.
D. Apakah sesuai dengan teori fundamental pertumbuhan atau tidak.

3. Bacalah kutipan teks laporan percobaan berikut dengan cermat!


Membuat Teropong Bintang
Kita dapat melihat benda-benda di langit secara langsung menggunakan
teropong bintang. Ada beberapa jenis teropong bintang. Masing-masing memiliki
spesifikasi yang berbeda-beda. Karena harganya yang cukup mahal, kita pun dapat
membuat teropong sesuai dengan spesifikasi yang kita inginkan.

Yang merupakan Frase benda pada teks laporan di atasa adalah …

A. Teropong bintang

B. Spesifikasi

C. Secara langsung

D Cukup mahal

4. Bacalah kutipan teks berikut!

Bencana alam adalah suatu peristiwa alam yang mengakibatkan dampak besar bagi populasi
manusia. Bencana alam dapat terjadi di manapun dan kapan pun, tak terkecuali di Indonesia.
Indonesia merupakan negara kepulauan yang rawan bencana alam. Itulah sebabnya Indonesia
banyak mengalami bencana alam terutama gempa bumi, gunung meletus, banjir, dan tanah
longsor.

Dari teks tersebut yang termasuk dalam definisi umum laporan adalah....
A. Bencana alam adalah suatu peristiwa alam yang mengakibatkan dampak besar bagi
populasi manusia.
B. Bencana alam dapat terjadi di manapun dan kapan pun, tak terkecuali di Indonesia.
C. Indonesia merupakan negara kepulauan yang rawan bencana alam
D. Itulah sebabnya Indonesia banyak mengalami bencana

5. Terumbu karang memiliki bentuk unik ... warna beraneka rupa serta menghasilkan CaCO3.

Konjungsi yang tepat untuk melengkapi kalimat rumpang tersebut adalah...


A. yang
B. saat
C. dan
D. tetapi

6. Dalam teks laporan hasil percobaan menggunakan kata kerja aktif, misalnya kata kerja
aktif terdapat pada kalimat …
A. Andry melakukan percobaan terhadap air mineral.
B. Air yang jernih itu harus dituang ke dalam botol terlebih dahulu.
C. Sebelum dilakukan percobaan, air tersebut harus disimpan di lemari pendingin.
D. Air yang akan dicoba didinginkan dahulu untuk beberapa menit.

7. Perhatikan kutipan berikut!


Hasil percobaan menunjukkan bahwa efek pelangi terbentuk pada kertas tisu yang
ditempatkan di atas permukaan minyak. Hal ini terjadi karena pembiasan cahaya yang
melalui lapisan-lapisan tipis minyak dan pewarna makanan. Pewarna makanan di permukaan
minyak membentuk lapisan tipis dengan ketebalan yang bervariasi, sehingga menyebabkan
interferensi cahaya dan menciptakan efek warna pelangi.

Tujuan dari teks di atas adalah…


A. Menentukan pelangi
B. Menentukan warna tisu
C. Menjelaskan proses pembentukan efek pelangi pada kertas tisu
D. Proses pewarna makanan bila diberi pewarna dan minyak.

8. Seseorang yang kecanduan game online sangat mudah dikenali. Mata mereka akan
kelihatan sayu dan kemerahan karena sering terpapar layar gawai.

Mereka sering menyendiri karena terbiasa asyik dengan gawainya. Di sekolah pun mereka
tidak bisa konsentrasi dan merasa cepat bosan dengan aktivitas di kelas. Karena dampaknya
yang buruk, mari kita kurangi game online agar tidak kecanduan.
Isi informasi teks pidato persuasif di atas adalah ...
A. Ajakan untuk mengurangi kegiatan game online.
B. Trik jitu agar seseorang tidak kecanduan game online.
C. Ciri-ciri orang yang belum kecanduan game online.
D. Dampak positif seseorang yang menyukai game online.

9. Perhatikan penggalan teks pidato persuasif berikut!


Budaya menyontek berdampak buruk bagi siswa. Daya kreativitas mereka dalam belajar tidak
akan berkembang karena selalu mengandalkan hasil menyontek.

Degradasi kepercayaan diri juga pasti akan dialami siswa. Menyontek juga membiasakan
siswa untuk berbohong. Karena dampak negatifnya sangat besar, mari hindari kebiasaan
menyontek.

Pernyataan yang sesuai dengan isi teks pidato persuasif di atas adalah …
A. Kebiasaan menyontek membuat siswa lebih kreatif.
B. Kepercayaan diri siswa akan turun jika membiasakan menyontek.
C. Menyontek bukanlah termasuk perbuatan yang memiliki dampak negatif.
D. Menyontek jauh berbeda dengan tindakan berbohong.

10. Bacalah kutipan teks berikut!


Mari kita pertimbangkan dampak lingkungan yang ditimbulkan oleh plastik sekali pakai.
Plastik yang sulit terurai alami, mencemari lautan, hutan, dan tanah. Hewan-hewan laut yang
kita kagumi, seperti penyu dan ikan, seringkali terperangkap oleh sampah plastik dan
menderita akibatnya. Selain itu, plastik yang terurai menjadi mikroplastik dapat mencemari
air yang kita minum dan makanan yang kita konsumsi, berpotensi membawa risiko kesehatan
yang serius bagi manusia.

Kalimat yang merupakan argumentasi dari kutipan tersebut adalah ...


A. Hewan-hewan laut yang kita kagumi, seperti penyu dan ikan, seringkali terperangkap oleh
sampah plastik
B. Plastik yang sulit terurai alami, mencemari lautan, hutan, dan tanah.
C. Selain itu, plastik yang terurai menjadi mikroplastik dapat mencemari air yang kita minum
dan makanan yang kita konsumsi, berpotensi membawa risiko kesehatan yang serius bagi
manusia.
D. Mari kita pertimbangkan dampak lingkungan yang ditimbulkan oleh plastik sekali pakai.

11. Bacalah kutipan teks berikut!


Selain itu, kita bisa mendukung gerakan daur ulang plastik. Dengan mendaur ulang, kita
dapat meminimalkan jumlah plastik yang berakhir di tempat pembuangan akhir. Bahkan,
dengan melakukan tindakan kecil ini, kita bisa memberikan kontribusi besar dalam
mengurangi dampak plastik sekali pakai terhadap bumi kita yang rapuh.

Bagian yang bukan merupakan isi pidato dari kutipan tersebut adalah …
A. Mengurangi penggunaan plastik dengan cara didaur ulang
B. Melakukan tindakan kecil untuk menyelamatkan alam
C. Mendukung gerakan daur ulang sampah sebagai wujud nyata menjaga alam
D. Dampak dari penggunaan limbah rumah tangga bagi lingkungan

12. Bacalah kutipan teks berikut!


Saat ini Bangsa kita tengah menghadapi persoalan yang sangat serius, yaitu menurunnya
moral di kalangan remaja. Banyak remaja yang sudah terpengaruh oleh sikap hidup hedonis
dan kehidupan yang mengabaikan nilai-nilai dan norma yang berlaku di masyarakat.

Pesan yang terkandung dalam kutipan tersebut adalah ...


A. Lunturnya moral remaja Indonesia
B. Gaya hidup hedonis menghancurkan moral bangsa
C. Norma-norma yang mulai luntur dari kehidupan bangsa
D. Tantangan bangsa Indonesia di kalangan remaja

13. Perhatikan penggalan teks pidato berikut!


Budaya menyontek berdampak buruk bagi siswa. Daya kreativitas mereka dalam belajar tidak
akan berkembang karena selalu mengandalkan hasil menyontek.

Degradasi kepercayaan diri juga pasti akan dialami siswa. Menyontek juga membiasakan
siswa untuk berbohong. Karena dampak negatifnya sangat besar, mari hindari kebiasaan
menyontek.

Simpulan yang tepat dari teks pidato di atas adalah …


A. Kebiasaan menyontek membuat siswa lebih kreatif.
B. Kepercayaan diri siswa akan turun jika membiasakan menyontek.
C. Menyontek bukanlah termasuk perbuatan yang memiliki dampak negatif.
D. Menyontek jauh berbeda dengan tindakan berbohong.
14. Perhatikan penggalan teks pidato berikut!
Selamat Siang, salam sejahtera dan semoga kita semua tetap dalam lindungan Tuhan yang
Maha Esa. yang saya hormati Bapak Kepala SMP Negeri 8 Kota Depok, seluruh staf guru,
dan siswa-siswi yang saya sayangi.

Kata sapaan yang tepat dari teks pidato di atas adalah, kecuali …
A. Selamat siang, salam Sejahtera
B. Dalam lindungan Tuhan yang Maha Esa
C. Yang saya hormati Bapak Kepala SMPN 8 Depok
D. Siswa-siswi yang saya sayangi

15. Perhatikan penggalan teks pidato berikut!

Sudah cukup rasanya tiga juta orang di Indonesia yang telah terjebak dalam
kenikmatan fananya. Ya, Anda tidak salah mendengarnya tiga juta orang dan masih terus
bertambah telah terjerumus ke lubang hitam benda haram ini berdasarkan data yang
dihimpun oleh BNN atau Badan Narkotika Nasional pada 2023.

Oleh karena itu saya menyatakan dengan tegas, tolong jangan dekati benda haram ini!
Bukan hanya masalah hukum dan bahkan hukum hanya perantara saja. Narkoba harus
dijauhi karena benda ini benar-benar destruktif dan akan menghancurkan kehidupan
Anda! Terima kasih.

Kalimat persuasif yang tepat dari teks pidato di atas adalah …

A. Oleh karena itu saya menyatakan dengan tegas, tolong jangan dekati benda haram ini!
B. Sudah cukup rasanya tiga juta orang di Indonesia yang telah terjebak dalam
kenikmatan fananya
C. Berdasarkan data yang dihimpun oleh BNN atau Badan Narkotika Nasional pada
2023.
D. Narkoba harus dijauhi karena benda ini benar-benar destruktif dan akan
menghancurkan kehidupan

16. . Cermatilah kutipan teks pidato persuasif berikut!

……

Kemudian, marilah kita menjaga semangat persatuan antarbangsa. Sebagai pemuda kita
jangan sampai mudah terpengaruh oleh pihak-pihak lain yang ingin memecah belah kita.
Apabila kita sudah terpecah, maka pihak lain akan masuk dengan mudah dan mengambil alih
bangsa ini. Oleh karena itu, marilah kita bangun kembali semangat persatuan yang sempat
hilang dalam diri kita. Jangan lagi ada perselisihan antar suku dan agama karena
sesungguhnya kita adalah satu yaitu bangsa Indonesia.

…..

Kalimat emotif pada kutipan teks pidato tersebut adalah..

A. Marilah kita menjaga semangat persatuan antarbangsa


B. Sebagai Pemuda kita jangan sampai mudah terpengaruh oleh pihak -pihak lain yang
ingin memecah belah.
C. Apabila kita sudah terpecah belah maka pihak lain akan masuk dengan mudah.
D. Jangan lagi ada perselisihan antar suku dan bangsa

17. Bacalah kutipan teks cerpen berikut!


Bobi suka sekali membaca majalah. Dia tidak mau diganggu. Setiap hari ia mencoba
untuk menulis pada secarik kertas. Di ruang belajarnya terdapat tumpukan kertas yang berisi
tulisan. Ternyata yang ditulis sebuah cerpen.

Bukti watak Bobi berkemauan keras pada cuplikan cerita tersebut adalah..

A. Bobi suka membaca majalah


B. Dia tidak mau diganggu
C. Setiap hari ia mencoba untuk menulis pada secarik keras
D. Ternyata yang ditulis sebuah cerpen

18. Bacalah kutipan teks berikut!


Sebelum duduk di bawah tugu,sebagai orang kota sejati, aku beberkan sapu tanganku ke
rumput supaya pantalon tropical yang kupakai tidak kotor. Kemudian, aku memandang pada
makam sambal menyalakan sigaret lagi. Di sekitar tampat rindang itu matahari memanas
terik.

Latar waktu dalam kutipan cerpen tersebut adalah…


A. pagi
B. siang
C. sore
D. senja

19. Bacalah kutipan cerpen berikut dengan cermat!



Saya tinggal di desa Maju Sari, Pak. Kebetulan, untuk sekarang saya juga masih menganggur.
Masih menunggu beberapa panggilan kerja, tetapi sudah beberapa bulan belum ada kabar,
pak.” Jawabku dengan jujur.
….

Suasana yang tergambar dalam kutipan cerpen tersebut adalah ....


A. haru
B. sedih
C. penasaran
D. tenang

20.Bacalah kutipan teks berikut


Pagi itu, hujan turun dengan lebat. Ani merasa bingung bagaimana cara pergi ke sekolah
di tengah cuaca buruk tersebut. Ketika Ani sedang memandang hujan dari jendela, tiba-
tiba telepon genggamnya berdering di kamar. Ani segera masuk ke kamar dan menjawab
panggilan tersebut.

Kutipan cerpen tersebut menggunakan sudut pandang ….


A. orang ketiga
B. orang pertama
C. pengamat
D. orang kedua

21.Bacalah kutipan cerpen berikut dengan saksama!

Anak Rajin dan Pohon Pengetahuan

Glory Gracia Christabelle


(1) Pada suatu waktu, hiduplah seorang anak yang rajin belajar. Mogu
namanya. Usianya 7 tahun. Sehari-hari ia berladang. Juga mencari kayu bakar di
hutan. Hidupnya sebatang kara. Mogu amat rajin membaca. Semua buku habis
dilahapnya. Ia rindu akan pengetahuan.
(2) Suatu hari ia tersesat di hutan. Hari sudah gelap. Akhirnya Mogu
memutuskan untuk bermalam di hutan. Ia bersandar di pohon dan jatuh tertidur.
(3) Dalam tidurnya, samar-samar Mogu mendengar suara
memanggilnya. Mula-mula ia berpikir itu hanya mimpi. Namun, di saat ia
terbangun, suara itu masih memanggilnya. "Anak muda, bangunlah! Siapakah
engkau? Mengapa kau ada di sini?" Mogu amat bingung. Dari mana suara itu
berasal? Ia mencoba melihat ke sekeliling. "Aku di sini. Aku pohon yang kau
sandari!" ujar suara itu lagi.
(4) Seketika Mogu menengok. Alangkah terkejutnya ia! Pohon yang
disandarinya ternyata memiliki wajah di batangnya.
(5) "Jangan takut! Aku bukan makhluk jahat. Aku Tule, pohon
pengetahuan. Nah, perkenalkan dirimu," ujar pohon itu lagi lembut.

Struktur bagian orientasi pada kutipan cerpen tersebut ditandai dengan nomor ….
A. (1)
B. (2)
C. (3)
D. (4)

22. Cermati kutipan cerpen acak berikut!

1. Tante Hilda gelisah dan cemas.


2. Padahal tadi tante Hilda masih sempat melihat anaknya itu bermain sendirian di
teras depan.
3. Upik, putri tunggalnya hilang.
4. Upik yang baru berusia empat tahun itu, tidak tahu pergi ke mana.

Agar menjadi teks cerpen yang padu, susunan kalimat yang tepat adalah ....
A. (1) - (2) - (3) - (4)
B. (1) - (4) - (3) - (2)
C. (4) - (3) - (2) - (1)
D. (1) - (3) - (4) - (2)

23. Cermati kutipan teks berikut ini!

Sinarnya mulai merangkak naik ketika Sang nenek membangunkan Bawang Merah dari
tidurnya.

Kalimat yang memiliki majas sama dengan kalimat di atas adalah ….

A. Sesampainya di rumah Bawang Merah mengetuk-ngetuk pintu sambil berteriak-


teriak.
B. Dengan perut yang mulai lapar, pemuda itu pulang ke rumahnya.
C. Tiba-tiba dari langit terdengarlah suara petir menggelegar setelah Sang suami
melanggar sumpahnya.
D. Waktu berlari dengan cepatnya, Malin Kundang telah menjadi seorang pemuda
tampan dan gagah.

24. Bacalah kutipan teks tanggapan berikut dengan saksama!


Era globalisasi membuat banyak,temasuk mudahnya berbagai transportasi canggih masuk ke
dalam negeri. Hal ini membuat berbagai tren baru muncul di kalangan remaja. Salah satunya,
pelajar membawa motor ke sekolah, padahal mereka tahu itu pelanggaran karena masih di bawah
umur.
Fenomena pelajar mengendarai motor ke sekolah tidak hanya terjadi di daerah perkotaan,
namun di daerah terpencil juga, mereka cenderung ingin selalu tampil keren dengan perkembangan
zaman dan kebanyakan mereka tidak ingin dikatakan ketinggalan zaman. Alasan yang semakin
memperkuat adalah kesibukan orangtua yang sangat padat sehingga tidak memperhatikan anaknya
dan tidak memiliki waktu mengantar jemput anaknya ke sekolah. Dalam tren ini memunculkan
berbagai pendapat tentang kelebihan dan kekurangan.

Kutipan teks tersebut berisi informasi tentang ....

A. Era globalisasi dapat memudahkan berbagai transportasi.


B. Banyak pelajar yang membawa motor ke sekolah.
C. Remaja yang ingin selalu tampil keren.
D. Orang tua yang sibuk tak sempat mengantar anaknya.

25. Bacalah kutipan teks tanggapan berikut dengan saksama!


(1) Era globalisasi membuat berbagai transportasi canggih masuk ke dalam negeri. Hal ini
membuat berbagai tren baru muncul di kalangan remaja. Salah satunya, pelajar membawa motor ke
sekolah, padahal mereka tahu itu pelanggaran karena masih di bawah umur.
(2) Fenomena pelajar mengendarai motor ke sekolah tidak hanya terjadi di daerah perkotaan,
juga di daerah terpencil, mereka cenderung ingin tampil keren dan tidak ingin dikatakan
ketinggalan zaman. Alasan yang memperkuat adalah orangtua yang sangat sibuk sehingga tidak
memperhatikan dan tidak sempat mengantar jemput anaknya ke sekolah. Tren ini memunculkan
berbagai pendapat.
(3) Kelebihan berkendara motor sendiri ke sekoah adalah lebih praktis, henat waktu, dan
efisien, siswa tidak perlu menunggu jemputan atau kendaraan umum, uga tidak perlu mengeluarkan
biaya ongkos
(4) Sedangkan kekurangannya anak di bawah umur tidak memiliki SIM sehingga dapat
diatakan melanggar peraturan. Mereka yang masih sekolah sering membawa motor ugal-ugalan
terkadang membahayakan pengguna jalan lainya. Saat pulang sekolah terkadang mereka nongkrong
dulu dan tidak lansgsing ke rumah yang dapat menyebabkan kekhawatiran orang tua dan pihak
sekolah.

Kalimat yang menyatakan konteks pada kutipan terdapat pada nomor ....
A. (1)
B. (2)
C. (3)
D. (4)

26. Bacalah kutipan teks tanggapan berikut dengan saksama!


(1) Era globalisasi membuat berbagai transportasi canggih masuk ke dalam negeri. Hal ini
membuat berbagai tren baru muncul di kalangan remaja. Salah satunya, pelajar membawa motor ke
sekolah, padahal mereka tahu itu pelanggaran karena masih di bawah umur.
(2) Fenomena pelajar mengendarai motor ke sekolah tidak hanya terjadi di daerah perkotaan,
juga di daerah terpencil, mereka cenderung ingin tampil keren dan tidak ingin dikatakan
ketinggalan zaman. Alasan yang memperkuat adalah orangtua yang sangat sibuk sehingga tidak
memperhatikan dan tidak sempat mengantar jemput anaknya ke sekolah. Tren ini memunculkan
berbagai pendapat.
(3) Kelebihan berkendara motor sendiri ke sekoah adalah lebih praktis, henat waktu, dan
efisien, siswa tidak perlu menunggu jemputan atau kendaraan umum, uga tidak perlu mengeluarkan
biaya ongkos
(4) Sedangkan kekurangannya anak di bawah umur tidak memiliki SIM sehingga dapat
diatakan melanggar peraturan. Mereka yang masih sekolah sering membawa motor ugal-ugalan
terkadang membahayakan pengguna jalan lainya. Saat pulang sekolah terkadang mereka nongkrong
dulu dan tidak lansgsing ke rumah yang dapat menyebabkan kekhawatiran orang tua dan pihak
sekolah.

Struktur bagian deskripsi pada teks tanggapan tersebut ditandai dengan nomor ....
A. (1), (2), (3)
B. (2), (3), (4)
C. (3), (4), (1)
D. (1), (4), (2)

27. Bacalah kutipan teks tanggapan berikut dengan saksama!


Sekolah era new normal harus menetapkan peraturan yang ketat. Siswa dilarang jajan
sembarangan dan harus bermain di dalam lingkungan sekolah. Sekolah juga wajib membuat
peraturan di mana seluruh siswa wajib memakai masker.

Meski sekolah new normal baru berjalan beberapa bulan, nyatanya siswa masih sulit diatur
karena sepulang sekolah, mereka justru duduk-duduk di depan sekolah sambil jajan
sembarangan.

Yang bukan termasuk kalimat kompleks dalam kalimat tersebut adalah…

A. Sekolah era new normal harus menetapkan peraturan yang ketat


B. Siswa dilarang jajan sembarangan dan harus bermain di dalam lingkungan sekolah.
C. Siswa dilarang jajan sembarangan dan harus bermain di dalam lingkungan sekolah.
D. Meski sekolah new normal baru berjalan beberapa bulan, nyatanya siswa masih sulit
diatur karena sepulang sekolah, mereka justru duduk-duduk di depan sekolah sambil
jajan sembarangan

28. Bacalah kutipan teks tanggapan berikut dengan saksama!

Meski sekolah new normal baru berjalan beberapa bulan, nyatanya siswa masih sulit diatur
karena sepulang sekolah, mereka justru duduk-duduk di depan sekolah sambil jajan
sembarangan.

Kata “mereka” pada teks tanggapan tersebut merujuk pada ....


A. Sekolah
B. siswa
C. .New normal
D. .Jalan

29. Bacalah penggalan teks tanggapan berikut!

Gadget juga memiliki dampak negatif bagi kesehatan tubuh, karena gadget
mengeluarkan banyak radiasi yang secara tidak langsung dapat merusak mata dan saraf
otak. Terlebih lagi, remaja tersebut tidak memperhatikan jarak antara mata dan gadget
saat mempergunakannya.

Namun, banyak juga sisi positif yang bisa didapatkan oleh remaja jika mereka
mempergunakan gadget dengan benar. Remaja bisa lebih mudah mencari informasi
mengenai tugas-tugas mereka di sekolah ketika informasi dari buku pelajaran dirasa
kurang. Tentunya, hal ini dapat meningkatkan prestasi dan pengetahuan mereka.
Dengan adanya gadget, orangtua juga bisa dengan mudah mengontrol anaknya saat
berada di luar rumah. Remaja bisa dengan mudah memberi kabar kepada orangtua jika
ada sesuatu yang mendadak. Selain itu, mereka juga bisa menghubungi teman maupun
sanak saudara mereka yang lokasinya berjauhan. Canggihnya gadget juga bisa membuat
remaja mengetahui perkembangan dunia IPTEK dengan mudah.

(Contoh teks tanggapan “Pengaruh Gadget Terhadap Remaja” dikutip dari detik.com
dengan beberapa penyesuaian.)

Kalimat yang menunjukkan penilaian pada kutipan teks tanggapan tersebut adalah…
A. Gadget juga memiliki dampak negatif bagi kesehatan tubuh, karena gadget
mengeluarkan banyak radiasi yang secara tidak langsung dapat merusak mata dan
saraf otak.

B. Canggihnya gadget juga bisa membuat remaja mengetahui perkembangan dunia


IPTEK dengan mudah.
C. Namun, banyak juga sisi positif yang bisa didapatkan oleh remaja jika mereka
mempergunakan gadget dengan benar
D. Tentunya, hal ini dapat meningkatkan prestasi dan pengetahuan mereka.

30. Penggunaan Galon Sekali Pakai


Untuk alasan kebersihan dan lebih steril, penggunaan galon sekali pakai saat ini
beredar cukup luas di kalangan masyarakat. Namun menurut saya gagasan ini merupakan
tindakan yang tidak mempedulikan efek dan dampaknya bagi lingkungan, yaitu
menyebabkan peningkatan sampah plastik.
Selama ini bumi kita sudah cukup kesakitan akibat berbagai pencemaran lingkungan
dan juga pemanasan global yang setiap tahunnya semakin memperburuk keadaan. Dilansir
dari NASA, tercatat suhu permukaan rata-rata bumi telah meningkat sekitar 2,05 derajat
Fahrenheit (1,14 derajat Celcius) sejak akhir abad ke-19.

Kalimat yang menunjukkan kritikan pada kutipan teks tanggapan tersebut adalah…

A. Untuk alasan kebersihan dan lebih steril, penggunaan galon sekali pakai saat ini
beredar cukup luas di kalangan masyarakat
B. Dilansir dari NASA, tercatat suhu permukaan rata-rata bumi telah meningkat sekitar
2,05 derajat Fahrenheit (1,14 derajat Celcius) sejak akhir abad ke-19.
C. Selama ini bumi kita sudah cukup kesakitan akibat berbagai pencemaran lingkungan
dan juga pemanasan global yang setiap tahunnya semakin memperburuk keadaan.
D. Namun menurut saya gagasan ini merupakan tindakan yang tidak mempedulikan efek
dan dampaknya bagi lingkungan, yaitu menyebabkan peningkatan sampah plastik.

B. ISIAN
31. Bacalah teks berikut
Nala dan Bila melakukan percobaan uji zat makanan di Laboratorium SMP Merdeka pada
tanggal 18 Februari 2023. Percobaan tersebut untuk mengetahui zat-zat berupa lemak, amilum,
dan glukosa yang terdapat dalam makanan. Bahan yang digunakan, yaitu mentega, tepung,
telur, kertas buram, lugol, dan benedict. Alat yang digunakan, yaitu tabung reaksi, gelas kimia,
pembakar spritus, pipet, mortar, dan alat tumbuk. Nala dan Bila pertama-tama menyiapkan alat
dan bahan yang diperlukan. Kemudian, mereka melakukan uji makanan.

Apa saja bahan dan alat yang diperlukan dalam percobaan tersebut?
Jawaban
Bahan : mentega, tepung, telur, kertas buram, lugol, dan benedict.
Alat : tabung reaksi, gelas kimia, pembakar spritus, pipet, mortar, dan alat tumbuk.
32. Bacalah kalimat rumpang berikut
Percobaan dilakukan dengan dua kelompok variabel, yaitu kelompok kontrol dan kelompok
eksperimen …. hasilnya dapat dibandingkan untuk menilai pengaruh variabel yang diuji.
Konjungsi yang tepat untuk melengkapi kalimat rumpang di atas adalah …
Jawaban Agar

33. Perhatikan penggalan teks pidato persuasif berikut!


Kita harus mengatur waktu tidur kita. Tidur cukup bisa membuat badan menjadi segar dan sehat.
Seseorang akan bersemangat saat memulai aktivitas di pagi hari jika waktu tidurnya cukup. Mari
kurangi begadang dan tidur yang cukup pada malam hari.
Tema penggalan teks pidato persuasif di atas adalah …
Jawaban Kesehatan

34.Bacalah kutipan teks pidato berikut ini!


Dalam kesempatan ini, perkenankan kami menyampaikan sebuah uraian tentang indahnya
kebersamaan. Kami berharap uraian ini akan memberikan manfaat kepada hadirin yang ada di
sini.
Uraian teks di atas merupakan kutipan teks pidato bagian…
Jawaban Pembuka

35.Cermati kutipan teks pidato persuasif di bawah ini.


Pikirkan masa depan. Tekunlah belajar
Kata emotif pada kutipan teks pidato persuasif tersebut adalah …….

36. Perhatikan kutipan teks cerpen berikut ini!

Tetapi, bagaimanapun juga, Kusno tak akan putus asa. Ia dilahirkan dalam kesengsaraan, hidup
bersama kesengsaraan. Dan meskipun celana 1001-nya lenyap, Kusno akan berjuang terus
melawan kesengsaraan, biarpun hanya untuk mendapatkan sebuah celana 1001 yang lain.

Tema pada kutipan cerpen tersebut adalah ….

Jawaban kesengsaraan

37. Bacalah kutipan cerpen berikut!

“Hai Gemblong!” sapa teman-teman Daryati dengan nada penuh candaan. Memang, ia sering
mendapatkan sapaan seperti itu di kelasnya, bahkan juga dari teman kelas lain. Tubuh Daryati
tidak gemuk. Sapaan itu diberikan karena ibunya memang seorang bakul gemblong.. Namun
untaian candaan itu tak pernah dipedulikannya meski secara jujur Daryati merasa tak
nyaman….
https://zuhriindonesia.blogspot.com/2018/12/
Penyebab konflik yang dialami tokoh pada kutipan cerita tersebut adalah …
Jawaban sering dipanggil dengan nada ejekan

38. Bacalah kutipan cerpen berikut!`

Sore yang begitu indah. Kulangkahkan kakiku menuju pantai.


Ombak yang berkejar kejaran menambah indahnya
pemandangan. Aku sangat bersyukur masih dapat berdiri di
sini.
Majas yang terdapat pada kutipan cerpen di atas adalah … .
Jawaban Personifikasi

39."Film ini memberikan pandangan yang baru terkait isu


lingkungan."
Objek apa yang ditanggapi dalam kutipan tersebut?
Jawaban Film

40. "Sementara ada beberapa fakta yang disajikan dengan baik, tetapi saya merasa ada
kekurangan informasi penting."
Bagian mana yang menunjukkan sanggahan dalam kutipan tersebut?
Jawaban saya merasa ada kekurangan informasi penting

C. URAIAN

41. Bacalah kutipan teks berikut!

Timbang sampel makanan (cilok) sebanyak 25 g, lalu haluskan. Masukkan ke tabung reaksi,
tambahkan akuades hangat sebanyak 50 ml. Kemudian, aduk dan biarkan mengendap sampai
jernih. Setelah mengendap, ambil air bagian atas sebanyak 5 ml dan masukkan ke tabung.
Tetesi air sampel dengan larutan formaldehyde 10 tetes dan homogenkan. Celupkan stik
indikator formalin pada air sampel selama 3 detik sampai basah sempurna dan keringkan.
Samakan warna stik indikator dengan range warna kandungan formalin. Makin pudar atau
pucat warna yang ditunjukkan pada stik indikator makin kecil pula kandungan formalin pada
makanan tersebut.
https://web.archive.org/web/20210217030525/https://www.academia.edu/39193312/LAPORAN_PRAKTIKUM_KIMIA_LINGKUNGAN_
UJI_FORMALIN_PADA_MAKANAN diakses 13 Februari 2021space

Apa tujuan kutipan teks laporan percobaan tersebut?


Jawaban: Menguji kandungan formalin pada cilok
42. Bacalah kutipan teks pidato berikut!
Saudara-saudara yang berbahagia,

Saat ini sering kita temui makanan yang mengandung formalin. Menurut ilmu kesehatan,
makanan yang kita konsumsi tidak boleh mengandung formalin. Mengapa? Alasannya
formalin adalah salah satu jenis zat pengawet yang digunakan dalam dunia kedokteran. Zat ini
disalahgunakan oleh pihak yang tidak bertanggung jawab untuk mengawetkan makanan.

Manakah kalimat yang menggunakan kata ilmiah pada kutipan tersebut !


Jawaban : Saat ini sering kita temui makanan yang mengandung formalin.

43. Bacalah Kutipan teks cerpen berikut !


Dua minggu setelah telegram dari Imam datang pula wesel dan surat dari Santo, ibu sangat bangga
sampai meneteskan air mata bahagia, ibu menyatakan pada anak-anaknya agar tetap hidup seperti
apa yang dilakukan selama ini. Jangan congkak karena kakak mereka banyak membantu.

Tentukan amanat yang terkandung dalam cuplikan cerpen tersebut!


Jawaban Hendaknya tetap hidup dalam kesederhanaan.

44.Bacalah kutipan teks cerpen berikut !

1. Malam ini Vina menengadahkan kepalanya, berusaha menyapa bulan. (2) Kendati
penantiannya telah habiskan lebih dari separuh kesabarannya, bulan tak kunjung membalas
sapanya. (3) Rupanya, awanlah yang menghalangi terangnya bulan.
Manakah majas personifikasi pada kutipan teks cerpen tersebut !
Jawaban :Malam ini Vina menengadahkan kepalanya, berusaha menyapa bulan.

45. Bacalah kutipan teks tanggapan berikut!

Kemajuan teknologi informasi membuat banyak perubahan, termasuk perkembangan


gadget di kalangan remaja. Banyak remaja yang menghabiskan waktunya untuk bermain
gadget seperti bermain game dan membuka situs-situs yang dilarang. Hal tersebut pasti
dapat membawa dampak dan pengaruh negatif terhadap perkembangan pola pikir dan
lingkungan remaja. Selain itu, hal ini juga dapat menurunkan konsentrasi mereka dalam
belajar.

Namun, banyak juga sisi positif yang bisa didapatkan oleh remaja jika mereka
mempergunakan gadget dengan benar. Remaja bisa lebih mudah mencari informasi
mengenai tugas-tugas mereka di sekolah ketika informasi dari buku pelajaran dirasa
kurang. Tentunya, hal ini dapat meningkatkan prestasi dan pengetahuan mereka.
https://www.kompas.com/skola/read/2022/02/16/080000369/teks-tanggapan-pengertian-fungsi-struktur-dan-
contonya

Tuliskan tanggapan yang sesuai dengan kutipan teks tersebut adalah!


Kunci Jawaban:
Secara umum, saya setuju dengan alasan-alasan yang mendukung penggunaan gadget di
lingkungan remaja. Akan tetapi, ada juga dampak negatif yang bisa timbul akibat pemakaian
gadget. Jadi, sebaiknya para orang tua tidak memberikan kebebasan dan memberikan batasan
tertentu kepada anaknya untuk bermain dan memiliki gadget, karena gadget dapat membawa
dampak negatif pada kehidupan remaja.

Soal Baru !

1. Meskipun hanya meraih satu medali emas dalam Olimpiade Fisika


Internasional di Pohang, Korea Selatan 14-22 Juli 2004, Indonesia tidak
perlu berkecil hati. Emas persembahan Yudistira Virgus itu
menempatkan Indonesia sebagai salah satu negara yang disegani dalam
perkembangan ilmu fisika. Gagasan besar pokok paragraf tersebut
adalah..
a. Mahalnya harga sekeping medali emas
b. Keberhasilan Indonesia dalam Olimpiade Fisika Internasional
c. Yudistira Virgus mempersembahkan satu medali emas
d. Indonesia disegani dalam perkembangan ilmu fisika
2. Bangunan sisa kolonial Belanda itu berdiri di pusat kota, di balik pagar
seng. Ada yang mengatakan dindingnya kokoh, meski termakan usia,
seakan berbicara betapa banyak sejarah yang telah dilaluinya.
Dindingnya mungkin sudah lapuk ditumbuhi tanaman liar dan lumut.
Menurutnya, catnya pun tidak terlihat lagi, menunjukkan bahwa tak
pernah ada tangan manusia yang merawatnya.

Kalimat berisi fakta terdapat pada kalimat.. a. Pertama b. Kedua c.


Ketiga d. Keempat Jawab: A

3. Fadhila akan menulis sebuah artikel dengan topik Upaya Peningkatan


Keberhasilan Kelas. Masalah yang tepat untuk dibahas dalam artikel
tersebut adalah.. a.
Bagaimanakah cara meningkatkan kebersihan kelas?
b. Bagaimanakah kebersihan kelas kita?
c. Bagaimanakah kebersihan kelas di masa mendatang?
d. Bagaimanakah kebersihan kelas kita masing-masing? Jawab: A

4. ... memiliki fungsi mempermanis penampilan. Istilah yang tepat untuk


melengkapi kalimat tersebut adalah..
a. Souvenir b. Aksesori5.

Untukmu Sahabatku...
(1) Di keheningan malam ini (2) Kucoba hayati irama detak jantungku (3)
Karena tenggelamnya perahu persahabatan (4) Bilakah kau akan
kembali (5) Sejak kepergianmu pahit hidupku
5. Larik puisi tersebut yang menggunakan sinestesia ditandai nomor.. a.
(2) b. (3 c. 4 d. 1

6.
1.Kepala sekolah akan menyetujui usul ini ……………….2. Kau
mengembalikan semua uang yang kau terima
Kata penghubung yang tepat untuk menggabungkan kedua kalimat
tersebut adalah.. a. Oleh karena itu b. Sebab c. Asalkan d.
Sebagaimana

7. Kapten kesebelasan Persija berkata bahwa kesebelasan lawan


memang tangguh." b. Kapten kesebelasan Persija berkata:
"Kesebelasan lawan memang tangguh. c. "Kesebelasan lawan memang
tangguh." Kata kapten kesebelasan Persija d. "Kesebelasan lawan
memang tangguh." kata kapten kesebelasan Persija
Cermati kutipan teks berikut!
Kebiasaan menyontek justru akan memadamkan semangat belajar. Para penyontek biasa berpikir
untuk menyiapkan bahan-bahan untuk membuat sontekan, nilai yang diperoleh pun kadang lebih
bagus dari siswa yang jujur. Nilai boleh saja bagus, tetapi sebenarnya ia tidak tahu apa-apa dan
tidak ada apa-apanya. Ilmu pengetahuannya tidak pernah bertambah karena rajin menyontek.
Lalu apa bedanya dengan mereka yang tidak pernah sekolah?

1. Simpulan isi penggalan teks tanggapan tersebut adalah...


A. Hindari menyontek karena tidak akan menjadikan kita pandai. v
B. Menyontek justru akan memadamkan semangat belajar.
C. Ilmu pengetahuan tidak akan bertambah karena menyontek.
D. Kebiasaan menyontek membuat tidak tahu apa-apa.

2. Contoh kalimat yang menyatakan hasil kegiatan percobaan adalah ....


A. Banyak lubang di sepanjang jalan itu.
B. Tidak lama lagi, musim hujan akan turun di daerah kita
C. Sepertinya perlu sistem pengelolaan sampah yang lebih baik dengan melibatkan banyak pihak.
D. Karena terkena cahaya, pertumbuhan tanaman itu, menjadi lebih baik v

Cermati kutipan teks berikut!


Berkali-kali setiap hari kita memutar gagang untuk mengalirkan air bersih. Dengan menekan
gagang yang lain kita menyiram kotoran badan kita. Akan tetapi, seberapa seringkah kita
memikirkan sistem yang kompleks dalam penyediaan air dan pembuangan kotoran kota?
Sumber air kota biasanya berasal dari tempat yang jauh, tetapi sistem pembuangan kotorannya
memasuki perairan-perairan yang dekat. Jika pengaruh zat dalam ekosistem air tidak mampu
menanggulangi banyak buangan kotoran, pantai dan sumber-sumber dalam wilayah yang jauh
searah dengan aliran kota akan menjadi tercemar.

3. Kritikan logis terhadap isi bacaan di atas adalah….


A. Seharusnya kita tetap harus memperhatikan sistem pembuangan kotoran..
B. Kita tidak harus mandi menggunakan air bersih.
C. Seharusnya kita tidak menggunakan sumber air dari pegunungan.
D. Sistem pembuangan kotoran memasuki perairan yang dekat sangat merusak lingkungan v

Cermati kutipan teks berikut!


Kegiatan membaca merupakan upaya dalam …..informasi dari bacaan hal tersebut merupakan
langkah awal dalam upaya memperkaya diri dengan ilmu pengetahuan langkah selanjutnya
adalah…..upaya pemahaman yang tepat terhadap isi bacaan.
4. Isian yang tepat adalah….
A. menyerap dan melakukan.. v B. menyerap dan dilakukan
C. menterapkan dan melakukan. D. penyerapan dan dilakukan

Cermati kalimat berikut!


Para hadirin dipersilakan mencari tempat duduk sesuai dengan nomornya.
5. Perbaikan kalimat tersebut agar efektif adalah...
A. Hadirin silakan mencari tempat duduk sesuai nomor. v
B. Hadirin dipersilakan duduk sesuai dengan nomornya.
C. Para hadirin dipersilakan mencari kursi sesuai nomornya.
D. Hadirin dipersilakan mencari tempat duduk dengan nomornya
Cermati kutipan teks berikut!
Husin adalah seorang anak dari keluarga yang sederhana. Orang tuanya tidak mampu untuk
membiayai Husin untuk melanjutkan pendidikan ke perguruan tinggi. Walaupun orang tuanya tidak
mampu membiayai dirinya ke perguruan tinggi, Husin tetap semangat belajar. Karena Husin tetap
semangat belajar dan tidak putus asa, Husin berhasil meraih beasiswa penuh untuk melanjutkan
pendidikan ke perguruan tinggi.
6. Pesan yang ingin disampaikan penulis dalam teks cerita inspiratif di atas adalah...
A. cari peluang lain dan tidak perlu berusaha melanjutkan pendidikan jika orang tua tidak mampu
B. cari pekerjaan saja jika sudah tidak ada kemampuan ekonomi untuk melanjutkan pendidikan
C. cari beasiswa dan mintalah bantuan ke pihak yang mampu untuk membantu biaya pendidikan
D. tetap semangat dan selalu berusaha menghadapi tantangan/masalah untuk meraih cita-cita
v

Cermati kutipan teks berikut!


Buku kumpulan cerpen senyum karyamin karya Ahmad Tohari terbitan PT. Gramedia berisi 13
cerpen. Kesemua cerpen tersebut diramu dengan bahasa yang lancar dengan selingan sealak-
alek jawa. Lah-lho gusti pangeran dan lain-lain. Meskipuan begitu, buku ini juga cocok untuk
konsumsi orang lain selain keluarga jawa terutama orang yang ingin mengetahui kultur tempat
cerpen itu disampulnya bergambar seorang anak muda yang sedang memakul kampak sebagai
tanda optimisme kerja.
7. Tanggapan tersebut berisi tentang …..
A. isi buku dan permasalahnya. B. kemenarikan buku dan perwajahannya
C. bahasa dan perwajahan/sampul buku.. v D. judul buku dan pengarangnya

Cermatilah teks fabel berikut untuk menjawab soal nomor 8&9!


...Kura-kura lalu meminta bantuan kepada monyet.
"Maukah kau membantuku memetik buah pisang ini?" tanya kura-kura.
"Aku bersedia, tetapi buah pisang itu nanti dibagi dua," jawab monyet.
"Baik!" jawab kura-kura. Monyet lalu memanjat pohon pisang kura-kura. Bau harum buah pisang
menggoda selera monyet. la lupa akan janjinya. Kura-kura menunggu di bawah pohon pisang.
"Nyet, Nyet, mana pisang bagianku?" teriak kura-kura.
"Sebiji pun tidak ada," jawab monyet rakus.
"Nyet, ini pohon pisangku!" rengek kura-kura hampir menangis.
"Salah sendiri mengapa tidak bisa memanjat pohon!" ejek monyet.
Raut wajah Kura-kura memerah, ia mulai menangis. Hatinya sedih bercampur marah. la lalu
menggoyang-goyang pohon pisang itu. Tiba-tiba .... bruk! Pohon pisang itu tumbang. Monyet itu
jatuh. Dia mengerang kesakitan. Tubuhnya tertimpa batang pohon pisang.
"Ampun kura-kura, tolong aku! Aku menyesal ...” kata monyet.
Tetapi, kura-kura sudah berlalu. la mencari sahabat baru.

8. Pernyataan yang sesuai dengan kutipan teks fabel tersebut adalah...


A. Monyet meminta Kura-kura menolong dirinya dan berjanji akan mengembalikan pisangnya.
v
B. Kura-kura yang merasa sedih itu meninggalkan Monyet yang masih berada di atas pohon.
C. Kura-kura bersedia membantu Monyet asalkan buah pisang hasil petiknya dibagi dua.
D. Monyet tidak memenuhi janji membagi pisang sehingga Kura-kura merasa sedih dan marah.

9. Makna simbol wajahnya merah pada teks fabel tersebut adalah...


A. marah v B. terharu C. senang D. malu

Perhatikan kutipan teks drama berikut!


Ara : To, kamu dengar tidak, kalau kampung sebelah ada yang terkena virus Corona?
Anto : Ya, aku sempat dengar dari beberapa warga. Pak RT juga sudah sering sosialisasi tentang
waspada virus Corona.
Ara : Oh, yang cuci tangan pakai sabun dan pakai masker itu ya?
Anto : [....]
Ara : Wah, begitu yah? Itu sesuai juga dengan anjuran pemerintah ya?
Anto : Iya, Ra. Karena itu, sebaiknya kamu jangan dekat-dekat aku sekarang.
Ara : Hahaha, kamu bisa saja Ra.

10. Dialog yang tepat untuk melengkapi bagian rumpang kutipan teks drama tersebut
adalah...
A. Ya, kamu harus pakai sabun yang bagus dan makser yang mahal agar aman.
B. Jangan lupa bawa disinfektan dan pakai setiap kamu pegang benda atau bersentuhan dengan
orang
lain.
C. Bukan cuma itu, kita juga harus mengkonsumsi makanan sehat dan bergizi. Jangan lupa juga
untuk
jaga jarak atau social distancing. v
D. Selain itu, kamu juga jangan banyak berpergian. Lebih baik diam di rumah dan isolasi mandiri.

Cermati kutipan teks dialog berikut!


Ulangan sedang berlangsung. Suasana benar-benar hening. Para siswa tampak fokus pada soal
yang dihadapinya. Pak Rudin duduk sambil sesekali melempar pandangan ke luar Ruang 09.
Bandi : Din, aku minta jawaban soal nomor 8 dan 9!
Andin : A dan D!
Ita : […. ]
Banu : 10 A, 11 D, nomor 15 aku belum!
Adi : Huss, jangan kencang-kencang nanti pengawasya dengar!
Ita : Soalnya sulit sekali, masih banyak yang belum aku kerjakan.

11. Kalimat yang tepat untuk melengkapi kutipan teks drama tersebut adalah...
A. Ulangan sedang berlangsung. Jangan rebut-ribut dong!
B. Kalau soal nomor 10, 11, dan 15 jawabannya apa, Ban? v
C. Soalnya sulit sekali, masih banyak yang sudah aku kerjakan
D. Memang betul jawabannya tidak sulit ?

Cermati kalimat berikut!


Para hadirin dipersilakan mencari tempat duduk sesuai dengan nomornya.
12. Perbaikan kalimat tersebut agar efektif adalah...
A. Hadirin silakan mencari tempat duduk sesuai nomor. v
B. Hadirin dipersilakan duduk sesuai dengan nomornya.
C. Para hadirin dipersilakan mencari kursi sesuai nomornya.
D. Hadirin dipersilakan mencari tempat duduk dengan nomornya.

Bacalah kalimat-kalimat berikut dengan cermat!


(1) Mentega dan minyak mengandung lemak karena saat dioleskan ke kertas buram, kertas
tersebut berubah menjadi transparan.(2) Kita dapat mempunyai slime yang aman digunakan
tanpa menggunakan boraks setelah melakukan percobaan ini.(3) Ragi dikenal sebagai bahan
yang umum digunakan dalam fermentasi untuk menghasilkan etanol.(4) Manfaat percobaan ini
adalah dapat mengetahui perubahan berat kentang yang dimasukan pada larutan gula sebanyak
15 persen dan 50 persen, dan pada air biasa.

13. Kalimat kompleks (majemuk) yang mengandung hubungan penyebaban ditandai nomor ....
A. (1) v B. (2) C. (3) D. (4)
Bacalah kutipan teks dibawah ini!
Ayahku adalah seorang laki-laki yang lumayan tampan. Saat ini, usianya 35 tahun. Dia memiliki
tubuh atletis karena selalu berolah raga setiap hari. Dia memiliki tinggi 170 cm. Dia memiliki
rambut pendek hitam, mata yang hitam, berwajah oval dan dagu yang lancip. Dia memiliki kumis
dan jenggot yang tipis. Warna kulitnya sedikit coklat.
14. Informasi yang terdapat dalam teks deskripsi tersebut adalah ….
A. hobi ayah B.sosok ayah v C. keburukan ayah D. istri
ayah

Bacalah paragraf-paragraf berikut dengan cermat!

1. Melalui media cetak dan elektronik kita dapat menyaksikan bagaimana para pengedar narkoba
membangun jaringan secara intensif. Akibatnya, peredaran narkoba begitu merebak hingga ke
daerah-daerah pelosok bahkan sampai ke lembaga pemasyarakatan.
2. Mungkin banyak di antara hadirin yang belum memahami isu globalisasi. Untuk itu, kali ini saya
ingin mengajak hadirin untuk mengingat kembali beberapa pola hidup yang dapat mengakibatkan
globalisasi, apa pengaruhnya bagi kita semua, dan bagaimana kita menghadapi pengaruhnya.
Idealnya, biaya pembangunan di suatu negara harusnya dibiayai oleh rakyat di negara tersebut
melalui setoran pajak.
3. Inilah mengapa dikatakan pajak sebagai harga diri bangsa. Karena pajak sebagai harga diri
bangsa, seharusnya setiap wajib pajak merasa bangga setelah melakukan kewajibannya kepada
negara.
4. Langkah pertama adalah dengan memasukkan air hangat dengan takaran sesuai selera.
Selanjutnya, mencelupkan teh gantung ke dalam gelas berisi air hangat. Jangan lupa
memasukkan gula secukupnya kemudian aduk hingga larut. Teh siap di sajikan.

15. Paragraf-paragraf tersebut termasuk kutipan teks pidato persuasif, kecuali yang ditandai
dengan nomor ....
A. (1) B. (2) C. (3) D. (4) v

16. unsur cerpen yang berkaitan dengan tempat cerita berlangsung, kapan terjadinya cerita, dan
dalam keadaan bagaimana cerita tersebut terjadi disebut dengan…
A. Alur B. Tema C. Latar v D. Tokoh

Cermati paragraf berikut!


(1) Letusan atau erupsi gunung Merapi sangat dasyat hingga berdampak pada masyarakat.
(2) Pada saat musim hujan, akan sering terlihat warna-warni bercahaya di ujung cakrawala.
(3) Banjir berpotensi mengakibatkan kerusakan pada berbagai aspek kehidupan dan properti.
(4) Gempa tektonik disebabkan oleh pergeseran lempeng-lempeng tektonik secara mendadak.
17. Kata bergaris bawah yang bersinonim terdapat pada kalimat bernomor...
A. (4) B. (3) C. (2) D. (1) v

Cermati teks diskusi berikut untuk menjawab soal nomor 18-19 !

Dampak Bermain Game Online pada Anak


Istilah game online sudah tidak asing lagi bagi anak-anak dan remaja masa kini. Dengan daya
jelajah internet yang sangat luas dan tanpa batas, game online sudah bisa dinikmati kapan pun
dan di mana pun. Dampaknya pun kini mulai dirasakan banyak pihak. ( .... ) Akan tetapi, banyak
juga pihak yang berpendapat bahwa game online juga memiliki sisi positif. Dengan demikian,
game online kini menimbulkan pro-kontra di masyarakat.

18. Kalimat yang tepat untuk melengkapi bagian yang rumpang dalam paragraf tersebut adalah...
A. Terutama para orang tua yang memiliki anak remaja yang setiap hari gemar bermain game
online.
B. Ada pihak yang beranggapan bahwa game online banyak menimbulkan dampak negatif.
v
C. Karena itu, persoalan ini tentunya memerlukan perhatian dan pengawasan dari para orang tua.
D. Anak-anak dan remaja pun kini bisa bermain game online melalui telepon genggam mereka.

19. Variasi judul yang sesuai dengan isi teks tersebut di no 18 adalah...
A. Hubungan Game Online dan Anak di Masyarakat
B. Dampak Anak yang Bermain Game Online di Masyarakat
C. Pengaruh Bermain Game Online Bersama Anak
D. Pro dan Kontra Pengaruh Game Online pada Anak ….v

20. Kalimat di bawah ini yang menggunakan kata hubung yang tepat adalah …
A. Lebih baik Shalat daripada tidur v
B. Ia salah satu pengusaha daripada yang berhasil
C.Orang tua daripada Abdullah mendapat kecelakaan
D.Bahasa daripada orang itu tidak dapat dimengerti

Cermati teks berikut!


Amir, Ridho, Ali, dan Surya tergabung dalam regu piket hari Kamis. Surya selalu datang terlambat
ke sekolah sehingga jarang melaksanakan piket pagi. Saat teman-temannya sibuk membersihkan
kelas, Surya juga tidak mau membantu.
21. Berdasarkan ilustrasi di atas, ungkapan yang tepat untuk menunjukan sifat Surya adalah...
A. berat tangan v B. panjang tangan C. ringan tangan D. cuci
tangan

Bacalah paragraf berikut untuk menjawab soal nomor 22,23 dan 24!
Untuk menghindari peredaran dan penyalahgunaan narkoba di kalangan peserta didik, kesadaran
dan pengetahuan tentang bahaya narkoba perlu ditanamkan. Banyak cara yang dapat dilakukan,
baik melalui penyampaian materi, forum diskusi, maupun dalam bentuk pemberian tugas-tugas
sekolah.Pemberian tugas sekolah dapat dilakukan guru dalam bentuk tugas membuat poster,
cerpen, atau tulisan yang bertema anti narkoba.
Dengan cara-cara di atassecara tidak langsung para peserta didik akan mengenal dampak
penyalahgunaan narkoba. Melalui pengenalan itu, terbentuklah pola pikir untuk menjauhi
penggunaan narkoba pada diri peserta didik.
22. Kutipan teks pidato tersebut bertopik ....
A. peredaran dan penyalahgunaan narkoba di indonesia
B. pemberantasan mata rantai peredaran narkoba
C. antisipasi penyalahgunaan narkoba di kalangan peserta didik v
D. pemberlakuan hukuman bagi pengedar narkoba

23. Kutipan teks pidato tersebut di no 22 bertujuan mengajak pembaca/pendengar untuk ....
A. membantu polisi dalam pemberantasan peredaran narkoba
B. mengantisipasi penyalahgunaan narkoba di kalangan peserta didik
C. mengajukan usul pemberlakukan hukuman seberat-beratnya kepada pengedar narkoba
D. berupaya aktif dalam upaya penanganan penyalahgunaan narkoba v

24. Struktur dalam teks diskusi adalah ....


A. Pendahuluan, Isi & Nasihat B. Pendahuluan, Isi & Simpulan v
C. Pendahuluan, Simpulan & Isi D. Simpulan, Isi & Nasihat

25. Kalimat pro di bawah ini adalah…


A. Saya senang sekali dengan adanya kegiatan pentas seni ini. v
B. Apa yang disampaikannya sama sekali tidak sesuai dengan hatiku.
C. Lebih baik ditinjau ulang kembali saran yang sudah disampaikan itu.
D. Pendapatmu itu sangat menentang aturan yang ada.

Cermati kutipan teks cerita inspiratif berikut !


”Bu..., aku memberi sedekah kepadanya sebanyak itu. Saat menerimanya, ia berucap hamdalah
berkali-kali seraya bersyukur kepada Allah. Tidak itu saja, ia mendoakan aku, mendoakan dirimu,
anak-anak, dan keluarga kita. Panjang sekali ia berdoa! Dia hanya menerima karunia dari Allah
Swt. sebesar 10 ribu saja sudah sedemikian hebatnya bersyukur. Padahal, aku sebelumnya
melihat di ATM saat aku mengecek saldo dan ternyata di sana ada jumlah yang mungkin ratusan
bahkan ribuan kali lipat dari 10 ribu rupiah. Saat melihat saldo itu, aku hanya mengangguk-angguk
dan tersenyum. Aku terlupa bersyukur, dan aku lupa berucap hamdalah.
26. Struktur cerita di atas termasuk ….A. Orientasi B. Resolusi v C. Komplikasi D.
koda

27. Contoh judul yang tidak tepat dari cerita inspiratif adalah …
A. Aku dan Impianku B. Cara Membuat Pepes Udang
v
C. Sedekah Uang Sepuluh Ribu Rupiah C. Perjuangan Untuk Diriku Sendiri

Cermati teks berikut!


Pada saat kita tidur, baik siang maupun malam hari, selalu saja ada makhluk kecil bersayap
bernama nyamuk. Hal ini mungkin hal yang sepele bagi sebagian orang dan dapat diatasi dengan
menyemprotkan cairan pembasmi nyamuk yang banyak beredar di pasar. Namun, tahukah Anda
jika cairan pembasmi tersebut mengandung berbagai bahan kimia yang dapat membahayakan
kesehatan kita? Untuk mengatasi hal tersebut, kini terdapat cara alternatif yang lebih aman untuk
menangkap dan memerangkap nyamuk. Cara membuatnya cukup sederhana dan bahan untuk
membuatnya pun dapat kita temukan dalam kehidupan sehari-hari.

28. Judul yang tepat untuk melengkapi kutipan teks laporan hasil percobaan di atas adalah ....
A. Cara Membuat Perangkap Alternatif dan Aman
B. Cara Aman Membuat Cairan Pembasmi Nyamuk v
C. Cara Alternatif Membuat Nyamuk Terperangkap
D. Cara Membuat Perangkap Nyamuk Sederhana

Cermati kutipan teks berikut!


OSIS SMP PGRI Kalimulya mengadakan kegiatan gotong royong membersihkan parit, selokan,
dan sampah di sekitar lingkungan. Kegiatan ini bertujuan agar lingkungan sekitar tetap sehat dan
bersih.

29. Slogan yang sesuai dengan ilustrasi tersebut adalah …


A. Gotong royong menjalin kerja sama sesama warga.
B. Lingkungan bersih dan sehat tanggung jawab kita bersama. v
C. Mari bersama-sama bergotong royong di lingkungan sekitar.
D. Hidup sehat dan bersih merupakan cita-cita warga bersama.

Bacalah teks berikut dengan cermat!


(1) Apa yang telah dilakukan oleh para pendahulu kita sangatlah penting untuk diteladani.
(2) Mereka sadar bahwa kaum mudalah yang akan menentukan nasib bangsa ini. (3)
Untuk itulah, kita sebaga pemuda harus meneruskan semangat persatuan mereka. (4)
Ayo, wahai pemuda harapan bangsa, kita songsong masa depan yang lebih cerah untuk
nusa dan bangsa kita!

30. Kalimat emotif pada teks terdapat pada no .... A. (1) B. (2) v C. (3) D.
(4)
Cermati kutipan teks cerita inspiratif berikut !
Deggg...!!! Hati Budiman tergedor dengan begitu kencang. Rupanya wanita tadi sungguh berharap
tambahan sedekah agar ia dan putrinya bisa makan. Sejurus kemudian mata Budiman
membuntuti kepergian mereka berdua yang berlari menyeberang jalan, lalu masuk ke sebuah
warung tegal untuk makan di sana.
31. Struktur cerita di atas termasuk ….A. Orientasi B. Resolusi C. Komplikasi v D.
koda

Cermati kutipan teks berikut!


Irfan anak dari keluarga yang kaya. Ia pandai di sekolahnya. Namun, ia tidak sombong
dan mau berteman dengan siapa saja. Sikap Irfan tersebut membuat teman- temannya
suka kepadanya. Irfan memang anak yang [ ...].
32. Ungkapan yang sesuai untuk melengkapi teks tersebut adalah ....
A. besar hati B. rendah hati v C. besar kepala D. hati emas

33. Dalam kutipan teks cerita di no. 32 di atas lebih menonjolkan unsur …
A. tema B. alur C. tokoh v D. latar

Cermati kutipan teks pantun yang rumpang berikut!


(1) Melihat perahu memasang layar
(2) ...
(3) Siswa baru sedang rajin belajar
(4) ...

34. Isian Larik/baris (no 2 dan 4) yang tepat untuk melengkapi bagian teks pantun tersebut
adalah...
A. (2) Pergi ke laut memancing ikan (4) Tidak lupa membaca buku
B. (2) Cuaca terang tiada terkira (4) Tetap semangat dan gembira
C. (2) Siap-siap juga membawa jaring (4) Tetap di rumah secara daring v
D. (2) Matahari terik memancar (4) Baca cerita diam di kamar

Cermati teks laporan hasil percobaan berikut!


Daun salam memiliki berbagai khasiat. Hal ini merupakan berita menggembirakan bagi orang-
orang yang kurang suka dengan obat-obatan kimiawi. Ada berbagai manfaat daun salam, antara
lain meningkatkan kesehatan jantung, mencegah stroke, menurunkan tekanan darah, dan
menurunkan kolesterol. [ ... ] Ternyata daun salam juga dapat mencegah berbagai macam kanker.

35. Kalimat yang tepat untuk melengkapi paragraf tersebut adalah ...
A. Ada beberapa langkah mengolah daun salam untuk mengobatan berbagai penyakit.
B. Selain manfaat tersebut, masih ada satu manfaat yang belum diketahui banyak orang. v
C. Penyakit yang bisa disembuhkan dengan daun salam adalah penyakit-penyakit ringan.
D. Para penderita cukup minum rebusan daun salam dalam takaran yang telah ditentukan

36. Majas personifikasi terdapat pada kalimat di bawah ini ....


A. Bulan menyapaku yang sedang termenung sendiri. v
B. Aku tak sudi berteman dengan orang yang sombong itu.
C. Dia memang anak yang tak bernyali besar.
D. Dia bagaikan tong kosong nyaring bunyinya.
37. Yang tidak termasuk Majas Hiperbola di bawah ini adalah…
A. Nina tukang bohong v
B. Adikku paling pintar sejagat alam raya ini.
C. Jantungku hampir copot saat melihat Agus lewat di depanku.
D. Langit seakan runtuh saat kudengar dia telah tiada.

Cermati kutipan teks berikut!

Teks 1 : Karimunjawa di Jepara berada di sebelah utara Pulau Jawa. Lokasi wisata bahari yang
satu ini sejak dulu sudah menjadi salah satu alternatif liburan yang sangat diinginkan oleh banyak
orang. Fasilitas di sana sangat lengkap. Bahkan, banyak juga backpacker yang berani untuk
berwisata ke sana dengan budget rendah.

Teks 2 : Kota Batu identik sebagai Kota Apel. Karena besarnya jumlah pengunjung untuk
berwisata di Kota Batu, potensi perkebunan apel juga dikembangkan sebagai objek wisata oleh
masyarakat sekitar. Saat ini, perkebunan apel yang tersebar di Kota Batu telah menjadi salah satu
objek wisata favorit, terutama bagi wisatawan lokal.

38. Persamaan isi kedua kutipan teks tersebut adalah ....


A. menjelaskan ketertarikan wisatawan luar negeri B. membahas penelitian pengembang
wisata
C. menggambarkan objek wisata domestic v D. menguraikan jenis wisata bahari di
Jawa

39. Kata yang bukan terdapat dalam kalimat aktif di bawah ini yaitu …
A. ditandai v B. memakan C. tertawa D. belajar

Cermati kutipan teks cerita inspiratif berikut!

Ada seorang sahabat menuturkan kisahnya. Dia bernama Budiman. Sore itu ia menemani istri dan
seorang putrinya berbelanja kebutuhan rumah tangga bulanan di sebuah toko swalayan. Usai
membayar, tangan-tangan mereka sarat dengan tas plastik belanjaan.

40. Struktur cerita di atas termasuk …. A. koda v B. Resolusi C. Komplikasi D.


Orientasi

B. Soal Isian Singkat !

Perhatikan urutan laporan percobaan berikut!


(1) Alat dan bahan (2) Tujuan (3) Hasil Pengamatan (4) Prosedur / Cara Kerja (5)
Simpulan

41. Sistematika/urutan laporan percobaan yang benar adalah nomor ..


42. Hikmah yang dapat diambil pelajarannya dalam cerita inspiratif disebut .

Cermati kutipan berikut !


Pakar transportasi mengatakan bahwa pemerintah lebih baik membangun infrastruktur transportasi
laut … udara karena Indonesia merupakan negara kepulauan.
43. Kata penghubung/konjungsi yang tepat untuk melengkapi kalimat tersebut adalah….

44. Bagian yang menyampaikan kisah tokoh inspiratif sebelum mengalami konflik disebut….
Cermati kutipan berikut !
(1)Seraya bersujud Budi berucap “Tuhanku, kuserahkan semua kepada-Mu. (2) Budi menyapu air
mata adiknya sambil berkata, “Diamlah, Gus, jangan menangis. Ini aku bawakan nasi bungkus”.
(3) Agus menerima bungkusan, lalu makanlah ia dalam gelap gulita itu. (4) Budi pun termenung
dalam kegelapan malam.

45. Bukti nilai agama terdapat dalam kalimat bernomor …

C. Soal Essay !

46. Struktur dalam pidato persuasif terdiri dari Pernyataan Posisi, Tahap Argumen dan
Penguatan Pernyataan Posisi. Jelaskan apa yang dimaksud dengan Pernyataan Posisi !

47. Ceritakan pengalamanmu (sebanyak dua paragraph) siapa dan bagaimanakah tokoh di
sekitarmu yang selalu menjadi inspirasi bagi kamu selama menjadi siswa di sekolahmu!
(Bebas mengambil tokoh siapa saja)

48. Tuliskan satu paragraf yang mengandung unsur cerita latar tempat, waktu dan suasana!

49. Tulis sebuah kalimat kritikan dan pujian saat menonton penampilan drama temanmu !

50. Sebutkan tiga macam cara memeruasi dalam pidato persuasif !

Selamat Berjuang! Nn.

1. Cermati kalimat berikut!


Presiden Joko Widodo menetapkan pandemi virus Corona sebagai bencana nasional.
Makna kata bergaris bawah adalah...
A. wabah yang menular dengan sangat cepat
B. wabah yang menjangkiti warga di suatu daerah tertentu
C. wabah yang diprediksi akan menyebar luas di kalangan tertentu
D. wabah yang berjangkit serempak di mana-mana mencakup daerah yang luas
Cermati kutipan teks berikut untuk menjawab soal nomor 2 dan 3!

Manajer pariwisata Taman Seroja memperkirakan bahwa sekitar 27.000 pelancong akan datang ke
lokasi ini pada akhir liburan semester tahun ini. Prediksi ini tentu tidak berlebihan karena
mencerminkan jumlah pelancong yang bepergian selama musim liburan tahun lalu. Untuk
meningkatkan minat dan daya tarik para pelancong, administrator akan menyiapkan berbagai barang
hiburan baru seperti wahana air, pertunjukan musik, sulap, dan banyak lagi.

2. Serangkaian pertanyaan berikut, jawaban yang ada dalam paragraf di atas, adalah …
a. Dari mana datangnya wisatawan yang mengunjungi taman?
b. Berapa banyak wisatawan yang diprediksi oleh Manajer Taman Lotus Park?
c. Hiburan macam apa yang paling populer di antara berbagai wahana bagi wisatawan di taman?
d. Siapakah tokoh nasional yang hadir pada pembukaan Wisata Taman Lotus?

3. Gagasan utama dalam paragraf di atas adalah …


a. Ada wahana air tambahan di taman objek wisata
b. Memprediksi peningkatan jumlah wisatawan di taman tahun ini
c. Taman Seroja adalah tempat wisata terbaik di Indonesia
d. Beberapa taman baru taman ini akan menarik minat para pelancong

4. Cermati kutipan teks berikut!

Kebiasaan menyontek justru akan memadamkan semangat belajar. Para penyontek biasa berpikir
untuk menyiapkan bahan-bahan untuk membuat sontekan, nilai yang diperoleh pun kadang lebih
bagus dari siswa yang jujur. Nilai boleh saja bagus, tetapi sebenarnya ia tidak tahu apa-apa dan tidak
ada apa-apanya. Ilmu pengetahuannya tidak pernah bertambah karena rajin menyontek. Lalu apa
bedanya dengan mereka yang tidak pernah sekolah?
Simpulan isi penggalan teks tanggapan tersebut adalah...
A. Hindari menyontek karena tidak akan menjadikan kita pandai.
B. Menyontek justru akan memadamkan semangat belajar.
C. Ilmu pengetahuan tidak akan bertambah karena menyontek.
D. Kebiasaan menyontek membuat tidak tahu apa-apa.

5. Cermati kutipan teks berikut!


Husin adalah seorang anak dari keluarga yang sederhana. Orang tuanya tidak mampu untuk
membiayai Husin untuk melanjutkan pendidikan ke perguruan tinggi. Walaupun orang tuanya tidak
mampu membiayai dirinya ke perguruan tinggi, Husin tetap semangat belajar. Karena Husin tetap
semangat belajar dan tidak putus asa, Husin berhasil meraih beasiswa penuh untuk melanjutkan
pendidikan ke perguruan tinggi.
Pesan yang ingin disampaikan penulis dalam teks cerita inspiratif di atas adalah...
A. cari peluang lain dan tidak perlu berusaha melanjutkan pendidikan jika orang tua tidak mampu
B. cari pekerjaan saja jika sudah tidak ada kemampuan ekonomi untuk melanjutkan pendidikan
C. cari beasiswa dan mintalah bantuan ke pihak yang mampu untuk membantu biaya pendidikan
D. tetap semangat dan selalu berusaha menghadapi tantangan/masalah untuk meraih cita-cita

Cermatilah teks fabel berikut untuk menjawab soal nomor 6, 7, dan 8!


...Kura-kura lalu meminta bantuan kepada monyet.
"Maukah kau membantuku memetik buah pisang ini?" tanya kura-kura.
"Aku bersedia, tetapi buah pisang itu nanti dibagi dua," jawab monyet.
"Baik!" jawab kura-kura. Monyet lalu memanjat pohon pisang kura-kura. Bau harum buah pisang
menggoda selera monyet. la lupa akan janjinya. Kura-kura menunggu di bawah pohon pisang.
"Nyet, Nyet, mana pisang bagianku?" teriak kura-kura.
"Sebiji pun tidak ada," jawab monyet rakus.
"Nyet, ini pohon pisangku!" rengek kura-kura hampir menangis.
"Salah sendiri mengapa tidak bisa memanjat pohon!" ejek monyet.
Raut wajah Kura-kura memerah, ia mulai menangis. Hatinya sedih bercampur marah. la lalu
menggoyang-goyang pohon pisang itu. Tiba-tiba .... bruk! Pohon pisang itu tumbang. Monyet itu
jatuh. Dia mengerang kesakitan. Tubuhnya tertimpa batang pohon pisang.
"Ampun kura-kura, tolong aku! Aku menyesal ...” kata monyet.
Tetapi, kura-kura sudah berlalu. la mencari sahabat baru.

6. Pernyataan yang sesuai dengan kutipan teks fabel tersebut adalah...


A. Monyet meminta Kura-kura menolong dirinya dan berjanji akan mengembalikan pisangnya.
B. Kura-kura yang merasa sedih itu meninggalkan Monyet yang masih berada di atas pohon.
C. Kura-kura bersedia membantu Monyet asalkan buah pisang hasil petiknya dibagi dua.
D. Monyet tidak memenuhi janji membagi pisang sehingga Kura-kura merasa sedih dan marah.

7. Makna simbol wajahnya merah pada teks fabel tersebut adalah...


A. marah B. terharu C. senang D. malu

8. Sebab terjadinya konflik pada teks fabel tersebut adalah...


A. Monyet membagi hasil panen pisang. B. Kura-kura tidak mau berbagi hasil.
C. Kura-kura tidak menepati janji. D. Monyet mengingkari janji.

9. Cermati peristiwa-peristiwa berikut!


(1) Akhirnya, semua penghuni sekolah mau bekerja sama dan mematuhi aturan yang dibuat oleh OSIS
SMP PGRI Kalimulya.
(2) Untuk mengurangi polusi di sekolah, OSIS SMP PGRI Kalimulya membuat peraturan bahwa
semua kendaraan guru, karyawan, dan tamu tidak boleh masuk lingkungan sekolah.
(3) Peraturan tersebut diberlakukan khusus hari ini mulai pukul 06.00 sampai dengan pukul 16.00.
(4) Dalam rangka memeringati Hari Bumi, OSIS SMP PGRI Kalimulya mengadakan berbagai
kegiatan yang bertema peduli lingkungan sekolah.
(5) Pada awalnya, banyak guru, karyawan, dan tamu memprotes aturan tersebut.
(6) Akan tetapi, pihak OSIS menjanjikan bahwa kendaraan yang ditempatkan di luar sekolah akan
dijamin keamanannya oleh satpam dan anggota OSIS.
Urutan peristiwa yang tepat adalah...
A. (4), (5), (6), (2), (3), (1) B. (4), (2), (3), (5), (6), (1)
C. (2), (3), (5), (6), (1), (4) D. (2), (5), (1), (3), (4), (6)

10. Bacalah teks berikut!


(1) ”Saya tidak suka kamu menasihati saya,” kata Nano sambil menunjuk jidat Kusno, “janganlah
merasa punya ilmu banyak!”
(2) “Bukan begitu, Nan, hidup ini harus saling mengisi, mengingatkanlah,” Kusno sedikit membela.
(3) “Boleh kamu mengingatkan dengan pengetahuanmu, tapi jangan lagi kepada saya!” kata Nano
ketus sambil ngeloyor entah ke mana.
(4) Kusno terdiam lama di bangku tunggu yang di teras kelurahan sambil mengelus-elus dahinya.
Bukti watak tokoh Nano seorang yang sombong terdapat pada bagian...
A. (1) dan (3) B. (2) dan (4) C. (3) dan (2) D. (4) dan (1)
11. Perhatikan kutipan teks drama berikut!
Ara : To, kamu dengar tidak, kalau kampung sebelah ada yang terkena virus Corona?
Anto : Ya, aku sempat dengar dari beberapa warga. Pak RT juga sudah sering sosialisasi tentang
waspada virus Corona.
Ara : Oh, yang cuci tangan pakai sabun dan pakai masker itu ya?
Anto : [....]
Ara : Wah, begitu yah? Itu sesuai juga dengan anjuran pemerintah ya?
Anto : Iya, Ra. Karena itu, sebaiknya kamu jangan dekat-dekat aku sekarang.
Ara : Hahaha, kamu bisa saja Ra.

Dialog yang tepat untuk melengkapi bagian rumpang kutipan teks tersebut adalah...
A. Ya, kamu harus pakai sabun yang bagus dan makser yang mahal agar aman.
B. Jangan lupa bawa disinfektan dan pakai setiap kamu pegang benda atau bersentuhan dengan orang
lain.
C. Bukan cuma itu, kita juga harus mengkonsumsi makanan sehat dan bergizi. Jangan lupa juga untuk
jaga jarak atau social distancing.
D. Selain itu, kamu juga jangan banyak berpergian. Lebih baik diam di rumah dan isolasi mandiri.

12. Cermati kutipan teks berikut!


[…. ]
Bandi : Din, aku minta jawaban soal nomor 8 dan 9!
Andin : A dan D!
Ita : Kalau soal nomor 10, 11, dan 15 jawabannya apa, Ban?
Banu : 10 A, 11 D, nomor 15 aku belum!
Adi : Huss, jangan kencang-kencang nanti pengawasya dengar!
Ita : Soalnya sulit sekali, masih banyak yang belum aku kerjakan.

Kalimat yang tepat untuk melengkapi kutipan teks drama tersebut adalah...
A. Ulangan sedang berlangsung. Suasana benar-benar hening. Para siswa tampak fokus pada soal
yang dihadapinya. Pak Rudin duduk sambil sesekali melempar pandangan ke luar Ruang 09.
B. Di Ruang 09, ulangan sedang berlangsung. Bebeberapa siswa tampak gelisah karena soal-soal yang
dihadapinya sangat sulit. Sementara itu, Pak Rudin tengah sibuk mengobrol.
C. Bebeberapa siswa di Ruang 09 tampak gelisah karena soal-soal yang dihadapinya sangat sulit.
Namun, ada hal yang melegakan. Pak Rudin pamit keluar sebentar karena ada sesuatu.
D. Di Ruang 09 ulangan sedang berlangsung. Suasana sangat tegang karena tatapan mata Pak Rudin
tak pernah lengah sedetik pun. Untunglah, soal-soal ulangan kali ini tidak terlalu sulit.

13. Bacalah penggalan teks berikut!


Denting jam tengah malam
[…]
Tuk bersujud
Kuagungkan Asma-Mu
Mohon ampunan-Mu
Larik bermajas yang tepat untuk melengkapi kutipan puisi tersebut adalah...
A. Bangunkan diriku
B. Terdengar nyaring
C. Kusebut nama-Mu
D. Pada pukul tiga tepat

14. Cermati beberapa teks berikut!


(1) Waspada Global Warming!
(2) Mengancam keselamatan planet bumi!
(3) Jika mencintai planet ini dan juga menyayangi anak cucu Anda.
(4) Bertindaklah sekarang juga!
Penjelasan kalimat nomor 4 dalam iklan di atas adalah...
A. Untuk menjaga alam adalah tanggung jawab pemerintah.
B. Mulai sekarang, kita harus ikut menjaga kelestarian alam.
C. Anak cucu kita akan ikut merasakan dampak global warming.
D. Cintailah planet ini agar keselamatan kita tidak terancam.

15. Cermati kedua kutipan teks berikut!


Teks 1 : Kementerian Pendidikan dan Kebudayaan (Kemendikbud) mengonfirmasi sekaligus
membenarkan kabar kepulangan 12 mahasiswa Indonesia dari Wuhan, Tiongkok ke Tanah Air.
Mereka dipulangkan dalam rangka evakuasi dari bahaya virus corona yang tengah mewabah di negara
tersebut.
Teks 2 : Kondisi Kota Wuhan seperti kota mati. Hal tersebut terjadi menyusul merebaknya virus
corona di kota itu. Rio mengatakan, seluruh transportasi umum seperti kereta listrik dan bus sudah
berhenti beroperasi. Otoritas setempat juga meminta penduduk tak keluar rumah jika tidak ada urusan
mendesak.
Perbedaan pola penyajian kedua kutipan teks berita tersebut adalah...
A. Teks 1 : diawali dengan di mana. Teks 2 : diawali dengan mengapa.
B. Teks 1 : diawali dengan di mana. Teks 2 : diawali dengan siapa.
C. Teks 1 : diawali dengan siapa. Teks 2 : diawali dengan bagaimana.
D. Teks 1 : diawali dengan siapa. Teks 2 : diawali dengan kapan.
16. Cermati kalimat berikut!
Para hadirin dipersilakan mencari tempat duduk sesuai dengan nomornya.
Perbaikan kalimat tersebut agar efektif adalah...
A. Hadirin silakan mencari tempat duduk sesuai nomor.
B. Hadirin dipersilakan duduk sesuai dengan nomornya.
C. Para hadirin dipersilakan mencari kursi sesuai nomornya.
D. Hadirin dipersilakan mencari tempat duduk dengan nomornya.

17. Cermati paragraf berikut!


(1) Letusan atau erupsi gunung Merapi sangat dasyat hingga berdampak pada masyarakat.
(2) Pada saat musim hujan, akan sering terlihat warna-warni bercahaya di ujung cakrawala.
(3) Banjir berpotensi mengakibatkan kerusakan pada berbagai aspek kehidupan dan properti.
(4) Gempa tektonik disebabkan oleh pergeseran lempeng-lempeng tektonik secara mendadak.
Kata bergaris bawah yang bersinonim terdapat pada kalimat bernomor...
A. (4) B. (3) C. (2) D. (1)

18. Cermati teks berikut!


Pada saat kita tidur, baik siang maupun malam hari, selalu saja ada makhluk kecil bersayap bernama
nyamuk. Hal ini mungkin hal yang sepele bagi sebagian orang dan dapat diatasi dengan
menyemprotkan cairan pembasmi nyamuk yang banyak beredar di pasar. Namun, tahukah Anda jika
cairan pembasmi tersebut mengandung berbagai bahan kimia yang dapat membahayakan kesehatan
kita? Untuk mengatasi hal tersebut, kini terdapat cara alternatif yang lebih aman untuk menangkap
dan memerangkap nyamuk. Cara membuatnya cukup sederhana dan bahan untuk membuatnya pun
dapat kita temukan dalam kehidupan sehari-hari.
Judul yang tepat untuk melengkapi kutipan teks laporan hasil percobaan di atas adalah ....
A. Cara Alternatif Membuat Nyamuk Terperangkap
B. Cara Membuat Perangkap Alternatif dan Aman
C. Cara Membuat Perangkap Nyamuk Sederhana
D. Cara Aman Membuat Cairan Pembasmi Nyamuk

19. Cermati kerangka teks berikut!


Bagian yang tepat untuk melengkapi kerangka pidato persuasif yang rumpang tersebut adalah ...
A. b) Tanggapan para orangtua tentang fenomena game online
d) Pro dan kontra di masyarakat tentang fenomena game online

B. b) Faktor penyebab maraknya game online pada remaja


d) Langkah mengantisipasi kecanduan game online pada remaja

C. b) Pendapat ahli tentang maraknya game online pada remaja


d) Tanggapan para remaja tentang maraknya game online

D. b) Pengertian game online menurut para ahli


d) Dampak positif game online

20. Cermati kutipan teks berikut!


Remaja Masjid Baiturahim mengadakan kegiatan gotong royong membersihkan parit, selokan, dan
sampah di sekitar lingkungan. Kegiatan ini bertujuan agar lingkungan sekitar tetap sehat dan bersih.
Slogan yang sesuai dengan ilustrasi tersebut adalah...
A. Hidup sehat dan bersih merupakan cita-cita warga bersama.
B. Gotong royong menjalin kerja sama sesama warga.
C. Mari bersama-sama bergotong royong di lingkungan sekitar.
D. Lingkungan bersih dan sehat tanggung jawab kita bersama.

21. Cermati ilustrasi berikut!


“Kami akan membawa lumba-lumba ini ke habitat aslinya.” kata Andi dan Supri.
Variasi kalimat yang tepat dari teks tersebut adalah...
A. Mereka berkata, “Andi dan Supri akan membawa lumba-lumba tersebut ke habitat aslinya.”
B. Andi dan Supri mengatakan bahwa mereka akan membawa lumba-lumba tersebut ke habitat
aslinya.
C. Andi dan Supri berkata, “Mereka akan membawa lumba-lumba tersebut ke habitat aslinya.”
D. Andi dan Supri berkata bahwa kami akan membawa lumba-lumba ini ke habitat aslinya.

22. Cermati kerangka surat pribadi berikut!


Surat untuk Kakak Pelangi
(1) dari Adikmu Tersayang, Dede Ayung
(2) Salam rindu, Kakak bagaimana kabarmu di sana?
(3) Wassalamualaikum wr. wb.
(4) Untuk Pelangi, Kakakku Tercinta, di Bandung
(5) Indramayu, 10 Maret 2020
(6) Isi tentang kerinduan adik pada kakak yang sedang kuliah di Bandung
(7) Akhir kata, moga Kakak selalu dalam lindungan-Nya.
Urutan kerangka untuk penulisan surat pribadi yang tepat adalah adalah...
A. (1), (7), (6), (4), (5), (2), (3)
B. (3), (2), (1), (4), (6), (5), (7)
C. (5), (4), (2), (6), (7), (3), (1)
D. (7), (2), (1), (3), (4), (6), (5)

Cermati teks diskusi berikut untuk menjawab soal nomor 23 s.d. 25 !

Dampak Bermain Game Online pada Anak

Istilah game online sudah tidak asing lagi bagi anak-anak dan remaja masa kini. Dengan daya jelajah
internet yang sangat luas dan tanpa batas, game online sudah bisa dinikmati kapan pun dan di mana
pun. Dampaknya pun kini mulai dirasakan banyak pihak. ( .... ) Akan tetapi, banyak juga pihak yang
berpendapat bahwa game online juga memiliki sisi positif. Dengan demikian, game online kini
menimbulkan pro-kontra di masyarakat.

23. Kalimat yang tepat untuk melengkapi bagian yang rumpang dalam paragraf tersebut adalah...
A. Terutama para orang tua yang memiliki anak remaja yang setiap hari gemar bermain game online.
B. Ada pihak yang beranggapan bahwa game online banyak menimbulkan dampak negatif.
C. Karena itu, persoalan ini tentunya memerlukan perhatian dan pengawasan dari para orang tua.
D. Anak-anak dan remaja pun kini bisa bermain game online melalui telepon genggam mereka.

24. Variasi judul yang sesuai dengan isi teks tersebut adalah...
A. Hubungan Game Online dan Anak di Masyarakat
B. Dampak Anak yang Bermain Game Online di Masyarakat
C. Pengaruh Bermain Game Online Bersama Anak
D. Pro dan Kontra Pengaruh Game Online pada Anak

25. Berdasarkan teks diskusi tersebut, pernyataan argumen kontra/ menentang yang paling tepat
adalah...
A. Bermain game online dapat menyebabkan kecanduan. Mereka yang ketagihan akan terus berusaha
mendapatkan skor terbanyak dan mencapai level tertinggi sehingga tidak akan meninggalkan game
tersebut. Jika ditinggalkan mereka akan terus memikirkan game itu, entah sampai kapan.

B. Game online bisa dijadikan aktivitas untuk refreshing. Anak yang baru pulang sekolah dapat
menghilangkan rasa jenuh setelah berjam-jam memikirkan pelajaran. Dengan game online, mereka
bisa melepaskan kepenatan sambil bermain game online.

C. Saat bermain game online, anak berlatih untuk bersabar dan tidak pantang menyerah. Karena, ada
beberapa jenis game yang bisa memancing emosi sang anak. Pada saat itulah anak diuji untuk dapat
mengendalikan emosinya, agar bisa memenangkan gamenya.

D. Bermain game online dianggap dapat meningkatkan kreativitas anak. Mereka akan berimajinasi
tentang hal-hal baru, mendapatkan ide menarik untuk menyelesaikan tugas dari gurunya di sekolah,
dan menciptakan sesuatu dari ide kreatifnya setelah bermain game online.
26. Cermati ilustrasi berikut!
Seorang siswa diminta membuat teks dengan judul Upaya Mengatasi Kecanduan Terhadap Game
Online pada Remaja.
Susunan kerangka isi teks yang paling tepat sesuai dengan judul tersebut adalah ....
A. 1) Latar belakang timbulnya kecanduan game online pada remaja
2) Tips mengantisipasi kecanduan game online pada remaja
3) Langkah mengatasi kecanduan game online pada remaja
4) Masalah yang timbul akibat kecanduan game online pada remaja

B. 1) Masalah yang timbul akibat kecanduan game online pada remaja


2) Langkah mengatasi kecanduan game online pada remaja
3) Tips mengantisipasi kecanduan game online bagi remaja
4) Pentingnya mengatasi masalah kecanduan game online pada remaja

C. 1) Fenomena maraknya kecanduan game online pada remaja


2) Masalah yang timbul akibat kecanduan game online pada remaja
3) Langkah mengatasi kecanduan game online pada remaja
4) Tips mengantisipasi kecanduan game online pada remaja

D. 1) Langkah mengatasi kecanduan game online pada remaja


2) Fenomena maraknya kecanduan game online pada remaja
3) Masalah yang timbul akibat kecanduan game online pada remaja
4) Tips mengantisipasi kecanduan game online pada remaja

27. Bacalah data-data berikut!


(1) Sungai Serayu bersih dan alami.
(2) Airnya jernih dan pemandangannya indah.
(3) Di pinggir sungai terdapat bebatuan
Teks laporan yang tepat berdasarkan data tersebut adalah...
A. Sungai Serayu merupakan salah satu objek wisata alam yang terdapat di Provinsi Jawa Tengah.
Sungai yang berhulu di dataran tinggi Dieng dan memiliki panjang sekitar 30 km ini melintasi lima
kabupaten, di Jawa Tengah.
B. Sungai Serayu tampak bersih dan alami. Sungai itu berair jernih. Pemandangan di sekitar sungai
pun tampak indah. Tidak terdapat sampah sedikit pun di sekitar sungai itu. Di pinggirnya, terdapat
bebatuan yang ditumpuk rata dan rapi. Orang-orang yang biasanya mancing di sekitar sungai itu
biasanya duduk-duduk di atas batu tersebut.
C. Sungai Serayu tampak bersih. Walaupun air sungai ini jernih dan tidak ada sampah sedikit pun,
tetapi tampak kurang indah. Di pinggirnya terdapat bebatuan yang ditumpuk ala kadarnya dan tidak
rapi. Orang-orang yang memancing di sekitar sungai itu biasanya duduk-duduk di atas batu tersebut.
D. Semua orang mengetahui kalau Sungai Serayu itu bersih dan alami. Sungai tersebut terlihat bersih
dan alami karena selalu dibersihkan. Oleh karena itu, setiap hari banyak sekali orang yang datang ke
sana untuk memancing.

28. Cermati teks berikut!


Amir, Ridho, Ali, dan Surya tergabung dalam regu piket hari Kamis. Surya selalu datang terlambat ke
sekolah sehingga jarang melaksanakan piket pagi. Saat teman-temannya sibuk membersihkan kelas,
Surya juga tidak mau membantu.
Berdasarkan ilustrasi di atas, ungkapan yang tepat untuk menunjukan sifat Surya adalah...
A. berat tangan
B. panjang tangan
C. ringan tangan
D. cuci tangan
29. Cermati kutipan teks pantun yang rumpang berikut!
(1) Melihat perahu memasang layar
(2) ...
(3) Siswa baru sedang rajin belajar
(4) ...
Larik yang tepat untuk melengkapi bagian teks pantun tersebut adalah...
A. (2) Pergi ke laut memancing ikan
(4) Tidak lupa membaca buku
B. (2) Cuaca terang tiada terkira
(4) Tetap semangat dan gembira
C. (2) Siap-siap juga membawa jaring
(4) Tetap di rumah secara daring
D. (2) Matahari terik memancar
(4) Baca cerita diam di kamar

30. Cermati teks berikut!

Daun salam memiliki berbagai khasiat. Hal ini merupakan berita menggembirakan bagi orang-orang
yang kurang suka dengan obat-obatan kimiawi. Ada berbagai manfaat daun salam, antara lain
meningkatkan kesehatan jantung, mencegah stroke, menurunkan tekanan darah, dan menurunkan
kolesterol. [ ... ] Ternyata daun salam juga dapat mencegah berbagai macam kanker.
Kalimat yang tepat untuk melengkapi paragraf tersebut adalah ...
A. Ada beberapa langkah mengolah daun salam untuk mengobatan berbagai penyakit.
B. Selain manfaat tersebut, masih ada satu manfaat yang belum diketahui banyak orang.
C. Penyakit yang bisa disembuhkan dengan daun salam adalah penyakit-penyakit ringan.
D. Para penderita cukup minum rebusan daun salam dalam takaran yang telah ditentukan.

31. Cermati teks berikut!

Wayang

Wayang adalah seni pertunjukan yang telah ditetapkan sebagai warisan budaya asli Indonesia.
UNESCO, lembaga yang membawahi kebudayaan dari PBB, pada 7 November 2003 menetapkan
wayang sebagai pertunjukan bayangan boneka tersohor dari Indonesia, sebuah warisan mahakarya
dunia yang tidak ternilai dalam seni bertutur.
Berdasarkan strukturnya, kutipan teks laporan tersebut terdapat pada bagian ....
A. definisi manfaat
B. definisi bagian
C. deskripsi bagian
D. deskripsi umum

32. Cermati teks berikut!


(1) Tataplah langit yang tersenyum
(2) Di tepi cahaya malam yang kian meredup
(3) Dengarkan pula detik-detik waktu di sudut sunyi
(4) Sebelum suasana kembali meriah
(5) Sementara daun-daun membisikan rindu yang kekal
Larik yang bermajas sama dalam puisi tersebut ditandai dengan nomor ....
A. (1) dan (3)
B. (1) dan (5)
C. (1) dan (2)
D. (1) dan (4)

33. Cermati kalimat berikut!


Kami membaca berulang-ulang cerpen “Ciwalung” dalam buku Antara Dua Dunia karya Firangha,
karena di dalamnya ada kisah kehidupan masyarakat terasing.
Penggunaan tanda baca pada kalimat tersebut tidak tepat karena ...
A. tanda petik (“...”) tidak digunakan pada penulisan judul cerpen.
B. tanda titik (.) tidak digunakan pada penulisan kalimat majemuk
C. tanda hubung (-) tidak digunakan jika yang diulang bukan kata dasar.
D. tanda koma (,) tidak digunakan jika induk kalimat mendahului anak kalimat.

34. Cermati kutipan teks berikut!


Teks 1 : Karimunjawa di Jepara berada di sebelah utara Pulau Jawa. Lokasi wisata bahari yang satu
ini sejak dulu sudah menjadi salah satu alternatif liburan yang sangat diinginkan oleh banyak orang.
Fasilitas di sana sangat lengkap. Bahkan, banyak juga backpacker yang berani untuk berwisata ke
sana dengan budget rendah.
Teks 2 : Kota Batu identik sebagai Kota Apel. Karena besarnya jumlah pengunjung untuk berwisata di
Kota Batu, potensi perkebunan apel juga dikembangkan sebagai objek wisata oleh masyarakat sekitar.
Saat ini, perkebunan apel yang tersebar di Kota Batu telah menjadi salah satu objek wisata favorit,
terutama bagi wisatawan lokal.
Persamaan isi kedua kutipan teks tersebut adalah ....
A. menjelaskan ketertarikan wisatawan luar negeri
B. membahas penelitian pengembang wisata
C. menggambarkan objek wisata domestik
D. menguraikan jenis wisata bahari di Jawa

35. Cermati kutipan cerita berikut!


Pisang raja sereh ditanam di pojok kanan belakang kebunnya. Sedangkan di sebelah kirinya ditanami
gayong.
Cabe rawit dan bayam petik berjajar di tengah kebun berselang-seling. Jalan dengan pembatas batu
memanjang di tengahnya. Pagar bambu belah setinggi betis membatasi tanaman dengan jalan tersebut.
“Wah … Bapak memang pekebun sejati,” celoteh Gamal mengikuti Pak Pandu yang dengan antusias
menerangkan perihal kebunnya.
“Ah, tidak. Saya hanya menanam lalu menyiramnya setiap saat. Itu saja,” kata Pak Pandu merendah.
Keunggulan kutipan cerita tersebut adalah...
A. penggambaran latar cerita sangat detail, rinci, dan jelas
B. kalimat-kalimatnya komunikatif dan sederhana
C. alur ceritanya runut, asyik, dan mudah diikuti
D. watak dan karakter tokoh yang sangat kuat

36. Cermati teks berikut!


Faktor cuaca adalah [ ... ] utama produksi garam nasional yang begitu minim selama setahun terakhir.
Hal lainnya yang menjadikan produksi garam di Indonesia relatif sedikit adalah [ ... ] garam yang
masih dilakukan secara tradisional.
Kata yang tepat untuk melengkapi teks tersebut adalah...
A. disebabkan, pembuat
B. menyebabkan, dibuatnya
C. penyebab, pembuatan
D. sebab, membuat
37. Cermati teks berikut!
Petunjuk Menulis Resensi Buku
1. Menuliskan pendapat pribadi sebagai tanggapan atas isi buku.
2. Meringkas atau mencatat bagian-bagian penting isi buku.
3. Memadukan rangkuman dan tanggapan pribadi ke dalam tulisan yang utuh.
4. Mendata identitas buku yang dibaca.
5. Mendaftar bagian-bagian yang merupakan kelebihan dan kekurangan buku.
Urutan petunjuk yang tepat adalah...
A. (4) – (2) – (5) – (1) – (3)
B. (3) – (1) – (4) – (2) – (5)
C. (2) – (1) – (4) – (3) – (5)
D. (5) – (2) – (4) – (3) – (1)

38. Cermati kutipan cerpen berikut!


Perasaannya hancur lebur setelah melihat sang kekasih meninggalkannya sendiri.
Penggunaan majas yang sama dengan kalimat di atas terdapat pada kalimat...
A. Malam tadi, si jago merah melalap habis pasar di ujung jalan.
B. Daun-daun berbisik menyampaikan kabar ayah yang sedang jauh di kota.
C. Wajahnya bersinar bagaikan rembulan malam yang sedang purnama
D. Ia berjuang mati-matian demi memenangi pertandingan final tersebut.

39. Cermati teks berikut!


Semua para siswa-siswa SMP Maju Perkasa diwajibkan mengenakan pakaian daerah pada perayaan
Hari Kartini di sekolah.
Alasan kesalahan yang terdapat pada bagian teks tersebut adalah...
A. penggunaan sapaan tidak sesuai
B. penggunaan kata tidak baku
C. penggunaan kata berlebihan
D. penggunaan kata dipengaruhi bahasa daerah

40. Cermati teks berikut!


Orang-orang film Indonesia memiliki potensial dalam membuat film. Para sineas muda di negeri ini
berpeluang besar membuat film-film alternatif yang cukup fenomena.
Perbaikan yang tepat terhadap kata bercetak miring pada teks tersebut adalah...
A. berfilm, berpotensi, fenomenil
B. perfilman, potensi, fenomenal
C. memfilmkan, potensikan, fenomenal
D. mengfilmkan, berpotensi, fenomena
41. Cermati teks berikut!
Widi melangkahkan kaki dengan tergesa. Ia sudah terlambat 10 menit dari jadwal hari ini, sehingga ia
tertinggal bus jemputan. Ia perlu keluar dari gerbang dan mencari ojek. Hari ini semakin sial, tidak
ada satupun ojek di pangkalan. Hari Senin seperti ini memang biasanya menjadi sangat sibuk, begitu
pun tukang ojek.

Ubahlah sudut pandang dalam kutipan cerpen tersebut menjadi sudut pandang orang pertama!

42. Cermati teks berikut!


Tanggul Sungai Cipanas di Kabupaten Indramayu, Jawa Barat, jebol, Sabtu (27/2/2021). Akibatnya,
ribuan rumah warga lima desa di Kecamatan Losarang, terendam. Lima desa terendam banjir itu
antara lain, Puntang, Krimun, Jangga, Muntur, dan Losarang. Ketinggian muka air (TMA) banjir rata-
rata 1 meter. Agar banjir tak semakin meluas, petugas Balai Besar Wilayah Sungai Cimanuk-
Cisanggarung dan Badan Penanggulangan Bencana Daerah (BPBD) Indramayu mengerahkan
excavator untuk memperbaiki tanggul yang jebol sepanjang 30 meter tersebut.

Tentukan unsur adiksimba (apa, di mana, kapan, siapa, mengapa, dan bagaimana) atau 5 WH sesuai
dengan kutipan teks berita tersebut!

43. Cermati teks cerita inspiratif berikut!

Kisah Tukang Kayu

Alkisah, seorang tukang kayu tua yang sudah bekerja puluhan tahun mulai merasa jika tubuhnya
sudah semakin tua untuk bekerja. Ia lalu memantapkan diri untuk berbicara dengan mandornya
tentang rencana untuk pensiun.

Mendengar rencana pensiun itu, sang mandor menjadi sedih. Sebab, tukang kayu yang hendak
pensiun itu merupakan tukang kayu terbaiknya. Mandor lalu mengizinkan tukang kayunya pensiun
setelah menyelesaikan satu proyek lagi.

Beberapa minggu kemudian, datang sebuah proyek baru untuk membuat rumah. Tukang kayu tua
yang hendak pensiun dipasrahi sebagai penanggung jawab, ia diberi kebebasan membuat bangunan itu
sesuka hatinya.

Setelah tahu proyek terakhirnya adalah sebuah rumah, tukang itu menjadi jengkel. Sebab, rumah
memakan waktu yang cukup lama untuk selesai. Ia pun bekerja malas-malasan dan membuat rumah
dengan asal-asalan. Bahan-bahan yang digunakan pun tidak diteliti kualitasnya.
Ketika rumah itu selesai, sang mandor mendatangi si tukang kayu tua dan berkata, “Rumah yang kau
bangun ini adalah hadiah dariku untukmu. Terima kasih telah bekerja dengan baik.”

Tulislah bagian koda atau pesan moral teks cerita inspiratif tersebut!

44. Cermatilah teks berikut!


sekali pun memiliki berbagai kekurangan buku karya ayu lestari ini tetap menarik dan sangat
bermanfaat sekali untuk dibaca oleh para remaja-remaja yang tertarik pada bidang penulisan cerita
populer.
Tulislah kembali tersebut sehingga menjadi kalimat yang efektif dengan memperhatikan penulisan
huruf dan tanda baca!

45. Cermatilah teks berikut!


Hari Minggu Banu sedang berbaring di kamarnya. Ibu masuk ke kamar menghampiri Banu.
“Banu! Bangun! Sudah siang, nanti kamu terlambat.” tegas ibunya.
“Bu, Banu masih mengantuk. Banu bolos sehari ya?” Banu memelas pada ibunya.
“Jangan begitu! Bayaran sekolahmu mahal, jangan menyepelekan menuntut ilmu,” jawab ibunya
menyanggah.
“Sehari saja, Bu. Banu mau tidur lagi.”
Melihat kelakuan Banu Ibunya geram. Akhirnya, ibunya mengajak Banu melihat kondisi anak-anak di
suatu panti asuhan.
“Nah, sekarang coba kamu buka mata kamu. Lihat! Mereka ingin sekolah sepertimu, namun tidak ada
orang tua yang membiayai mereka bersekolah,” jelas ibunya.
Dengan kejadian itu Banu tersadar dan mau berangkat sekolah walau terlambat. Di perjalanan menuju
sekolah, Banu melihat seorang anak yang kondisi tubuhnya tidak sempurna. Anak itu berseragam
sekolah yang sama dengannya. Dalam hati Banu berkata, “Aku bersyukur masih punya fisik yang
sempurna untuk bisa menuntut ilmu”.

Tulislah teks dialog drama singkat berdasarkan teks cerpen di atas dengan memperhatikan kesesuaian
isi, hubungan antardialog, ejaan, dan tanda baca!

I. PILIHAN GANDA
1D 11 C 21 B 31 D
2C 12 B 22 C 32 B
3A 13 A 23 B 33 D
4A 14 B 24 D 34 C
5D 15 C 25 A 35 A
6D 16 A 26 C 36 C
7A 17 A 27 B 37 A
8D 18 C 28 A 38 D
9B 19 B 29 C 39 C
10 A 20 D 30 B 40 B

II. URAIAN

41. Alternatif jawaban Kulangkahkan kaki dengan tergesa. Aku sudah terlambat 10 menit
dari jadwal hari ini, sehingga membuatku tertinggal bus jemputan. Aku perlu keluar dari
gerbang dan mencari ojek. Hari ini semakin sial, tidak ada satupun ojek di pangkalan.
Hari Senin seperti ini memang biasanya menjadi sangat sibuk, begitu pun tukang ojek.
42. Alternatif jawaban
apa : ribuan rumah warga lima desa di kecamatan Losarang terendam banjir
di mana : di sekitar sungai Cipanas, kecamatan Losarang, Kabupaten Indramayu
kapan : Sabtu, 27 Februari 2021
siapa : warga lima desa di kecamatan Losarang (Puntang, Krimun, Jangga, Muntur,
Losarang)
mengapa : tanggul sungai Cipanas Jebol sepanjang 30 meter
bagaimana : agar banjir tak semakin meluas, petugas Balai Besar Wilayah Sungai
Cimanuk-Cisanggarung dan Badan Penanggulangan Bencana Daerah (BPBD) Indramayu
mengerahkan excavator untuk memperbaiki tanggul yang jebol sepanjang 30 meter
tersebut.

43. Alternatif jawaban


Dalam melaksanakan tugas dan kewajiban, kita sebaiknya bersungguh-sungguh dan
melakukan yang terbaik apapun alasannya. Janganlah bermalas-malasan dalam
melaksanakan pekerjaan yang sudah menjadi tugas dan tanggung jawab kita karena hasil
yang kita peroleh pada akhirnya sesuai dengan apa yang telah kita lakukan.

44. Alternatif jawaban


Sekalipun memiliki berbagai kekurangan, buku karya Ayu Lestari ini tetap menarik dan
sangat bermanfaat untuk dibaca oleh para remaja yang tertarik pada bidang penulisan
cerita populer.

45. Alternatif jawaban


Hari Minggu Banu sedang berbaring di kamarnya. Ibu masuk ke kamar menghampiri
Banu. Ibu : “Banu! Bangun! Sudah siang, nanti kamu terlambat.”
Banu : “Bu, Banu masih mengantuk. Banu bolos sehari ya?” (Banu memelas pada
ibunya.) Ibu : “Jangan begitu! Bayaran sekolahmu mahal, jangan menyepelekan menuntut
ilmu,” (jawab ibunya menyanggah.)
Banu : “Sehari saja, Bu. Banu mau tidur lagi.” Melihat kelakuan Banu Ibunya geram.
Akhirnya, ibunya mengajak Banu melihat kondisi anak-anak di suatu panti asuhan.
Ibu : “Nah, sekarang coba kamu buka mata kamu. Lihat! Mereka ingin sekolah sepertimu,
namun tidak ada orang tua yang membiayai mereka bersekolah,”
Banu : “Oh, iya ya, Bu. Baiklah, Banu sadar sekarang. Banu mau berangkat sekolah
walau terlambat.”
Di perjalanan menuju sekolah, Banu melihat seorang anak yang kondisi tubuhnya tidak
sempurna. Anak itu berseragam sekolah yang sama dengannya. Dalam hati Banu berkata,
“Aku bersyukur masih punya fisik yang sempurna untuk bisa menuntut ilmu”

Soal Latihan 2
1. Perhatikan susunan tak beraturan dari kalimat-kalimat di bawah ini :
c. Santi mengawali harinya dengan bangun pukul 04.00 wib untuk bersiap-siap shalat subuh.
d. Setelah itu ia mulai membantu ibunya di dapur.
a. Setelah menyelesaikan urusan dapur, ia mandi dan bersiap-siap menuju ke sekolah.
b. Sesampainya di sekolah, Santi belajar bersama rekan-rekan sekelasnya.

Susunan yang tepat pada kalimat-kalimat di atas sehingga menjadi satu paragraf yang padu
adalah …
a. a-b-c-d
b. b-c-d-a
c. c-d-a-b
d. a-d-c-b
e. c-d-a-b

1. Penulisan kata bentukan yang tidak tepat terdapat pada kalimat…

A. Pada masa ORBA, Indonesia terkenal dengan swa sembada pangannya.

B. Pesawat itu telah lepas landas.

C. Amerika Serikat adalah negara adikuasa.

D. Zaman prasejarah memberi inspirasi pada kehidupan sekarang.

2. Kalimat yang menggunakan kata tidak baku adalah…

a. Indonesia terus bekerja keras untuk memajukan teknologinya.

b. Penulis sudah mengkaji hasil penelitiannya dengan seksama.

c. Indonesia belum juga menemukan langkah konkret untuk mengatasi masalah


pengangguran.

d. Untuk menjaga kedaulatannya, Indonesia menambah kuantitas Armada di Kawasan


Ambalat.

3. Tubuh biru
Tatapan mata biru Lelaki terguling di jalan
Lewat gardu belanda dengan bumi
Berlindung warna malam Sendiri masuk kota
Ingin ikut ngubur ibunya

Tema puisi tersebut adalah...

a. Perlawanan
b. Keberanian
c. Kekejaman
d. Perjuangan

4. Sebagian produk-produk industri dalam negeri khususnya dalam barang kebutuhan sehari-
hari kurang laku dipasaran, masyarakat lebih menyukai produk luar negeri karena kualitasnya
lebih baik. Tanggapan logis terhadap pernyataan di atas, yaitu...

A. Jumlah barang yang dijual dikurangi karena barang karena barang tersebut tidak laku di
pasaran

B. Barang-barang kebutuhan sehari-hari perlu diproduksi karena diperlukan.

C. Kualitas barang harus ditingkatkan karena telah ditingkatkan karena konsumen lebih
senang barang yang bermutu

D. Produksi dalam negeri dijual murah karena dengan harga murah barang cepat habis.

5. (1) Sebagai seorang sastrawan, pekerjaan Djenar Maesa Ayu menulis novel atau cerita
pendek.

(2) Dari karya tersebut tentu pembaca memperoleh berbagai manfaat.

(3). Penyajian pesan penulis sastra halus dan sopan.

(4). Hasil penulisannya jarang menimbulkan hal-hal yang berbau SARA.

Kalimat fakta dalam paragraf tersebut ditandai dengan no….

a. (1)

b. (2)

c. (3)

d. (4)

6. Terdapat banyak hal yang bisa dinikmati dari pemakaian media sosial. Pada teks berikut
akan dibahas mengenai beragam hal seputar pemakaian media sosial.

Bagian kutipan tersebut adalah...

A. Isi
B. Pembukaan
C. Penutup
D. Solusi

7. Pernyataan yang berisi kecaman dan sering diuraikan dengan pertimbangan baik buruknya
suatu karya, kebijakan, pendapat, maupun penampilan seseorang disebut dengan ….
A. Kritik
B. Sanggahan
C. Pujian
D. Penolakan

8. Pernyataan yang berisi ungkapan ketidak-setujuan dalam suatu permasalahan maupun


pembicaraan ialah ….

A. Kritik
B. Penolakan
C. Sanggahan
D. Pujian

9. Baca teks di bawah ini!


Dari permasalahan tersebut, ternyata bukan hanya guru saja yang bertanggung jawab atas
ketidakberlangsungan materi sastra di sekolah. Akan tetapi, perlu ditelaah lebih jauh tentang
kurikulum dari pemerintah dimana pembelajaran sastra hanya ditempelkan pada pelajaran
bahasa Indonesia. Sehingga, minat siswa pada sastra menjadi sangat lah rendah.
Teks di atas merupakan salah satu struktur teks tanggapan kritis …

A. Kesimpulan
B. Penegasan ulang
C. Deskripsi teks
D. Evaluasi

10. Di bawah ini yang merupakan ciri-ciri teks tanggapan kritis, kecuali ….

A. Berisi pengalaman hidup seseorang


B. Memakai proses penilaian subjektif dan objektif
C. Menghadirkan opini penulis dengan cara universal
D. Menunjukkan fakta serta data kualitatif maupun kuantitatif

11. Jenis diskusi yang dilakukan sekelompok orang yang membahas tentang topik yang
sedang jadi perhatian umum di depan pendengar radio atau penonton TV disebut dengan...

A. Diskusi kelompok

B. Diskusi panel

C. Rapat

D. Dialog interaktif

12. Di kelasmu akan diadakan diskusi mengenai "Bahaya Narkotika". Kalimat pendahuluan
yang tepat untuk memulai diskusi bagi moderator yaitu...

A. "Bahaya Narkotika" adalah tema yang akan kita diskusikan hari ini

B. Teman-teman, meski banyak yang belum hadir, kita mulai saja diskusinya
C. Mari kita mulai kegiatan diskusi hari ini mengenai "Bahaya Narkotika"

D. Sebaiknya diskusi dibuka setelah kepala sekolah hadir

13. Kalimat yang sesuai untuk menutup kegiatan diskusi adalah...

A. Sekian diskusi hari ini, saya ucapkan terima kasih

B. Demikian diskusi kita kali ini, atas perhatian Anda kami mengucapkan terima kasih

C. Demikian diskusi kita, terima kasih yang sebesar-besarnya atas partisipasi Anda

D. Sekian diskusi kita, semoga bermanfaat.

Baca Juga: 40 Contoh Soal Bilangan Bulat Positif dan Negatif Kelas 6 serta Jawaban

14. Rangkuman merupakan catatan isi pembicaraan selama berdiskusi. Berikut merupakan
pernyataan yang tepat mengenai rangkuman, kecuali...

A. Berisi berita terbaru dan terpercaya

B. Berisi ulasan pembicara mengenai tanggapan peserta

C. Berisi inti permasalahan yang dibahas

D. Berisi semua pembicaraan selama kegiatan diskusi

15. Tujuan teks diskusi adalah...

A. Mencari kesepakatan atau kesepahaman pendapat

B. Memberi pelajaran, gambaran serta kisah yang inspiratif

C. Menyelesaikan perbedaan

D. Menggambarkan objek

16. Hal yang tidak boleh dilakukan saat mengkritik yaitu...

A. Menawarkan solusi

B. Memakai bahasa yang berkonotasi negatif

C. Tidak menjatuhkan nama baik

D. Menunjukkan kekurangan karya seni

17. Yang merupakan salah satu ciri dari teks tanggapan yaitu...

A. Membuat pembaca merasakan (mengalami) sendiri


B. Menggambarkan sesuatu yang melibatkan pancaindera

C. Menjelaskan ciri objek seperti ukuran, warna, keadaan, dan bentuk

D. Mempunyai tiga struktur yakni konteks, deskripsi, serta penilaian

18. Pada sebuah teks tanggapan kritis, ada informasi mengenai alasan yang mendukung
pernyataan maupun menolak pernyataan, biasanya ada di bagian tengah teks adalah bagian
….

A. Evaluasi

B. Insiden

C. Orientasi

D. Deskripsi teks

Teks berikut untuk menjawab soal nomor 19 dan 20

Alasan keluasan negara, keefisienan, keamanan, serta kebanggaan adalah hal yang lebih
utama. Sehingga, pembelian pesawat kepresidenan sangat relevan untuk keamanan,
kenyamanan, kebutuhan mobilitas, serta efektivitas kegiatan presiden.

19. Struktur teks tanggapan kritis yang sesuai dengan paragraf di atas adalah ….

A. Abstrak

B. Kesimpulan

C. Tafsiran

D. Evaluasi

20. Kata "mobilitas" pada paragraf di atas mempunyai makna ….

A. Seperti gerakan mobil

B. Bergerak sendiri

C. Gerakan mempercepat

D. Gerakan berpindah-pindah

Perhatikan dan Baca Teks di Bawah Ini untuk Menjawab Pertanyaan Nomor 21-24

Pada era globalisasi ini, game online semakin marak di kalangan masyarakat. Peminatnya
berasal dari berbagai kalangan. Mulai dari anak-anak, remaja, dan bahkan hingga dewasa,
terutama pelajar. Kini, game online semakin mudah ditemukan dan banyak macamnya.
Sebagian besar masyarakat menggemarinya. Apalagi, kini banyak warnet yang menyediakan
game online dengan harga yang cukup murah. Membuat peminat game online terus
meningkat hingga sekarang.

Sebagian besar masyarakat menganggap bahwa game online memberikan dampak negatif
bagi para pemainnya. Namun, ada juga yang menganggap bahwa bermain game online akan
berdampak positif. Game online merupakan suatu aktivitas untuk refreshing. Dengan game
online anak yang baru pulang dari sekolah dapat menghilangkan rasa jenuh setelah berjam-
jam memikirkan pelajaran. Mereka bisa melepaskan kelelahan sambil bermain game online,
yang digemarinya. Selain itu, bermain game online dapat meningkatkan kreativitas sang
anak.

21. Paragraf pertama kutipan teks tersebut disebut bagian....

A. isu masalah

B. argumen

C. isi

D. simpulan

22. Topik masalah dalam kutipan teks tersebut adalah....

A. penggemar game online

B. maraknya game online

C. manfaat game online

D. dampak game online

23. Makna kata kreativitas pada paragraf kedua adalah....

A ketangkasan

B. kecerdasan

C. daya cipta

D. akal manusia

24. Pernyataan yang sesuai dengan isi paragraf kedua adalah....

A. Bisnis game online sangat menguntungkan masyarakat.

B. Game online meningkatkan kreativitas anak.

C. Game online membuat anak menjadi pemalas.

D. Game online merupakan dampak globalisasi.


25. Perhatikan susunan tak berarturan dari kalimat-kalimat di bawah ini :

C. Santi mengawali harinya dengan bangun pukul 04.00 wib untuk bersiap-siap shalat subuh.

D. Setelah itu ia mulai membantu ibunya di dapur.

A. Setelah menyelesaikan urusan dapur, ia mandi dan bersiap-siap menuju ke sekolah.

B. Sesampainya di sekolah, Santi belajar bersama rekan-rekan sekelasnya.

Susunan yang tepat pada kalimat-kalimat di atas sehingga menjadi satu paragraf yang padu
adalah...

A. a-b-c-d

B. b-c-d-a

C. c-d-a-b

D. a-d-c-b

E. c-d-a-b

26. Bacalah kutipan berikut kemudian pilihlah jawaban yang tepat!

"Internet memberikan dampak negatif yang tidak sedikit bagi pelajar."

Internet menyediakan situs yang isinya tidak pantas untuk dilihat oleh pelajar terutama
pelajar SD dan SMP Informasi pada kutipan tersebut termasuk...

A. Sanggahan

B. Kritik

C. Pujian

D. Evaluasi

27. Perhatikan susunan tak beraturan dari kalimat-kalimat di bawah ini:

C. Santi mengawali harinya dengan bangun pukul 04.00 wib untuk bersiap-siap shalat subuh.
D. Setelah itu ia mulai membantu ibunya di dapur.
A. Setelah menyelesaikan urusan dapur, ia mandi dan bersiap-siap menuju ke sekolah.
B. Sesampainya di sekolah, Santi belajar bersama rekan-rekan sekelasnya.

Susunan yang tepat pada kalimat-kalimat di atas sehingga menjadi satu paragraf yang padu
adalah ...

A. A-B-C-D
B. B-C-D-A

C. C-D-A-B

D. A-D-C-B

E. C-D-A-B

Bacalah kutipan berikut kemudian pilihlah jawaban yang tepat!

Alasan yang pertama, kita pasti sudah mengetahui jika lulusan SMK lebih mudah dan lebih
siap untuk bekerja jika dibandingkan dengan lulusan SMA. Karena di SMK siswa tidak
hanya diajari teoretik yang mendalam sesuai jurusannya saja, melainkan di sana siswa juga
dibekali dengan kemampuan praktik yang sangat menambah pengalaman dan memberikan
gambaran nyata tentang dunia kerja. Sebagaimana yang sering kita dengar atau lihat, siswa
SMK sering melaksanakan PKL (Praktik Kerja Lapangan) atau yang biasa disebut dengan
istilah “magang”. Saya sepakat dengan hal ini karena pada SMA, siswa lebih banyak belajar
teori secara mendalam, sehingga lulusan SMA harus berkuliah dulu jika ingin mendapatkan
pekerjaan.

28. Kutipan tersebut merupakan bagian teks tanggapan ….

A. Evaluasi

B. Deskripsi

C. Penegasan ulang

D. Kesimpulan

29. Bacalah kutipan berikut kemudian pilihlah jawaban yang tepat!

Internet memberikan dampak negatif yang tidak sedikit bagi pelajar.

Internet menyediakan situs yang isinya tidak pantas untuk dilihat oleh pelajar terutama
pelajar SD dan SMP Informasi pada kutipan tersebut termasuk….

A. Sanggahan

B. Kritik

C. Pujian

D. Evaluasi

Kutipan teks diskusi berikut untuk nomor 30-33

Sebagian besar peminat game online berasal dari kalangan pelajar. Bagi kalangan pelajar,
game online memiliki beberapa dampak salah satunya menyebabkan kecanduan. Jika sudah
ketagihan, mereka akan berusaha mendapatkan skor terbanyak dan mencapai level tertinggi.
Dampak lain adalah lupa waktu untuk melakukan aktivitas lain, seperti belajar, makan,
mandi, dan beribadah.

Perkembangan game online memiliki dampak positif dan negatif. Dampak yang dirasakan ini
tergantung dari pemainnya. Orang tua disarankan utuk melakukan tindakan yang dapat
mencegah dampak negatif dari internet, khususnya game online. Para orang tua harus bekerja
sama dengan guru untuk turut memantau perkembangan anaknya ketika di sekolah.

30. Konjungsi yang memiliki makna 'pengandaian' pada kutipan tersebut adalah....

A. Jika

B. Dan

C. Dengan

D. Ketika

31. Paragraf pertama kutipan teks tersebut memiliki ide pokok....

A. peminat game online

B. dampak negatif game online

C. perkembangan game online

D. game online sebabkan kecanduan

32. Isi paragraf kedua pada kutipan teks diskusi tersebut disebut bagian .....

A. isu masalah

B. argumen mendukung

C. argumen menentang

D. Simpulan

33. Pendapat yang sesuai dengan isi paragraf kedua adalah....

A. Sebaiknya, orang tua membatasi anak bermain game online.

B. Alangkah baiknya, orang tua membatasi jumlah uang saku anak.

C. Seharusnya, guru memberikan sanksi bagi anak yang bermain game online.

D. Sebaiknya, orang tua melarang anak keluar rumah setelah pulang sekolah.

34. Dalam acara pelepasan siswa SMP kelas XI, Diva berpidato mewakili teman-teman kelas
VII dan VIII. Isi penggalan pidato Diva yang sesuai adalah….
A. Berjuanglah terus, saya doakan semoga, sukses bahagia selalu, dan panjang umur.

B. Kalian akan menghadapi suasana baru, oleh karena itu janganlah bersikap seperti anak-
anak tetapi bersikaplah dewasa.

C. Selamat jalan kami ucapkan jangan lupa dibalik keberhasilan ini kalian masih punya
tantangan berat,yaitu memasuki jenjang sekolah yang baru.

D. Pertahankan prestasi kalian yang telah dicapai, jika dapat lebih tingkatkanlah.

35. Di kelasmu akan diadakan diskusi mengenai “Bahaya Narkotika”. Kalimat pendahuluan
yang tepat untuk memulai diskusi bagi moderator yaitu ….

A. “Bahaya Narkotika” adalah tema yang akan kita diskusikan hari ini.

B. Teman-teman, meski banyak yang belum hadir, kita mulai saja diskusinya

C. Mari kita mulai kegiatan diskusi hari ini mengenai “Bahaya narkotika”

D. Sebaiknya diskusi dibuka setelah kepala sekolah hadir

Kutipan teks diskusi berikut untuk soal nomor 36-40

Pada era globalisasi ini, game online semakin marak di kalangan masyarakat. Peminatnya
berasal dari berbagai kalangan. Mulai dari anak-anak, remaja, dan bahkan hingga dewasa,
terutama pelajar. Kini, game online semakin mudah ditemukan dan banyak macamnya.
Sebagian besar masyarakat menggemarinya. Apalagi, kini banyak warnet yang menyediakan
game online dengan harga yang cukup murah. Membuat peminat game online terus
meningkat hingga sekarang.

Sebagian besar masyarakat menganggap bahwa game online memberikan dampak negatif
bagi para pemainnya. Namun, ada juga yang menganggap bahwa bermain game online akan
berdampak positif. Game online merupakan suatu aktivitas untuk refreshing. Dengan game
online anak yang baru pulang dari sekolah dapat menghilangkan rasa jenuh setelah berjam-
jam memikirkan pelajaran. Mereka bisa melepaskan kelelahan sambil bermain game online,
yang digemarinya. Selain itu, bermain game online dapat meningkatkan kreativitas sang
anak.

36.Paragraf pertama kutipan teks tersebut disebut bagian...

A. Isu masalah

B. Argumen

C. Isi

D. Simpulan

37. Topik masalah dalam kutipan teks tersebut adalah....


A. Penggemar game online

B. Maraknya game online

C. Manfaat game online

D. Dampak game online

38. Makna kata kreativitas pada paragraf kedua adalah....

A. Ketangkasan

B. Kecerdasan

C. Daya cipta

D. Akal manusia

39. Pernyataan yang sesuai dengan isi paragraf kedua adalah....

A. Bisnis game online sangat menguntungkan masyarakat.

B. Game online meningkatkan kreativitas anak.

C. Game online membuat anak menjadi pemalas.

D. Game online merupakan dampak globalisasi.

40. Kalimat yang sesuai untuk menutup kegiatan diskusi adalah ….

A. Sekian diskusi hari ini, saya ucapkan terima kasih

B. Demikian diskusi kita kali ini, atas perhatian Anda kami mengucapkan terima kasih

C. Demikian diskusi kita, terima kasih yang sebesar-besarnya atas partisipasi Anda.

D. Sekian diskusi kita, semoga bermanfaat.

Kunci Jawaban

1. A
2. B
3. D
4. C
5. A
6. B
7. A
8. C
9. B
10. A
11. D
12. C
13. B
14. A
15. A
16. B
17. D
18. D
19. B
20. D
21.A
22. B
23. C
24. B
25. D
26. B
27. D
28. A
29. B
30. A
31. B
32. D
33. B
34. C
35. C
36. A
37. B
38. C
39. B
40. B

1. Bacalah kutipan berikut kemudian pilihlah jawaban yang tepat!

Alasan yang pertama, kita pasti sudah mengetahui jika lulusan SMK lebih mudah dan lebih
siap untuk bekerja jika dibandingkan dengan lulusan SMA. Karena di SMK siswa tidak
hanya diajari teoretik yang mendalam sesuai jurusannya saja, melainkan di sana siswa juga
dibekali dengan kemampuan praktik yang sangat menambah pengalaman dan memberikan
gambaran nyata tentang dunia kerja. Sebagaimana yang sering kita dengar atau lihat, siswa
SMK sering melaksanakan PKL (Praktik Kerja Lapangan) atau yang biasa disebut dengan
istilah "magang". Saya sepakat dengan hal ini karena pada SMA, siswa lebih banyak belajar
teori secara mendalam, sehingga lulusan SMA harus berkuliah dulu jika ingin mendapatkan
pekerjaan.

Kutipan tersebut merupakan bagian teks tanggapan ....

evaluasi
deskripsi
penegasan ulang
kesimpulan
Jawaban: A
Pembahasan: Deret penjelas terdapat pada kata kunci: jika...sehingga, dan berisi pernyataan
umum yang berhubungan dengan permasalahan yang telah disampaikan penulis.

2. Bacalah kutipan berikut kemudian pilihlah jawaban yang tepat!

Internet memberikan dampak negatif yang tidak sedikit bagi pelajar. Internet menyediakan
situs yang isinya tidak pantas untuk dilihat oleh pelajar terutama pelajar SD dan SMP

Informasi pada kutipan tersebut termasuk...

sanggahan
kritik
pujian
evaluasi

Jawaban: B
Pembahasan: Merupakan ungkapan ketidaksetujuan terhadap suatu hal yang dilengkapi
alasan membangun bukan alasan menjatuhkan.

3. Bacalah kutipan berikut kemudian pilihlah jawaban yang tepat!

Pencemaran Lingkungan

Pencemaran udara umumnya dihasilkan oleh pembakaran bahan bakar fosil yang tidak
sempurna, seperti pembakaran batu bara, kayu, minyak, dan gasolin. Polutan gas yang masuk
ke udara berasal dari aktivitas manusia atau terjadi secara alami. Polutan gas ini mengandung
karbon dioksida, karbon monoksida, timah, nitrogen oksida, dan sulfur dioksida.

Saat ini, jumlah karbon dioksida (CO2) yang dilepaskan ke udara terus mengalami
peningkatan sehingga terjadilah efek rumah kaca atau kenaikan suhu di bumi. Efek rumah
kaca ini menjadi masalah darurat yang dapat mengancam kehidupan manusia di bumi.
Peningkatan suhu di bumi menyebabkan salju di daerah kutub mencair sehingga permukaan
air laut meningkat. Hal tersebut menjadi salah satu faktor yang memicu semakin sering
terjadinya banjir di bumi.

Karbon monoksida membuat kemampuan darah untuk membawa oksigen ke jaringan tubuh
berkurang. Karbon monoksida ini dihasilkan oleh asap motor dan mobil. Sulfur dioksida
(SO2) yang meningkat di atmosfer menyebabkan gangguan kesehatan pada manusia,
terutama radang paru-paru, penyakit bronkitis, dan gagal jantung. Selain itu, SO2 juga
mampu merusak semua vegetasi hingga jarak yang jauh. SO2 merupakan komponen utama
yang menyebabkan terjadinya hujan asam. Hujan asam dapat menyebabkan korosi pada
bangunan dan kerusakan hutan.

Isi paragraf ke-1 bacaan tersebut adalah...


Polutan gas yang masuk ke udara berasal dari aktivitas manusia
Bahan bakar fosil yang tidak sempurna, seperti pembakaran batu bara, kayu, minyak, dan
gasolin.
Polutan gas ini mengandung karbon dioksida, karbon monoksida, dan sulfur dioksida.
Pencemaran udara umumnya dihasilkan oleh pembakaran bahan bakar fosil yang tidak
sempurna

Jawaban: D
Pembahasan: Ide pokok dari paragraf 1

4. (i) Jumlah karbon dioksida (CO2) yang dilepaskan ke udara terus mengalami peningkatan
sehingga terjadilah efek rumah kaca atau kenaikan suhu di bumi.
(ii) Efek rumah kaca ini tidak menjadi masalah yang dapat mengancam kehidupan manusia di
bumi.
(iii) Peningkatan suhu di bumi menyebabkan salju di daerah kutub mencair sehingga
permukaan air laut meningkat
(iv) Karena air laut meningkat, menjadi salah satu faktor yang memicu semakin sering
terjadinya banjir di bumi.
(v) Pencemaran udara umumnya dihasilkan oleh pembakaran bahan bakar fosil yang
sempurna.

Dari pilihan yang ada, manakah pandangan penulis tentang teks tanggapan yang berjudul
"Pencemaran Lingkungan"?

(iv) dan (iv)


(i), (ii), dan (iii)
(i), (iii), dan (iv)
(ii), (iii), dan (v)

Jawaban: C

Bacalah teks cerita inspiratif berikut untuk mengerjakan nomor 5 - 7!

Suatu ketika Dr. Mark, seorang dokter spesialis kanker yang terkemuka menerima undangan
untuk menghadiri sebuah konferensi, sekaligus untuk menerima penghargaan riset bidang
medis yang telah dipublikasikannya. Ia sangat bersemangat dan ingin secepatnya tiba di sana.
Ia telah bekerja keras dalam waktu yang lama untuk risetnya itu dan merasa ia layak
menerima penghargaan tersebut.

Akan tetapi, dua jam setelah lepas landas, pesawat yang ditumpanginya itu harus mendarat
darurat di bandara terdekat akibat kerusakan teknis. Karena kuatir akan terlambat tiba di
konferensi tersebut, Dr. Mark segera pergi ke bagian resepsionis dan mendapat informasi
bahwa penerbangan berikutnya baru ada sepuluh jam kemudian. Resepsionis menyarankan
agar ia menyewa mobil dan berkendara ke kota tempat konferensi itu, hanya empat jam saja
berkendara ke tujuannya. Karena tidak ada pilihan lain, ia memutuskan menggunakan mobil
meskipun ia tidak suka mengemudi untuk perjalanan jauh. Dr. Mark menyewa mobil dan
memulai perjalanannya. Akan tetapi, segera setelah ia berangkat, tiba-tiba cuaca menjadi
buruk dan sebuah badai besar terjadi. Hujan yang terus turun menyulitkannya untuk melihat
jalan dan tandatanda arah lokasi sehingga akhirnya ia melewati belokan yang seharusnya ia
ambil.

Setelah dua jam lamanya berkendara, ia sadar bahwa ia telah tersesat. Mengemudi dalam
hujan yang lebat di jalan yang terpencil membuatnya merasa lapar dan letih. Dengan rasa
gelisah ia mulai mencari tanda-tanda rumah penduduk yang ada di sekitar lokasi itu. Tidak
lama kemudian ia menjumpai sebuah rumah tua kecil. Lalu ia keluar dari mobil dan
mengetuk pintu. Seorang wanita sederhana membuka pintu. Dr. Mark menjelaskan
masalahnya dan meminta tolong kepada wanita tersebut apakah ia bisa meminjamkan
teleponnya.

Tetapi wanita itu memberitahukan bahwa ia tidak memiliki telepon atau pun peralatan
telekomunikasi lainnya. Wanita itu mengajak dokter Mark untuk masuk ke rumahnya dan
menunggu hingga cuaca membaik. Dalam kondisi lapar, basah dan kelelahan membuat sang
dokter menerima tawaran baik dari wanita itu dan masuk ke dalam rumah. Wanita itu
memberinya teh panas dan roti kering untuk dimakan. Wanita itu mengajaknya untuk berdoa
bersama. Sambil tersenyum Dr. Mark berkata bahwa ia hanya percaya akan kerja keras. Ia
lalu mempersilakan wanita itu untuk melanjutkan doanya.

Sambil duduk menikmati tehnya, Mark memperhatikan wanita itu berdoa di keredupan
cahaya lilin. Ia berdoa di samping sesuatu yang tampak seperti tempat tidur bayi kecil. Setiap
saat wanita itu selesai berdoa ia segera melanjutkannya dengan doa lainnya. Merasa bahwa
wanita itu sedang membutuhkan pertolongan, sang dokter mengambil kesempatan untuk
berbicara segera setelah wanita itu selesai berdoa. Ia bertanya kepada wanita itu apakah
sebenarnya yang diinginkannya dan apakah Tuhan akan mendengar doa-doanya. Lalu sang
dokter bertanya tentang anak yang ada di tempat tidur bayi tersebut yang sepertinya sedang ia
doakan. Dengan tersenyum getir wanita itu berkata bahwa anak itu adalah anaknya. Ia sedang
menderita suatu jenis penyakit kanker yang jarang ditemukan dan hanya ada satu dokter yang
dianggap mampu menanganinya, namanya Dr. Mark. Dokter itulah yang dapat
menyembuhkannya. Tapi wanita itu tidak mampu untuk membayar Dr. Mark, disamping itu
Dr. Mark tinggal di kota lain yang jauh dari tempatnya tinggal.

Kemudian wanita itu melanjutkan, "Sejauh ini memang Tuhan belum menjawab doaku.
Tetapi suatu hari nanti Tuhan akan memberikan jalan keluar, dan aku tidak akan membiarkan
kekuatiranku mengalahkan imanku". Kagum dan tak mampu berkata apaapa, Dr. Mark
berlinangan air mata. Ia berbisik: "God is great... Tuhan Maha Besar."

Lalu ia mengingat kembali rangkaian peristiwa yang ia alami, ada kerusakan teknis di
pesawat, badai yang melanda, ia tersesat di jalan, dan semuanya ini terjadi karena Tuhan
tidak hanya menjawab doa wanita tersebut. Tetapi juga memberinya sebuah kesempatan
untuk keluar dari dunia yang materialistis dan memberikan sebagian keahliannya dan
waktunya untuk orang-orang miskin yang sedang putus asa, yang sama sekali tidak memiliki
apapun selain doa-doa yang kaya yang diimaninya jawaban yang pasti.

5. Tokoh utama teks inspiratif di atas adalah...

Seorang dr
Dr. Mark
Resepsionis
Wanita

Jawaban: B

6. Berikut ini adalah peristiwa yang dialami oleh tokoh utama, kecuali ....

Pesawat yang ditumpanginya itu harus mendarat darurat


Mendapat informasi bahwa penerbangan berikutnya baru ada sepuluh jam kemudian.
Tiba-tiba cuaca menjadi buruk dan sebuah badai besar terjadi.
Menerima undangan untuk menghadiri sebuah konferensi

Jawaban: D

7. Penyebab tokoh utama tersesat saat mengemudikan mobil menuju lokasi konferensi adalah
....

Karena hujan sangat deras


Karena sempat beristirahat cukup lama
Melewati belokan yang seharusnya ia ambil.
Tidak tahu jalan mana yang harus dilewati

Jawaban: C

1. Cermati kalimat berikut!

Para orang tua memberikan ... kepada anak-anaknya agar menjadi anak yang berguna bagi
bangsa dan negara, demikian pula para guru di sekolah memberikan ... agar siswa siswinya
mau belajar supaya bisa lulus SNMPTN.

Untuk mengisi bagian yang rumpang menggunakan kata baku yaitu ....

A.nasihat, motipasi

B.nasehat, motipasi

C.nasehat, motivasi

D.nasihat, motivasi

E.nasihat, motifasi

2. Bacalah pengumuman penerimaan tenaga kerja berikut!

PENGUMUMAN

PT Indo Garmen membutuhkan segera seorang sekretaris, maksimal 30 tahun, pendidikan


minimal S1, diutamakan berpengalaman.Lamaran dikirim ke alamat kami jalan Aceh 124,
Bandung.

Berdasarkan pengumuman di atas, kalimat pembuka surat lamaran pekerjaan yang tepat
adalah ...

A. Sesuai pengumuman PT Indo Garmen yang beralamat di Jalan Aceh 124 membutuhkan
segera tenaga sekretaris, dengan ini, saya ingin mengajukan lamaran pekerjaan pada
perusahaan Bapak.

B. Sehubungan dengan pengumuman bahwa PT Indo Garmen yang beralamat di Jalan Aceh
124 membutuhkan segera tenaga sekretaris, dengan ini, saya ingin mengajukan lamaran.
C. Setelah ada pengumuman bahwa PT Indo Garmen yang beralamat di Jalan Aceh 124
membutuhkan segera tenaga sekretaris, maka ....

D. Berdasarkan pengumuman PT Indo Garmen yang beralamat di Jalan Aceh 124 Bandung
membutuhkan segera tenaga sekretaris, S1, dan sudah berpengalaman. Dengan ini, saya
mengajukan lamaran pekerjaan pada perusahaan Bapak.

E. Nama saya, Indah Susanto, ingin mengajukan lamaran pekerjaan pada perusahaan yang
Bapak pimpin.

Bacalah kutipan teks berikut dengan cermat untuk menjawab soal nomor 2 dan 3

2. Teks tersebut merupakan kutipan teks laporan percobaan bagian ....

A. kajian pustaka
B. hasil percobaan
C. prosedur kerja
D. pembahasan hasil percobaan

Jawaban: B

3. Berdasarkan data dalam tabel, setelah dimasukkan ke dalam larutan gula 15 persen, berat
kentang mengalami ....

A. kenaikan sebesar 5 gram


B. penurunan sebesar 5,3 gram
C. kenaikan sebesar 35 gram
D. penurunan sebesar 0,4 gram

Jawaban: B

Bacalah paragraf-paragraf berikut dengan cermat!


Melalui media cetak dan elektronik kita dapat menyaksikan bagaimana para pengedar
narkoba membangun jaringan secara intensif. Akibatnya, peredaran narkoba begitu
merebak hingga ke daerah-daerah pelosok bahkan sampai ke lembaga pemasyarakatan.
Mungkin banyak di antara hadirin yang belum memahami isu globalisasi. Untuk itu, kali ini
saya ingin mengajak hadirin untuk mengingat kembali beberapa pola hidup yang dapat
mengakibatkan globalisasi, apa pengaruhnya bagi kita semua, dan bagaimana kita
menghadapi pengaruhnya.
Idealnya, biaya pembangunan di suatu negara harusnya dibiayai oleh rakyat di negara
tersebut melalui setoran pajak. Inilah mengapa dikatakan pajak sebagai harga diri bangsa.
Karena pajak sebagai harga diri bangsa, seharusnya setiap wajib pajak merasa bangga
setelah melakukan kewajibannya kepada negara.
Langkah pertama adalah dengan memasukkan air hangat dengan takaran sesuai selera.
Selanjutnya, mencelupkan teh gantung ke dalam gelas berisi air hangat. Jangan lupa
memasukkan gula secukupnya kemudian aduk hingga larut. Teh siap di sajikan.

5. Paragraf-paragraf tersebut termasuk kutipan teks pidato persuasif, kecuali yang ditandai
dengan nomor ....

A. (1)
B. (2)
C. (3)
D. (4)

Jawaban: D

Bacalah teks berikut dengan cermat untuk menjawab soal nomor 6, 7, 8!

....
Untuk menghindari peredaran dan penyalahgunaan narkoba di kalangan peserta didik,
kesadaran dan pengetahuan tentang bahaya narkoba perlu ditanamkan. Banyak cara yang
dapat dilakukan, baik melalui penyampaian materi, forum diskusi, maupun dalam bentuk
pemberian tugas-tugas sekolah.Pemberian tugas sekolah dapat dilakukan guru dalam bentuk
tugas membuat poster, cerpen, atau tulisan yang bertema anti narkoba.
Dengan cara-cara di atassecara tidak langsung para peserta didik akan mengenal dampak
penyalahgunaan narkoba. Melalui pengenalan itu, terbentuklah pola pikir untuk menjauhi
penggunaan narkoba pada diri peserta didik.
....

6. Kutipan teks pidato tersebut bertopik ....

A. peredaran dan penyalahgunaan narkoba di indonesia


B. pemberantasan mata rantai peredaran narkoba
C. antisipasi penyalahgunaan narkoba di kalangan peserta didik
D. pemberlakuan hukuman bagi pengedar narkoba

Jawaban: C

7. Kutipan teks pidato tersebut bertujuan mengajak pembaca/pendengar untuk ....


A. membantu polisi dalam pemberantasan peredaran narkoba
B. mengantisipasi penyalahgunaan narkoba di kalangan peserta didik
C. mengajukan usul pemberlakukan hukuman seberat-beratnya kepada pengedar narkoba
D. berupaya aktif dalam upaya penanganan penyalahgunaan narkoba

Jawaban: B

8. Alasan orator (penyampai pidato) menghindari peredaran dan penyalahgunaan narkoba


dengan cara pemberian tugas-tugas sekolah adalah ....

A. mengenalkan cara mengatasi bahaya narkoba kepada peserta didik


B. menanamkan sikap kerja sama peserta didik dalam pemberantasan narkoba
C. membentuk pola pikir peserta didik menjauhi penggunaan narkoba
D. mengefektifkan pembelajaran di kelas agar mencapai hasil belajar yang maksimal

Jawaban: C

Bacalah teks berikut dengan cermat!

(1) Apa yang telah dilakukan oleh para pendahulu kita sangatlah penting untuk diteladani.
(2) Mereka sadar bahwa kaum mudalah yang akan menentukan nasib bangsa ini. (3) Untuk
itulah, kita sebaga pemuda harus meneruskan semangat persatuan mereka. (4) Ayo, wahai
pemuda harapan bangsa, kita songsong masa depan yang lebih cerah untuk nusa dan bangsa
kita!

9. Kalimat emotif pada teks tersebut ditandai dengan nomor ....

A. (1)
B. (2)
C. (3)
D. (4)

Jawaban: A

Bacalah kalimat-kalimat berikut!

Bapak dan Ibu, sekalian, izinkanlah saya menyampaikan pidato singkat tentang peranan
orang tua dalam pembentukan karakter anak.
Intoleransi dan radikalisme harus kita jauhkan dari pikiran putra-putri kita karena akan
merusak masa depan mereka dan bangsa kita tercinta.
Untuk bisa meraihkesuksesan, kita harus mampu mengubah potensi dalam diri kita menjadi
kompetensi yang diharapkan.
Ketika dunia usaha menjadi pilihan hidup, kita harus siap secara mental menghadapi
keberhasilan maupun kegagalan.

10. Kata bersinonim terdapat pada kalimat yang ditandai dengan nomor ....

A. (1)
B. (2)
C. (3)
D. (4)

Jawaban: C

Bacalah teks berikut dengan cermat!

(1) Pendidikan merupakan elemen penting bagi seluruh kehidupan manusia. (2)
Keberadaannya sangat membantu kemajuan bangsa. (3) Namun, di era globalisasi ini
kondisi pendidikan di Indonesia sebagian masih memprihatinkan. (4) Fasilitas pendidikan
yang layak bagi mereka yang berada di daerah terpencil belum terpenuhi.

Kalimat aktif pada teks tersebut ditandai dengan nomor ....

A. (1)
B. (2)
C. (3)
D. (4)

Jawaban: B

Bacalah paragraf berikut dengan cermat untuk menjawab pertanyaan 12 dan 13!

(1) Malam ini Vina tengah menengadahkan kepalanya, berusaha menyapa bulan. (2)
Kendati penantiannya telah habiskan lebih dari separuh kesabarannya, bulan tak kunjung
membalas sapanya. (3) Rupanya, awanlah yang menghalangi terangnya bulan. (4)
Sebenarnya, malam ini terasa lebih dingin dari malam-malam sebelumnya. (5) Namun, entah
mengapa dingin itu tak ia rasakan sedikit pun. (6) Mungkin pikiran tentang kejadian tadi
siang yang hilangkan rasa itu.

12. Majas personifikasi terdapat pada kalimat yang ditandai dengan nomor ....

A. (1) dan (2)


B. (3) dan (4)
C. (4) dan (5)
D. (5) dan (6)

Jawaban: A

13. Kalimat ekspresif yang menggambarkan ketidaksabaran tokoh ditandai dengan nomor ....

A. (2)
B. (3)
C. (5)
D. (6)

Jawaban: A

Bacalah teks berikut dengan cermat untuk menjawab soal nomor 14, 15, 16!
(1) Tanah di pekuburan umum itu masih basah ketika para pentakziah sudah pulang.
Sementara Ogal masih duduk sambil sesekali menyeka air matanya. Ibu yang selama ini
paling dia hormati dan cintai, tadi malam telah menghadap Tuhan Yang Maha Esa.

(2) ....
“Maksud Ibu?” Ogal tidak mengerti.
“Ya, rupanya engkau ditakdirkan untuk aku asuh dan menjadi anak kami. Tetapi kami
bertekad untuk menjadi orang tuamu, bukan sekedar orang tua asuh.”
....
Sebenarnya Ogal masih ragu-ragu, apakah dia akan ikut Bu Tutik atau bertahan hidup
dengan mandiri. Jika dia ikut Bu Tutik, tentu tidak dapat bekerja seperti ketika ia masih
hidup bersama ibunya. Hal itu menjadikannya manja. Tetapi jika menolak kebaikan Bu
Tutik, terasa tidak enak. Pengorbanan Ibu Guru itu sudah sedemikian besarnya. Dari
pengalaman hidupnya selama ini, banyak hal yang dapat Ogal petik. Ia biasa bekerja keras,
tidak suka menggantungkan pada orang lain. Ia juga biasa hidup prihatin sehingga tidak
suka berfoya-foya.

(3) “Tapi, saya tidak enak kalau menganggur, Bu!”


“Di rumahku engkau tidak mungkinmenganggur. Engkau bisa belajar
menggunakankomputer, mengetik, nonton TV, danmemelihara kebun.”
“Tapi, saya akan tidak bekerja, Bu!”
“Pada hakikatnya engkau bekerja juga.Memelihara kebun atau membantuku dirumah juga
bekerja.”
“Jadi, tidak harus menjajakan kue, Bu?”Bu Tutik mengangguk.
“Kalau begitu, tolong carikan pekerjaanyang bisa saya lakukan.”
Bu Tutik tersenyum.
“Jangan khawatir.”

(4) Bu Tutik ternyata dapat memenuhiharapan Ogal. Banyak pekerjaan yang dapatdilakukan
Ogal. Misalnya, memelihara kebun mangga, mencatat keluar masuknya barang, dan
sebagainya. Kali ini Ogal tidak kalah sibuknya dengan sewaktu berada di desa nelayan.
Bahkan, mungkin boleh dikatakan sangat sibuk. Pekerjaan di rumah Bu Tutik tidak hanya
satu, melainkan sangat banyak. Walaupun begitu, Bu Tutik tidak pernah memaksa Ogal
untuk bekerja. Semua itu hanya semata-mata menuruti keinginan Ogal.

14. Makna ungkapan menghadap Tuhan Yang Maha Esa pada bagian (1) teks tersebut
adalah ....

A. memohon dan berdoa


B. meninggal dunia
C. berserah diri kepada Tuhan
D. merasa tidak berdaya

Jawaban: B

15. Bukti karakter Ogal adalah seorang pekerja keras sesuai dengan bagian (3) teks tersebut
adalah ...

A. Ogal tidak mau meninggalkan pekerjaan lamanya sebagai penjaja kue.


B. Ogal memenuhi permintaan Bu Tutik untuk belajar lebih giat lagi.
C. Ogal tidak ingin dirinya menjadi beban Bu Tutik.
D. Ogal meminta pekerjaan untuk dirinya kepada Bu Tutik.

Jawaban: D

16. Pada bagian (2) teks tersebut pengarang menggambarkanOgal sebagai tokoh yang tidak
mau hidup berfoya-foya, yaitu dengan cara ....

A. melalui perkataan tokoh Ogal


B. melalui ekspresi tokoh Bu Tutik
C. pengarang menjelaskan secara langsung
D. melalui perasaan tokoh Ogal

Jawaban: C

Bacalah kalimat-kalimat berikut dengan cermat!

Sebagian besar pihak yang menjadi sponsor Asian Para Games 2018 sempat mengira ajang
tersebut sama dengan Asian Games.
Dalam standar olimpiade saat ini, setiap medali emas yang menjadi hadiah bagi para atlit
pemenang mengandung kadar emas sebesar 6 gram yang dicampur dengan perak sebesar
494 gram.
Lagu“Laskar Pelangi” memberikan inspirasi bahwa ”mimpi” bisa menjadi ”kunci” yang
bisa digunakan untuk membuka harapan kita untuk ”menaklukan dunia” yang kemudian
menjadi penyemangat untuk meraih cita-cita.
BNPB menyemprotkan cairan desinfektan melalui udara dengan metode water boming di
titik lokasi gempa likuifaksi, Balaroa dan Sigi, Sulawesi Tengah, Kamis, 18 Oktober 2018.

17. Kalimat yang termasuk teks tanggapan ditandai dengan nomor ....

A. (1)
B. (2)
C. (3)
D. (4)

Jawaban: C

Bacalah kalimat berikut dengan cermat!

Album yang dikemas dengan dominan warna hitam ini menyuguhkan dua instumen. Pertama
adalah “Sunset” didominasi oleh gitar. Nuansa ini sekilas terdengar ala Kings of
Convenience ini. Sementara itu, pada lagu kesembilan kita dibawa mendengarkan dentingan
piano yang menenangkan setelah diajak menggoyangkan tubuh pada lagu sebelumnya,
“Cadilac Model”.

18. Kata-kata yang memerlukan penjelasan tambahan (kata yang dianggap sukar) dalam teks
tersebut adalah ....

A. album
B. nuansa
C. Sunset
D. Kings of Convenience

Jawaban: D

Perhatikan kalimat berikut!

Jika Anda adalah pecinta musik sekaligus penikmat fotografi, di album ini kita bisa
menikmati keduanya sekaligus karena Atlesta mengemas lirik-lirik dalam album Sensation
itu ke dalam 14 lembar foto menarik. Sayangnya lirik-lirik tersebut tidak semuanya tercetak
dengan baik, dengan huruf handwriting yang cukup sulit untuk dibaca.

19. Komentar yang sesuai dengan cuplikan teks tanggapan tersebut adalah ....

A. Penggunaan istilah asing yang terlalu banyak mengganggu pemahaman.


B. Banyak ditemukan penulisan huruf kapital dan tanda baca yang tidak tepat.

C. Menggunakan kalimat yang tidak efektif sehingga sulit dipahami.


D. Pernyataan banyak menggunakan ungkapan yang sulit diketahui maknanya.

Jawaban: D

Perhatikan kalimat berikut!

(1) Alat dan bahan


(2) Tujuan
(3) Hasil Pengamatan
(4) Prosedur / Cara Kerja
(5) Simpulan

20. Sistematika laporan percobaan yang benar adalah..


A. (2), (1), (3), (4), (5)
B. (2), (1), (4), (3), (5)
C. (2), (1), (5), (3), (4)
D. (2), (3), (4), (1), (5)

Jawaban: B

1. Penulisan kata bentukan yang tidak tepat terdapat pada kalimat…

A. Pada masa ORBA, Indonesia terkenal dengan swa sembada pangannya.

B. Pesawat itu telah lepas landas.

C. Amerika Serikat adalah negara adikuasa.

D. Zaman prasejarah memberi inspirasi pada kehidupan sekarang.

2. Kalimat yang menggunakan kata tidak baku adalah…


a. Indonesia terus bekerja keras untuk memajukan teknologinya.

b. Penulis sudah mengkaji hasil penelitiannya dengan seksama.

c. Indonesia belum juga menemukan langkah konkret untuk mengatasi masalah


pengangguran.

d. Untuk menjaga kedaulatannya, Indonesia menambah kuantitas Armada di Kawasan


Ambalat.

Anda mungkin juga menyukai